Anda di halaman 1dari 381
TEORIA ELEMENTAR DOS NUMEROS Edgard de Alencar Filho Ex-professor de GEOMETRIA ANALITICA e CALCULO da Escola Militar do Realengo e de MATEMATICA da Escola Preparatéria de Sio Paulo CIP-Brasil. Catalogacdo-na-Fonte Camara Brasileira do Livro, sP Alencar Filho, Edgard de, 1913- A353t Teoria elementar dos nimeros/Eagard de Alencar Filho. -- Sao Paulo : Nobel, 1981. Bibliografia. ISBN 35-213-0040-9 1, Nimeros - Teoria I, Titulo. 17. CDD=-512.31 1s. S12. 31-0333 Indices vara catalogo sistematico: 1. Teoria dos nimeros 512.31 (17.) 512.7 (13.) INDICE CAPITULO 1 ~ NUMEROS INTEIROS: NOGOES FUNDAMENTAIS 1.1 Nameros inteiros ....... 1.2 Propriedades dos inteiros F 1.3 Valor absoluto de um inteiro 1.4 Fatorial ss 1.5 Namero binomial .. m 1.6 Nimeros binomiais complementares . : 1.7 Mimeros binomiais consecutivos Exercicios . caPiTuLo 2 - INDUGAO MATEMATICA 2.1 Elemento minimo de um conjunto de inteiros 2.2 Principio da boa ordenacao . as 2.3 Prineipio de indugao finita ... a 2.4 Indugdo matematica ..... 2.5 2.6 Outras formas de indugao matendtica . Exercicios ...... detec eeeee seeeee CAPITULO 3 - SOMATORIOS E PRODUTORIOS Somatorios . Propriedades dos somatorios . Somatorios duplos .. Produtérios ...... Propriedades dos produtdrios Teorema do bindmio Triangulo de PASCAL Propriedades do triangulo de PASCAL .. Nimeros triangulares .............. Exercicios ....... teeee PELYLLULUY bo mUTAueone 13 14 16 13 20 21 21 25 31 32 34 35 37 40 45 47 49 50 52 53 55 57 59 61 65 CAPITULO 4 - DIVISIBILIDADE 4.1 Relagao de divisibilidade emZ..., 63 4.2 Conjunto dos divisores de um inteiro . sy TL 4.3 Divisores comuns de dois inteiros .. 72 4.4 Algoritmo da divisao 74 4.5 Paridade de um inteiro ..... - B IERQEGIGLOB! 5 Siaeoeee iene nase mig we. 80 CAPITULO 5 - MAXIMO DIVISOR COMUM 5.1 Maximo divisor comum de dois inteiros 84 5.2 Existéncia e unicidade do mde 385 5.3 Inteiros primos entre si ....... 89 5.4 Caracterizacao do mdc de dois inteiros . 93 5.5 Mde de varios inteiros ; 94 Exercicios .........., ry 96 CAPITULO 6 - ALGORITMO DE EUCLIDES. MINIMO MULTIPLO comuM -1 Algoritmo de EUCLIDES ............ +2 Maltiplos comms de dois inteiros . +3 Minimo miltiplo comum de dois inteiros 5 4 ac 5 Relagao entre o mdc eo me Mme de varios inteiros . Exercicios .........4. DAKAR CAPITULO 7 - NOMEROS PRIMOS 7.1 Nimeros primos e compostos .. 116 7.2 Teorema fundamental da Aritmatica LL 7.3. Formulas que dao primos ., +125 7.4 Crivo de ERATOSTENES ... +126 7.5 Primos gémeos aes Sa ane: 7.6 Sequéncias de inteiros consecutivos compostos128 7.7 Conjectura de GOLDBACH ...... vee eee e129 7.8 Método de fatoragao de FERMAT . ald, Exercicios ..... alae CAPITULO 8 - EQUACOES DIOFANTINAS LINEARES 8.1 Generalidades .........0., oa «137 8.2 Condigao de existéncia de solugao .. 138 8.3 Solugoes da equagao ax + by =c e139 Exercicios ., 146 CAPITULO 9 ~ CONGRUENCIAS 9.1 Inteiros congruentes 148 9.2 Caracterizagao de inteiros congruentes . 150 9.3. Propriedades das congruéncias . 152 9.4 Sistemas completos de restos . 159 Exercicios 162 CAPITULO 10 - CONGRUENCIAS LINEARES 10.1 Generalidades .. 165 10.2 Condigao de existéncia de solugao colt 10.3 Solugdes da congruéncia ax = b (mod. m) .... 168 10.4 Resolugao de equagoes diofantinas lineares por congruéncias .... Es ee DS 10.5 Inverso de um inteiro . 178 Exercicios sseseeeres 179 CAPITULO 11 - SISTEMAS DE CONGRUENCIAS LINEARES 11.1 Generalidades ....... te 181 11.2 Teorema do resto chinez 133 Exercicios ....seeeeeees 191 CAPITULO 12 - TEOREMAS DE FERMAT E WILSON 12.1 Teorema de FERMAT . 193 12.2 Teorema de WILSON . 199 Exercicios 203 CAPITULO 13 - DIVISORES DE UM INTETRO 13.1 Divisores de um inteiro . 205 13.2 Numero de divisores 208 13.3 Soma de divisores 212 13.4 Notagao 13.5 Produto de Exercicios CAPITULO 14 - FUNGOES ARITMETICAS 14.1 Conceito de fungao aritmética ....- 14.2 Fungoes aritmeticas miltiplicativas 14.3. Fungdo de MOBIUS .+e++eesesee 14.4 Fungoes aritméticas mltiplicativas completas 14.5 Fungao maior inteiro .....-.+sseseeeee ee waa 14.6 Formula de inversao de MOBIUS Exercicios CAPITULO 15 - FUNGAO E TEOREMA DE EULER 15.1 Fungao de EULER .... 15.2 Calculo de $(n) 15.3 Propriedades da fungdo de EULER . 15.4 Teorema de EULER ..... 15.5 Relagao entre as funcoes Exercicios ..seesseseeee CAPITULO 16 — NOMEROS PERFELTOS 16.1 Numeros perfeitos .... 16.2 Nimeros multiperfeitos . 16.3 Niimeros amigos ....... 16.4 Nimeros deficientes e abundantes 16.5 Nameros de MERSENNE 16.6 Nameros de FERMAT . Exercicios CAPITULO 17 ~ NOMEROS DE FIBONACCI 17.1 Sequéncias recorrentes . 17,2 Sequéncia de FIBONACCI . 17.3. Somas de niimeros de FIBONACCI 17.4 Soma dos quadrados de nimeros de FIBONACCT. 17.5 Identidades entre niimeros de FIBONACCI . 17.6 Propriedades dos niimeros de FIBONACCI Exercicios ........ CAPITULO 18 - TERNOS PITAGORICOS 18.1 Conceito de terno pitagdrico 18.2 FOrmulas que dao ternos pitagoricos . 18.3 Ternos pitagéricos primitivos ... 18.4 Propriedades dos ternos pitagdricos . Exercicios . CAPITULO 19 - CLASSES RESIDUAIS 19,1 Conceito de classe residual . 19.2 Propriedades das classes residuais 19.3 Conjunto das classes residuais . EBxercicios ..4.0+a0we « . 233 235 237 » 240 244 249 255 258 265 266 267 1 263 272 274 278 279 280 234 285 237 292 296 297 299 301 305 303 310 312 417 CAPITULO 20 - REPRESENTACAO DOS INTEIROS EM OUTRAS BASES. CRITERIOS DE DIVISIBILIDADE Representacao dos inteiros em outras bases.. Critérios de divisibilidade .... Critério de divisibilidade por 9 . Critério de divisibilidade por 11. Exercicios . 8sss ReRY Pera CAPITULO 21 - RAIZES PRIMITIVAS 21.1 Ordem de um inteiro médulo n . 21.2 Raiz primitiva de um inteiro Exercicios ...., 318 323 325 327 323 PREFACIO 0 conceito de nimero desempenha papel fundamental na eivi lizagao moderna, @ as ciéncias mais avangadas sao prec samente aquelas que mais empregam a linguagem dos name- ros. Alias, a Matematica vem se tornando cada vez mais ne Cess@ria em todos os ramos do conhecimento humano, desde a Fisica 4 Biologia, bem como nas ciéncias sociais(Econo- mia, Finangas, etc.). Até mesmo na Psicologia tem sido utilizada com éxito. 0 Livro que ora publicamos pela conceituada "Editora NO- BEL" tem por iinica finalidade fornecer aos estudantes as primeiras nogoes da “Teoria dos Nimeros". Desta forma pro curo contribuir, na medida das minhas forcas, para a me- lhoria do ensino da Matematica em nosso Pais, como,alias, venho fazendo ha muitos anos. Em cada um dos Capitulos do livro os conceitos sao defini, dos com precisdo e as proposicdes fundamentais pertinen= tes sdo todas demonstradas e, logo em seguida, exemplifi- cadas, tendo em vista o leitor que pela primeira vez abor da esta fascinante parte da Matematica pura. Além disso, cada Capitulo termina por um conjunto de Exercicios, numé ricos e tedricos, cujas respostas sao dadas no final do livro, os quais permitem completar a aprendizagem. Edgard de Alencar Filho capitulo 1 NUMEROS INTEIROS NOCOES FUNDAMENTAIS 1.1 NOMEROS INTETROS Os nimeros intetros ou apenas os intetros sio: »73,-2,-1,0,1,2,35... cujo conjunto representa-se pela letra Z, isto é: 2m Lenn -2,-1,0,1,2,3,... 3 Neste conjunto 2 destacam-se os seguintes euboonjuntos: (1) Conjunto 2* dos intedros nao nuloe (# 0): m= {xeZ|x#O}={41, +2, +3, 0.3 (2) Conjunto 2, dos intetros nao negativos (20): 2, ={x€Z[x>0}={0,1,2,3, ...} (3) Conjunto Z_ dos intetros nao positives (<0): 2_ = xe€ 2[x<0}= {0,-1,-2,-3, ...} 14 (4) Conjunto Z* dos intetros posttivos (> 0): 2 = {x€Z[x>0}={1,2,3,...} (5) Conjunto 2* dos tntetros negativos (<0): 2k ={xé2|x<0}={-1,-2,-3, ...} Os intetros postttvos sao também denominados intetros na- turats e por isso o conjunto dos inteiros positivos @ ha- bitualmente designado pela letra N(N = 2%), 1.2 PROPRIEDADES DOS INTEIROS © conjunto Z dos inteiros mmido das operagées de adigdo (+) e multiplicagao(.) possui as propriedades £ undamen- tais que a seguir enumeramos, onde a, bec sao intei- Tos quaisquer, isto @, elementos de 2: qa) (2) (3) (4) (5) (6) atb=bta (a+b) +e= O+asaie va = (-la a(b +c) = ab 0.a=0,e se e e ab =ba as(b +c) e lanza (abe = a(be) an-az=at (-a) =0 + ac ab = 0, entao a= 0 ou b=0. Tanbém existe una "relagdo de ordem" entre os inteiros, representada pelo sinal "<(menor que)", que possui as se guintes propriedades: (7) Se (8) Se (9) Se (10) Se (11) se 15 a#0, entao a<0 ou Ob2c, etc. | 1.3 VALOR ABSOLUTO DE UM INTETRO Definigdo 1.1 Chama-se valor abeoluto de um inteiro a,0 inteiro que se indica por |a|, e tal que a se az0 “a se a<0 Assim, p.ex [3 = 3 |-5] = -(-5) = 5 Consoante a definigao de |a|, para todo inteiro a, temos: 0 valor absoluto |a| de um inteiro a também pode ser de~ finido pelas igualdades: laj=Va, la] =max(-a,a) 17 onde denota a raiz quadrada nao negativa de a e max(-a,a) indica o maior dos dois inteiros -a e a. Assim, p.ex.: 1 -4[ = V ay? V6 2g [| -6! = max(-6,6) = 6 Teorema 1.1 Se ae b sao dois inteiros, entao: lab] = [al . |b] Demonstracao: Com efeito: Jab] = V (ab)? = Vary? eV a? , Vp? Teorema 1.2 Se ae b sao dois inteiros, entao: Ja + bl < fal+ |b] Demonstraga Com efeito, pela definicao de |a|, temos: -lal<¢adq@lal, - [b] <[bl< |b] Somando ordenadamente estas desigualdades, obtemos: (lal + Ib])0 ek dois inteiros tais que O com 1¢k (nt)*. Decompor o inteiro 565 numa soma de cinco inteiros im pares consecutivos. Achar todas as solugdes inteiras e positivas da equa- gio: (e+ L(y + 2) = dxy 26 10. Achar un inteiro positive de dois algarismos-que “se Ja igual ap quadruple da soma dos seus algarismos. Li. Achar o menor e o maior inteiro positive de a algaris mos. 12. Resolver a equaga (x + 2)! = 72.21, 13. Resolver a equagao: 14. Demonstrar: 15. Achar todas as solugoes inteiras e positivas da equa~ ior x? - y2 = 9g 16. Verificar que o quadrado de um inteiro nio pode termi “nar em 2, 3, 7 ou 8. 17. Reconstituir as adigoes: (a) 3* 76+ 244445 * 28 = 17838 (b) 5 * 23 + 40 * * + 1269 = 1 * 927 18. Reconstituir as subtracoes: (a) 1 * 256 ~ 431 * = a9 #6 (b) 63 * 1-43 % 25 % 96 19, 20. 21. 22, 23. 24. 27 © produto de um inteiro positivo de trés algarismos por 7 termina a direita por 638. Achar esse inteiro. Determinar quantos algarismos se empregam para nume- Tar todas as paginas de um livro que tem 2748 pagi- nas. Reconstituir as multiplicacdes: (a) 435 (b) 2*03 *6 #2 *6*0 4806 1305 14*1%* i 1*660 148986 Caleular a soma dos trés matores inteiros de, respec- tivamente, trés, quatro e cinco algarismos. Determinar a diferenga entre o maior inteiro com seis algarismos diferentes e 0 maior inteiro com cinco al- garismos também diferentes. i Um livro tem 1235 paginas. Determinar o numero de ve- zes que o algarismo 1 aparece na mumeracao das pagi- nas deste livro. 28 25. 26, 27. 28. bok 30. 31. 32. Reconstituir as divisoes: (a) # ae Be ae () *O*eL| kHR *91 32 59 6 ae Mostrar que o produto de quatro inteiros consecutivos, aumentado de 1, @ um quadrado perfeito. A soma dos quadrados de dois inteiros 2 3332 e um de- les @ 0 quadruplo do outro. Achar os dois inteiros. Sejam a e b dois inteiros. Demonstrar: max(a,b) = (a +b + [a - b])/2 min(a,b) = (a +b = Ja - b/)/2 Determinar o inteiro n>1 de modo que a soma 1i+ 204 3! +...+ af seja um quadrado perfeito, A média aritmética de dois inteiros positives €5ea média geométrica é 4. Achar os dois inteiros. Achar os cinco inteiros positivos consecutivos cuja soma dos quadrados @ igual a 2010. 0 resto por falta da raiz quadrada de um inteiro posi tivo € 135 eo resto por excesso @ 38. Achar esse in- teiro. 29 xi + 3(x = 2)! — 31 33, Resolver a equacio: PIGS 34, Achar o inteiro que deve ser somado a’cada um dos in- teiros 2, 6 e 14 para que, nesta ordem, formen uma proporeao continua. 35. Mostrar que o produto 12345679 x 9 x k, sendo k#0 um algarismo, @ kkk. kkk. kkk. 36. Achar o valor minimo de uma soma de 10 inteiros posi- tivos distintos, cada um dos quais se escreve com trés algarismos. 37. Mostrar que o produto 37037037 x 3.x k, sendo k #0 um algarismo, @ kkk. kkk.kkk. 38. Um estudante ao efetuar a multiplicagao de 7432 por tm certo inteiro achou o produto 1731656, tendo troca do, por engano, o algarismo das dezenas do multiplica dor, tomando um 3 em vez de 8. Achar o verdadeiro pro duto. 39. Achar o menor inteiro cujo produto por 21 @ um intei- ro formado apenas com algarismos 4. 40. Escreve-se a sequéncia natural dos inteiros positi- vos, sem separar os algarismos: 123456789101112131415... Determinar: 30 41. 42, 43. 4a. 45. 46. 47. (a) 0 4359 algarismo que se escreve; (b) 0 17569 algarismo que se escreve; (c) 0 123879 algarismo que se escreve. Escreve~se a sequéncia natural dos inteiros positi- vos pares, sem separar os algarismos: 24681012141618... Determinar o 25749 algarismo que se escreve. Reconstituir as multiplicagoes: (a) kaa ) ne a4 RRS ARR RE a eas teat ak 90329 36733 Mostrar que o produto de dois fatores entre 10 e 20 @ © décuplo da soma do primeiro com as unidades do se- gundo, mais o produto das unidades dos dois. Achar o menor inteiro positivo que multiplicado por 33 d@ um produto cujos algarismos sao todos 7. Os inteiros ae b sao tais que 4 (a€A e (YxEA) (a12} temo elemento minimo, que € 13 (minA = 13), porque 13€A e 13<¢x pa ra todo x€A, Exemplo 2.3 0 conjunto Z_ = {0,-1,-2,-3,...} dos intet- P08 nao positives ndo tem o elemento minimo, porque nao existe a€Z_ tal que a J mina Exemplo 2.5 0 conjunto A = {1,3,5,7,...} dos intetros posttivos impares @ um subconjunto nao vazio de 2, #acz,). Logo, pelo "Principio da boa ordenagao", A possuio ele- mento minimo (minA = 1), Exemplo 2 O conjunto P = {2,3,5,7,11,...} dos intet- vos primos @ um subconjunto nao vazio de 2.06 # PCZ,), Logo, pelo "Prinetpto da boa ordenagéo", P possuio ele- mento minimo (minP = 2). Teorema 2.2 (de ARCHIMEDES) Se a eb sao dois inteiros positivos quaisquer, entao existe um inteiro positivo n tal que na2b. Demonstragao: Suponhamos que a eb sao dois inteiros positivos para os quais na11; Gi) sea=9 e b=5, entaon= 1, porque 1.9>5, 2.3 PRINCEPIO DE typuGKO FINTTA Teorema 2.3 Seja S um subconjunto do conjunto N dos in- teiros positivos (SCN) que satisfaz as duas seguintes condicoes: (1) 1 pertence a § (1€5); (2) para todo inteiro, positivo k, se kes, entéo k+1€s, Nestas condigdes, S$ @ 0 conjunto N dos inteiros positi- vos: S =N, Demonstrag! Suponhamos, por absurdo, que $ nao @ 0 conjunto N dos in- teiros positivos (S #N) e seja X o conjunto de todos os 35 inteiros positivos que nao pertencem a S, isto é@: K={x| xen e xéS}=n-s Entao, X @ um subeonjunto nao vazio de N(d # XCN) e, pe- lo “Principio da boa ordenagdo", existe o elemento minino X de X(mink = x,). Pela condigéo (1), 1€8, de modo que x >1e, portanto, - 1 ¢um inteiro positivo que nao pertence a X. Logo, Xy — L€S e, pela condicdo (2), segue-se que (x-D)41 = = X)€S, 0 que & uma contradigao, pois, x,€ X= x= S, is to pron »yfS Assim sendo, X= gegen. Consoante este "Principio de tndugao finita", o nico sub, conjunto de N que satisfaz 3s condigdes (1) e (2) 30 ped prio N. 2.4 INDUGAO MATEMATICA Teorema 2.4 Seja P(n) uma proposicdo associada a cada in teiro positivo n e que satisfaz as duas seguintes condi- goes: (1) PQ) @ verdadeira; (2) para todo inteiro positive k, se P(e) @ verdadeira, entao P(k+1) também @ verdadeira. Nestas condigoes, a proposicao P(n) é verdadeira para to- do inteiro positivo n. 36 Demonstragao Seja S 0 conjunto de todos os inteiros positives n para os quais a proposigao P(n) @ verdadeira, isto é: S={n€N | P(n) é verdadeira } Pela condigdo (1), P(1) @ verdadeira e, portanto, 1€S.Pe la condigao (2), para todo inteire positivo k, se k€S,en tao k+l€S. Logo, o conjunto $ satisfaz as condigoes (1) e (2) do "Princtpto de indugéo fintta" e, portanto, SeN, isto @, a proposigdo P(N) é verdadeira para todo inteiro positivo n. NOTA, 0 teorema 2.4 @ geralmente denominado "Teorema da ‘indugao matenatica" ou "Prinetpto de indugao matemittca rs ea “demonstragao de uma proposigao usando-se este teorema chama-se "demonstragdo por indugao matemattca" ou "demons tragdo por indugio sobre xn" Na "demonstragdo por indugdo matemdtica" de uma dada pro- posigao P(n) @ obrigatério verificar que as condicées (1) e (2) sao ambas satisfeitas. A verificagao da condicgao(1) @ geralmente mito facil, mas a verificagao da condicao (2) implica em demonstrar o teorema auxtliar cuja hipote- se é: H: proposigao P(k) @ verdadeira, kEN denominada "tipétese de indugdo", e cuja tese ou conelu- so é: 7 T: proposigao P(k+l) € verdadeira. 37 2.5 EXEMPLOS DE DEMONSTRACAO POR INDUGAO MATEMATICA Exemplo 2.7 Demonstrar a proposigao: Pla) 2143454 ..+ Qn=1) w02, Vaew Demonstraga (1) PQ) € verdadeira, visto que 1 = 12 (2) A hipdtese de indug&o & que a proposigao: P(e): L4+34+5 4... + (2-1) ken é verdadeira. Adicionando 2k+1 a ambos os membros desta igualdade, ob temas: L4+345 4.0.4 (2kel) + (2ke1) = m2 + (2K+1) = (Kt1)? e isto significa que a proposicao P(k+1) @ verdadeira. Logo, pelo "Teorema da indugdo matemitica", a proposi- sao P(n) & verdadeira para todo inteiro positivo n. Exemplo 2.8 Demonstrar a proposicao: Cee , aie *) ta 7+ 2.a* se? ---* aay ae Ven Demonstragao (1) P(L1) @ verdadeira, visto que “Tal * (2) & hipétese de indugao € que 3 proposicao: 1 k Eke) “were | KEN 1 zat @ verdadeira. Adieionando 757 Tea @ anbos os menbros desta igualda de, obtemos: 1 1 "Ete * Tei ceay 1 122% kel i Tz ea . = BL k+l © Cer) (47 +1) (k+2) +2 e isto significa que a proposigao P(k+1) é verdadeira.Lo go, pelo "Teorema da indugao matematica", a proposicao P(n) @ verdadeira para todo inteiro positivo a. Exeniplo 2.9 Demonstrar a proposi¢ao: Pa): 3/(7"-a), ¥ nen Demonstragao: (1) PQ) é verdadeira, visto que 3{ (2? - 1). (2) A hipétese de indugéo que a propdsicio: Pw): a] -1), ken @ verdadeira. Portanto: 2h < 9 wig, com d€z © que implica: 39 22 (+1) 2 42k Ts 2p gk i 4.2 eg 24th age 4.3q + 3 = 3(4q + 1) isto @, a Proposicao P(k+1) @ verdadeira. Logo, pelo "Teg vema da indugdo matemdtica”, a Proposicao P(n) @ verdadei ra para todo inteiro positivo n. Exemplo 2.10 Demonstrar a Proposigao: P(a): 2">n, Ynew Demoustracao: () PCL) @ verdadeira, visto que 2! = 251, (2) A hipétese de indupéo & que a proposicao: P(k): >, ken @ verdadeira. Portanto: 2.2%> 2% on Moses tone 2 que implica: 2! >4 41, isto 3, a Proposicao P(k+1) @ verdadeira. Logo, pelo "eorema da indugdo matematica", 2 proposicao P(n) & verdadeira para todo inteiro positivo 40 2.6 OUTRAS FORMAS DA INDUCAO MATEMATICA Teorema 2.5 Seja r um inteiro positivo fixo e seja P(n) uma proposigao associada a cada inteiro n>r e que sa- tisfaz Gs duas seguintes condicgdes: (1) P(r) é verdadeira; (2) para todo inteiro kr, se P(k) @ verdadeira, entao P(k+1) também @ verdadeira. Nestas condigoes, P(n) @ verdadeira para todo inteiro n2r. Demonstraga Seja S$ o conjunto de todos os inteiros positivos n para os quais a proposicgao P(r + n - 1) @ verdadeira, isto é: S={n€N |P(r +n - 1) @ verdadeira } Pela condigao (1), P(r) = P(r + 1 - 1) @ verdadeira, isto &, 1€S. £, pela condigao (2), se P(r +k ~-1) é verda- deira, entao: P(r +k - 1) +1) = P(r + (k +1) - 1) também é verdadeira, isto €, se k€S, entao k+1ES. Lo go, pelo "Principio de indugao finita", S € 0 conjunto dos inteiros positivos: $= N, isto é, a proposicao P(r +n - 1) @ verdadeira para’tedo n€N, ou seja, o que @ a mesma coisa, a proposigao P(n) @ verdadeira para todo inteiro n2r. 41 Exemplo 1 Demonstrar a Proposicao: Pn): 272ne2, Yada Demonstragao: : (1) P(3) @ verdadeira, visto que 32 =9>2.3+127. 42 (2) Suponhamos, agora, que & verdadeira a proposigao: P(e): k2> 2ke1 * k23 Entao, temos: 2 + (2k#1) > (241) + (2k41) (k+1)? > 2Gc+1)# 2k > 2(ke1) + 6 e, portanto: (er)? > 2¢ke1) + 1 isto @, a proposigao P(k+1) @ verdadeira. Logo,pelo teo- rema 2.5, a proposigao P(n) @ verdadeira para todo intei- ro n>3. Observe-se que a proposigao P(n) @ falsa paran=len-= = 2, pois, temos: 2 caret e 2e2241 Teorema 2.6 Seja P(n) uma proposicao associada a cada inteiro positive ne que satisfaz as duas seguintes condi goes: (1) P(1) @ verdadeira; (2) para todo inteiro positivo k, se P(1),P(2),...,P(k) sao todas verdadeiras, entao P(k+1) também é verda deira, 43 Nestas condigoes, a proposicao P(n) & verdadeira para to do inteiro positive n. Demonstragao : Seja $ 0 conjunto de todos os inteiros positives n para os quais a proposicao P(n) @ verdadeira, isto é: S={n€N |P(n) @ verdadeira } Suponhamos, por absurdo, que $#N e sejaXo conjunto de todos os inteiros positivos que nao pertencem a S » is~ to é: i Ka{x|xen e xds}en-s Entao, X @ um subconjunto nao vazio de Ne, pelo "Princd- pto da boa ordenagao", existe 0 elemento minimo ide x (minX = 3), Pela condigao (1), 1€S, de modo que j>1, e como j Zo menor inteiro positivo que nao pertence a S, segue-se que aS proposigoes P(1), P(2),...,P(j-1) sao todas verdadei- ras, Entao, pela condigao (2), a proposigao P(j) € verda deira e j€S, o que é uma econtradigao, pois, j€X, isto , j¢S. Assim sendo, S=N ea proposigao P(n) & ver- dadeira para todo inteiro positivo n. Teorema 2.7 Seja r um inteiro positivo fixo e seja P(n) uma proposicdo associada a cada inteiro n%r e que sa- tisfaz as duas seguintes condicdes: 4a (1) P(x) @ verdadeira; (2) para todo inteiro k>r, se P(m) @ verdadeira para todo inteiro m tal que rr. Demonstrag: Seja S o conjunto de todos os inteiros nzr para os quais a proposicao P(n) @ falsa, isto é: S=({n€nN|n>r e P(n) é falsa} Suponhamos, por absurdo, que S nao @ vazio (S #$). En tao, pelo "Principio da boa ordenagdo", existe o elemen- to minimo j de S(minS = j). Pela condigao (1), r¢S, de modo que j>r, e por conse- guinte P(m) @ verdadeira para todo inteiro m tal que r¢m02, YY nes () 2%>03, Vinzi0 (ay tout, y aps 2: (e) nt>n?,” Yn (f) nom, = yn2r6 Demonstrar por "indugao matematica"; (a2) 2) @Q™), Wnen (6) 6] @? =n), Yuen 46 (e) 5[(s"- 35, Vnen (a) 24 (571), Ynen (7/31, Ynew (£) 81977), Ynen Denonstrar que 10! - 99 - 10 @ um miltiplo de 81 pa ra todo inteiro positivo n. da #0 hae, 2.4 Demonstrar que > +e ea & um inteiro positive pa ra todo n€N. CAPITULO 3 SOMATORIOS & PRODUTORIOS 3.1 SOMATORTOS Sejamos n>1 inteiros 4) :895+++,4,. Para indicar, de modo abreviado, a soma a, + a) +...+ a, desses n intei- ros usa-se a notagao: que se 18: "somatorio de a; de lan". Em particular, para n = 2,3,...,temos: 2 3 Ele wie, © ay; i a, = isl z 2 isl A letra i chama-se o indice do somatério e pode ser subs- tituida por qualquer outra diferente de aeden- é@ um indice mo. E os inteiros 1 e n que figuram abaixo e aci ma da letra grega maiiscula £ (sigma) chamam-se respecti vamente limite inferior e Limite superior do indice i 438 0 niimero de pareelas de um somatério é sempre igual’ Z di- ferenca entre’os limites superior e inferior do seu Indi- ce mais uma unidade. Se men sao dois inteiros, com m¢n, entao, por defini gao: n - M7 nt Sued * oa + iar ism Exemplo 3.1 Temos: 7 E E Si=5.14+5.245.3+5.4+5.545.645.7 = isl = 5 +104 15 + 20 + 25 + 30 + 35 = 140 4 © (8§-3) = (8.1-3) + (8.2-3) + (8.3-3) + (8.4-3) = jel =5+13+ 21 + 29 = 68 8 Bke2k = 3.29 + 4.24 4 5,25 4 6.25 4 7.97 8.28 ke3 = 24 + 64 + 160 + 384 + 896 + 2048 = 3576 Exemplo 3.2 Temos: 6 ¥ 2+448+16+32+64= 5 2h isl 15 1+3+54...4292 £ (2) -1) j=l 49 3.2 PROPRIEDADES DOS SOMATORIOS a Teorema 3.1 = (a. Demonstracao: Com efeito, desenvolvendo o primeiro membro, temos: a sy (a;4b;) = (a,+b)) ¥ (agtb) + * (a) ° = Caytaytent a.) + (bytbyte tb) = 2 a =f + Ib, ist * ist 2 Teorema 3.2 To a=na . isl Demonstragao: Teorema 3 Demonstragao: Consoante os dois teoremas anteriores, temos: 50 0 n n Zo (a;+a)= Ea, + 2 as f a, tna isl * isl * Get ef n 2 Teorena 3.4 E ka, =k E a, isl * i=l * Demonstragao: Com efeito, desenvolvendo o primeiro membro, temos: a a Eo kay = kay + kayte..t ka = k(aytayte eta) =k Ea, isl isl 20 Exémplo 3.3 Calcular = (Si + 2) isi Consoante os teoremas anteriores temos, sucessivamente: 20 20 20 20 & (Sit2) = £ Si+ £ 255 F it 20.26 isl isl ist isl = S5(L+ 2+... + 20) +40 = da + 20)20 + 40 = 5,210 + 40 = 1090 3.3 SOMATORIOS DUPLOS a Teorema 3.5 Eo a.. = i, jal 31 Demonstraga: Com efeito, desenvolvendo o primeiro membro, temos: n Boigu Ait et ag tag tag tee tag t i,jel teeta tay tata ou seja: a a 2 a aon Eoa,= 2 a.,+ Da.+..+ fam E La, atygen jer pay 25 ga gen ger 25 mon a 2 Teorema 3.6 = Lab, = fa. E b, ca gelisr*> J js. t ga J Demons tragao: Com efeito, pelo teorema 3.4, temos: moa nfo 2 EOE ab, 5 E abs |= © = jetist * 40 ja | ier 44 jel a nm =ity . 5% i-l jar 3 . Be Bribe Exemplo 3.4 Caleuiar £ £ 2734 jel isl Consoante o teorema 3.6, temos: ma oe BoE ais = Eats ado (2 + 2742703 + 3%) © jel ieL isl jet = 14.12 = 168 3.4 PRODUTGRIOS Sejam os n>1 inteiros 811855 »4,+ Para indicar, de mo do abreviado, o produto a,a,...a, desses n inteiros usa~ se a notagao: a T a, ie. + que se 1é: "produtorio de a; dela a’. Em particular, para n= 2,3,..., temos: 2 3 T a. =aa,, T 4; =e,a,a,, ..: ie, rt ia. i res A letra i chama-se o indice do produtério e pode ser subs tituida por qualquer outra diferente de aeden- éum indice mido. E os inteiros 1 e n que figuram abaixo e aci ma da letra grega maiiiscula I (pi) chamam-se respectiva- nente limite inferior e limite superior do indice i. 0 nimero de fatores de um produtério é sempre igual a di- ferenga entre os limites superior e inferior do seu indi~ ce mais uma unidade. 53 Se men sao dois inteiros, com mén, entao, por defi gio: Exemplo 3.5 Temos: 6 JW 3i = (3 -1) (3.2) (3.3) (3.4) (3.5) (3.6) = Aol 3,6.9.12.15.18 = 524880 4 ; TM (5j - 3) = (5.1 - 3)(5.2 = 3)(5.3 = 3)(5.4 - 3) = jel = 2.7.12.17 = 2856 Exemplo 3.6 Temos: 6 3,.9.27.81.243.729 = 1 3 isl 1.2.3...(n-1)n = al 3.5 PROPRIEDADES DOS PRODUTGRIOS. Teorema 3.7 54 Demonstragao: Com efeito, desenvolvendo o primeiro membro, temos: n ie ash. = (ayby) Canby)... (a,b) - (a,25-.+a,) (b, = Pores n oreby) “ z a,. n Teorema 3.8 T asa Demonstraga Seja a; =a para i= 1,2,...,n. Entao, temos: a a Hoae M apm ajay ..ap-aa sas at isl isl a Teorema 3.9 fl ka, = 35 5 15 35: 1 6 a1 1 ? 59 (ii) cada linha tem um elemento a mais que a linha an- terior; (iii) todas as linhas comegam e terminam por 1, visto que a. a, @=Qet (iv) os elementos da linha de ordem n+l sao precisa~ mente os coeficie ntes do desenvolvimento de n (a+b). Assim, p.ex., os elementos da setima linha sao os coefi- cientes do desenvolvimento de (a+b)®, 3.8 PROPRIEDADES DO TRIANGULO DE PASCAL 0 triangulo de PASCAL goza de indmeras propriedades, den- tre as quais destacam-se as seguintes: (1) Dois elementos de uma mesma linha, equidistantes dos extremos, sao iguais. E consequéncia da férmula (1) = ( dois nimeros n mk? binomiais complementares sao iguais. (11) A soma dos elementos da linha de ordem n+l @ igual a a2 £ consequencia da formula: + G+ G+ + GMO + GD) wt 60 (IIT) (Iv) Cada elemento de uma linha, a partir do segundo,’ é @ soma do elemento que lhe fica acima com o que es ta @ esquerda deste dltimo. E consequéncia da relagdo de STIFEL: n ny n+l. D+ @- ath Esta propriedade permite a construcao rapida e f4- cil do tritngulo de Pascan. A soma de um nilmero quaiquer de elementos consecu- tivos de uma coluna,a partir do primeiro, @ igual ao elemento que se encontra na intersegao da linha e da coluna imediatamente posteriores aquelas 2 que Pertence o ultimo dos elementos considerados. Esta propriedade resulta da formula: @ = dap + pts gt +e Assim, p.ex., a soma dos n primeiros elementos da segunda cola: 1. 2 k- n=1 2. Qs Ds Os ne CD, ® 3 igual a ("$4), isto &: L+2434 0.4 (ol) +n = OD 61 (V) A soma de um niimero qualquer de elementos consecuti vos de uma mesma diagonal, a partir de um elemento da primeira coluna, & igual ao elemento que se en- contra na mesma coluna do iltimo elemento considera do e imediatamente abaixo deste. Esta propriedade decorre da formula: n n-L. n-2 Ge) * Got) * ore te + OH Assim, p.ex., sao elementos consecutivos de uma mes ma diagonal: 7 Ae Sete (p> Ge GIs ween ¢ e, portanto: n+l. D+ D+ Qtr y= ab 3.9. NOMEROS TRIANGULARES Os inteiros que formam a tercetra coluna do triangulo de PASCAL: L=1, 35142, 6214+2+3 lO=1+2+4+3+4+4, 15 L+24+34+4645, 0... cada um dos quais @ a soma de inteiros consecutivos come gando por 1, chamam-se nimmeros triangulares, porque cada 62 um deles da o numero de pontos que podem ser ‘dispostos em forma de triGngulo equildtero, conforme mostram as figu- ras abaixo: abd & Denotando o n-Gsino niinero triangular por t,, as proprie~ dades do triangulo de PASCAL permitem afirmar que t_ tem para expressao* n+1 n(ntl) ‘ist @ que a soma S, dos n primeiros nimeros triangulares é da da pela férmila: ct2y . n(n#l) (n42) $256 Teorema 3.10 A soma de dois niimeros triangulares consecu tivos @ um quadrado perfeito. Demonstra¢ai Com efeito: 63 NOTA. Os pitagoricos relacionaram os inteiros com a Geome tria, introduzindo os nimeros poligonats : nimeros trian- gulares, nimeros quadriticos, niimeros pentagonats, etc. A razao desta nomenclatura geométrica resulta da represen tagao dos niimeros poligonais mediante Pontos dispostos em forma de triangulos, quadrados, pentagonos, etc., como mostram as figuras que se seguem: a BE aE re Denotando o n-ésimo nimero quadratico por q,> @ imediato n?, @ temos (Teorema 3.10): que a. 64 isto @, o n-ésimo niimero quadratico a soma do n-ésimo nimero triangular e do seu antecessor. Fste resultado os pitagéricos conheciam, e o demonstraram Separando os pontos e contando-os, no modo indicado nas figuras. A soma Sq dos n primeiros nimeros quadraticos dado pela formula: vi = n(n + 1)(2n + 1)/6 conforme Exercicio la. do Capitulo 2, 65 EXERCICIOS Calculer: Sag di (@y Fa () = 6 isl i=3 4 3 (eo) 2% (+1) +2) @) = Ke jel kel Calcular 3 2 3 (a) TT (3+i*) () Ts isl k=l 6 7 oe fc) TM (j-1) G41) (a) oT n/2 jn2 n=3 Usando o simbolo de somatérto, escrever as seguintes expressoes: (a) apby + aby * ab, + aydy (b) 3.6.5 + 4.5.6 + 5.6.7 +... K(k) (+2) Ge) apby3 + Agghyq * Apbg3 + 24043 + Azsbs3 66 Dizer se & verdadetro (V) ou falso (F): n n+3 n n (a) E (443) = fog (b) £ (244) =2+ Ea i=l is4 i=l i=l a» n 5 5 (ec) Mat = (May) (a) Day= Ea, f=1 isl i=0 isl Demonstrar: m n n m I May 1 tay, d=1 j=1 430 21 ae1 15 Demonstrar as seguintes propriedades dos niime vos triangulares: (a) Um niimero & triangular se e somente se da forma n(n + 1)/2, onde n é um inteiro positivo. (b) 0 inteiro n & um niimero triangular se e somente se 8n+l @ um quadrado perfeito (c) Se n & um niimero triangular, entdo 9nt1, 25n+3 e 49n+6 também sao niimeros trtangu- laves. Na sequéncia dos nimeros triangulares, achar (a) dois nimeros triangulares cuja soma e di- ferenga também sejam nimeros triangulares; (b) trés nimeros triangulares consecutivos cujo produto seja um quadrado perfeito; 10. 67 (c) trés nimeros triangulares consecutivos cuja soma seja um cuadrado perfeito. Demonstrar: (a) ) = tee LBS 0, nol 0 zr 2 n ) GY #2) 43th = th, not fe) Gy + 2) + 2B ee. e2) By a3”, nol 2+ GQ) + tet = My, nee Calcular o termo independente de x no desen- ‘ volvimento de Sx + 29 No desenvolvimento de (a +b)" os coeficien- tes do 3? e do 8¢ termos sao tguats. Calcular n. CAPITULO 4 DIVISIBILIDADE 4.1 RELAGAO DE DIVISIBILIDADE EM Z Definigao 4.1 sejam ae b dois inteiros, coma ¥# 0. Diz~ Se que a divide b se e somente se existe um inteiro q tal que b = aq. sque & @ um miltipio de a, que 2 um fator de & ou que bé divieivel por a. Se a divide b também se diz que a um divisor de b, Com a notagao "a|b" indica-se que a #0 divide b e, por- tanto, a notagao b. b" significa que a ¢ 0 n@o divide A relagdo "a divide b (a]b)" denomina-se visibilidade om 2. relagdo de di~ Se a @ um divisor de b, entdo va também Zum dtvtsor de 5, porque a igualdade b = aq implica b = (wa)(-q), da modo que os dévisores de um inteiro qualquer sao dois a dois iguais em valor absoluto ¢ de sinais opostos (olmd- 69 tricos), Assim, p.ex.: 2|6, porque 6 = 2.3 -5|30, porque 30 = (-5)(-6) 7|-21, porque -21 = 7(-3) afro, porque nao existe q€Z tal que 10 = 32 Teorema 4.1 Quaisquer que sejam os inteiros a, b e c,tem se: (1) al, lla e ala (2) Se all, entdo a= 41 (3) Se alb ese cla , entao ac|ba (4) Se alb ese b lc, entao ale (5) Se alb ese bla, entao a +b (6) Se alb, comb #0, entao [a/<|b| (7) Se alb ese ale, entao al(dxtcy), Vx,yez Demonstragao: (1) Com efeito: O#a.0, asl.a, asad (2) Com efeito, se ali, entao 1 = aq, com qé Z,o que 70 implica a=leq*=loua--leq=-l, asttly (3) Com efeito: alb ===> b= aq, com q€Z cld = a= cqy, com qe Z Portanto: Bd = (ac) (qq) => ac|bd (4) Com efeito: alb —>s- aq, com q€Z ble =e = ba, com aye Portanto: c= a(qa,—==> ale (5S) Com efeito: alb——>b = aq, com qéZ pla a + bay, com 4,€ 2 Portanto: a = a(qq,) aq, = 1S gy Tait 4 eae eo (6) Com efeito: isto é: 71 alb,b #0——>b=aq, g¥0 = [df = fal] al Como q #0, segue-se que {q/ 21 e, portanto: |b] > Jal (7) Com efeito: alb => b= aq, com q€Z ale c= aq, com ez Portanto, quaisquer que sejam os inteiros x e y: bxtcy = agxtaqyy = a(qxtqyy) —> al (bxtcy) Esta propriedade (7) admite uma Gbvia generalizacao; isto 6, se entAo, quaisquer que sejam os inteiros KpoXpreees al (byx, + box, + we + bax) Consoante as propriedades (1) e (4), a relagdo de divisibilidade Z @ reflemtva e transttiva, mas nao é stmétrica, porque, p.ex., 3/6 e 64. 4.2 CONJUNTO DOS DIVISORES DE UM INTEIRO 9 conjunto de todos os dévisores de um intefro qualquer a indica~se por D(a), isto é: 72 D(a) = {x€2* | xa } onde 2Z* denota o conjunto dos inteiros ndo nulos (# 0). Assim, p.ex.t D(O) ={x€ 2*| xla}= ze D(L) ={x€2Z*| x{1}= {-1,1} D(2) = {xe Ze] x[2}= (41, +2} D8) =(x€ ZA) x|-8}={H1, + E imediato que, para todo inteiro a, se tem D(a) = D(-a), € como a= a.l = (-a).(-1) Segue-se que 1, -1, 2 e -a so divisores dea, denomi- nados divisores triviats de a, Em particular, o inteiro 1 (ou -1) 86 admite divisores triviais. Qualquer que seja o intéiro a #0, se xfa, entao: -agxga——> D(a) C [Fa,a] e isto significa que qualquer inteiro a #0 tem um niime— to finito de divisores. 4.3 DIVISORES COMUNS DE DOIS INTEIROS Definigdéo 4.2 Chama-se divtsor comm de dois inteiros a eb todo inteiro d #0 tal que dja e dlb. 73 Em outros termos, divisor comin de dois inteiros ae b ¢ todo inteiro @ #0 que pertence simultaneamente aos con juntos D(a) e D(b). 9 conjunto de todos os divisores comms a dois inteiros a eb indica-se por D(a,b). Portanto, simbolicamente: D(a,b) ={x€z* | xja e x/b} ou seja: D(a,b) ={x€zZ*| xeD(a) e x€D(b) } e, portanto: D(a,b) = D(a)MD(b) A intersegao (1) @ uma operagdo comitativa, de modo’ que D(a,b) nao depende da ordem dos inteiros dados aeb, is- to &: D(a,b) = D(b,a). Como -1 e 1 sao divisores comms de dois inteiros quais-— quer a e b, segue-se que o conjunto D(a,b) dos divisores comuns de a e b munca & vazto: D(a,b) # >. Em particular, se a= b= 0, entao todo inteiro nao nulo éum divisor comun de a eb, isto 8: D(a,b) = 2%. Exemplo 4.1 Sejam os inteiros a= 12 eb = -15, Temos: D(12) ={+1, +2, +3, +4, 46, +12} D(-15) ={+1, +3, #5, +15} 74 Portanto: D(42,-15) = D(12)N DG15) ={ +1, +3} 4.4 ALGORITMO DA DIVISAO Teorema 4.2 Se ae b sao dois inteiros, com b>0, entao extetem e sao inicos os inteiros q e r que satisfazen as condigoes: a=ba+re O¢r0, temos b>1 e, portanto, para x = - [a|, resulta: a-bx=atblal >a+t jal 20 Assim sendo, pelo "Principio da boa ordenagdo", existe 0° elemento minimo x de $ tal que r20 e r=a-ba ou a=bqtr, com ges Além disso, temos rb, terfamos: 75 Osr-b=a-bqa-b=a-b@4 ler isto @, r nao seria o elemento minimo de S. Para demonstrar a wietdade de q © x, suponhamos que exis tem dois outros inteiros q, e r, tais que asbq +r) € O¢r, 0, nada ha que demonstrar, e se b<0, 76 entao |b] >0, e por conseguinte existem e sao inicos os inteiros q, e x tais que a= [Bla +r e Ogr< || ou seja, por ser a= B(-q)) +r e O¢r< |b] Portanto, extetem e sao imtcos os inteiros q = “4, er tais que a=ba+r oe O¢r<|bl Os inteiros q e r chamam-se respectivamente o quoctiente e o'resto na divisao de a por by Observe-se que b 2 dévdsor de a se e somente se 0 resto r= 0. Neste caso, temos a= bq ¢€ 0 quoctente gna dim visdo exata de a por b indfea-se tanSéa por pou a/b (q = 2 = afb), que se 1@ "a sobre b". Exemplo 4.2 Achar 0 quociente qeorestorna divisde de a= 59 por b = -14 que satisfazem As condigoes da al- gorttmo da divisao. Efetuando a divisao usual dos valores abeolutoe de a o b. obtemos: 59 = 14.443 77 © que implica: 59 = C14)(-4) +3 e 0¢3<]-14] Logo, 0 quociente q=-4 eo resto r = 3, Exemplo 4.3 Achar 0 quoctente qe 0 resto rna divisao de a= -79 por 6 = 11 que satisfazem as condigées do at gorttmo da divisan, Efetuando a divisao usual dos valores absolutos de a e b, obtemos: 79 = 11,7 +2 © que implica: “79 = 11(-7) -2 Como 0 termo r= -2<0 nao satisfaz 4 condigdo O¢rqeoer=1 “2s C7).145 © O<5<|-7]} —> quier-s 78 61 = (-7)(-8) +5 e 0<5<|-7]——>q = Beras5 759 = (-7).9 +4 O Demonstrar que o mdc(n + k,k) = 1 see somente se o mde(n,k) = 1. Demonstrar que, se albc e¢ se o mdc(ayb) =d, entao aled. Demonstrar que, se alc, se bic e se o mdc(a,b) = den tao abled. Demonstrar que, se o mdc(a,b) = 1 e se o mdc(a,c) = d, entao mdc(a,bc) = d. 99 24. 0 inteiro impar a@um divisor de atbe de a-b. De- monstrar que d também é um divisor do mde(a,b). 25. Os inteiros positives a, bec sao tais que o mdc(a,b) = 1, alc e cb. Demonstrar que a= 1. 26. 0 mde(nyn + k) 1 para todo inteiro positive n. De~ monstrar que k lL ow k= -2. 27. Denonstrar que o mde (a,b) = ndc(a + kb,d) para todo in teiro k. 28. 0 mdc(a,4) = 2 = mde(b,4). Demonstrar que o mdc(a + b,4) = 4, 29. Os inteiros positivos men sao tais que o mdc(m,n) = = 4. Mostrar que o mdc(2™~- 1,2" - 1) = 24-4, 30. Demonstrar que o mdc(a,b) = mdc(a,b,a +b). 31. Demonstrar que o mdc(a,b) = mdc(a,b,ax + by), quaisque que sejam os inteiros x e y. 32. 0 mde(a,b) = p, sendo p um primo. Achar os possiveis valores do: (a) mde(a?,b): . 3 a 8 (b) mde(a”,b)5 (ce) mde (a”,b~). ao 3 2 33, Sabendo que o mdc(a,p) = p e que o mde(b,p”) = p’,on- de p um primo, calcular 0 mde(ab,p") @ 0 mde(a + bsp’). 100 34, Demonstrar que, se o mdc(a,b) = d, entao o ndc(a”,b?) = @2 35. Sejam ae k inteiros nao conjuntamente nulos. trar que o mdc(a, atk)|k. Demons- 36. tencoa rar que ica, b) = mdc(ayc) implica mde (a”, yb = ndc(a? sc Py 37. Demonstrar que mde(a,b) = mdc(a,c) implica mde(a,b) = mdc(a,b,c). 38. Demonstrar que mdc(a,b,c) = mde(mde(a,b) ,mde(a,c)). 39, Sejam ae b inteiros positivos tais que ab & um qua- drado perfeico eo mdc(a,b) = 1. Denonstrar que a eb s@o quadrados perfeitos. 40. Demonstrar que o mde (a+b, a~b) ymdc(a,b). 41. Mostrar que o mde(Sn+6, 5n#8) = 1, onde a @ um intei- ro tnpan. 42. Sejam a, b, c, d(b # d) inteiros tais que o mdc(a,b)= = mde(c,d) = 1. Mostrar que a soma a/b +c/d nio & um inteiro. 43. Determinar os inteiros posisivos a eb, sabendo: Bat? 6 Tuk oo mdede,b} = 22, 44. Dividindo-se dois’ inteiros positivos pelo seu mde, soma dos quocientes obtidos € 8. Determinar os dois in teiros, sabendo que a sua soma @ 384. caPiTuLo 6 ALGOR{TMO DE EUCLIDES MINIMO MULTIPLO COMUM | Lema 6.1 Se a=bq+r, entdoo mdc(a,b) = mdc(byr). Demonstragao: Se o mdc(a,b) =d, entdo dja e d|b, o que implica d|(a - bq) ou dlr, isto é, d @ um divisor comm deb e r (a[b e dr). Por outro lado, se ¢ & um divisor comm qualquer de be r (cb e clr), entdo c|(bq +r) ouc{a, isto é,c & um divisor comm de ae b, 0 que implica cb, tais que b nao divide a, isto @: a>b>0 e bY. Nestas condigoes, a aplicagao repetida do algoritmo da divisdo di-nos as igualdades: arba try, 0Ty > T4 > Ty Ty? aoe e existem apenas b-1 inteiros positivos menores que byne cessariamente se chega a uma divisdo cujo resto ri, = 0, “1 isto €, finalmente, teremos: Taz "oan tn) Oy Sy x ri, =0 m-1" "ntnel * Fned ? "nel 103 0 Gitimo resto r, #0 que aparece nesta sequéncia de divi s0e8 @ 0 maximo divisor come procurado de ae b, isto o mic (a,b) = r,, visto que, pelo Lema anterior, temos: mdc(a,b) = mdc(b,r)) = mdc(r vt) = mde(r syst) = nde(r,_) >t) Te Este processo pratico para o calculo do maximo divisor co- mm de dois inteiros positives ae b & denominado algonit mo de EUCLIDES ou processo das divisdes sucesstvas. E usual o seguinte dispositivo de cdlculo no emprego do al goritmo de EUCLIDES: 4 | a [ a det al[/> [x | Fit) F2 | 73 | | que se traduz na seguinte REGRA! Para se "achar" o mdc de dois inteiros positivos, divide-se o maior pelo menor, es~ te pelo primeiro resto obtido, © segundo resto pelo primei ro, e assim sucessivamente até se encontrar um resto mulo. 0 Gltimo resto nao nulo & 0 maximo divisor coma procuvada 0 algorttmo de EUCLIDES também ser usado para achar a ax- pressac do mdc(a,b) = r, como combtnagao ttnear de aeb. Para 0 que basta eliminar sucescivamente os restos ty, 104 Fa-gett+oT3eFyeTy entre as n primeiras igualdadés” anterio- res. Exemplo 6.1 Achar o méc(963,657) pelo algoritmo de BUCLI DES e a sua expresso como combinagdo linear de 963 © 657. Temos, sucessivamente: 963 = 657.1 + 306 657 = 306.2 + 45 i 306 = 45.6 + 36 963 657 | 306 45 36 9 45 = 36.149 306 || 45] 36 | 9 | o 36 = 9.440 | Portanto, o mdc(963,657) = 9 e a sua expressao como combi~ nagao Linear de 963 e 657 sé obtém eliminando os restos 36, 45 e 306 entre as quatro primeiras igualdades anteriores do seguinte modo: 9 = 45 - 36 = 45 - (306 - 45.6) = =-306 + 7.45 = -306 + 7(657 - 306.2) = = 7.657 - 15.306 = 7.657 - 15(963 - 657) = = 963(-15) + 657.2 isto é: 9 = mdc(963,657) = 963x + 657y onde x =-15 e y = 22. 105 Esta representagao do inteiro 9 = mdc(963,657) como combi- nagio linear de 963 e 657 nao é imica. Assim, p.ex.,so- mando e subtraindo o produto 963.657 ao segundo membro da igualdade: 9 = 963(-15) + 657,22 obtemos: 9 = 963(-15 + 657) + 657(22 - 963) = = 963.642 + 657(-941) que @ uma outra representagdo do inteiro 9 = mdc (963,657) como combinagdo Linear de 963 e 657. Exemplo 6.2 Achar o mdc(252,-180) pelo algordimo de EUCLI DES e a sua expressao como combinagao linear de 252 e -180. Temos, sucessivamente: 252 = 180.1 + 72 180 = 72.2 + 36 72 = 36.2 Portanto, o mde(252,-180) = mde(252,180) = 35, e como 36 = 180 - 72.2 = 180 - (252 - 130)2 = = 252(-2) + (-180) (-3) temos: 106 36 = mde(252,-180) = 252x + (~180)y onde x= -2 ¢ y= -3, que a expressio do mde (252, 180) como combinagdo Linear de 252 e -180. Outra representagdo do inteiro 36 = mdcl252,-180) como combinagdo linear de 252 ¢ -180 éa seguinte: 36 = 252(-2 + 180) + (-180)(-3 + 252) = = 252.178 + (-180)249 Exemplo 6.3 0 mde de dois inteiros positivos aebé 7% ena sua determinagio pelo algoritmo de EUCLIDES os quoci- entes obtides foram 1, 2, 2, 5, 1 e 3. Calculer a e be 74 Temos, sucessivamente: a=btr, be2ren, re2r +r) ms Sry #3, T= rz + 74, F3 = 74.3 = 222 Portanto: = 222 + 74 = 296, Ty = 56296 + 222 = 1702 r = 2.1702 + 296 = 3700, b = 2.3700 + 1702 = 9169 a = 9102 + 3700 = 12802 cceq « 107 Teorema 6.1 Se k>0, entdo o mdc(ka, kb) = k.mde (a,b). Demonstragao: Multiplicando ambos os menbros de cada uma das n+l igual dades. que dao o mdc(a,b) = r, pelo algoritmo de EUCLIDES por k, obtemos: ak = (bk)a) + rk, O0, nada ha que demonstrar, e se k<0, entao + rk = |k|>0 e, pelo teorema anterior, temos: mdc(ak, bk) = mdc(~ak, -bk) = = mde(a.[k], b.[k/) = |k]. mde(a,b) 6.2 MOLTIPLOS COMUNS DE DOIS INTEIROS 0 conjunto de todos 6s miltiplos de um inteiro qualquer a #0 indica-se por M(a), isto é: M(a) ={x€Z |alx}={aq | qez} Assim, p.ex.: M(-1) = M(1) =Z M(5) = {5q ]g€Z}={0, 45, +10, 415, +20, ...} E imediato que, para todo inteiro a #0, se tem M(a) = = M(-a). Definigao 6,1 Sejam 4 eb dois inteiros diferentes de ze- ro (a #0 eb #0). Chama-se miltiplo comm de aeb todo inteiro x tal que alx e© bly. Em outros termes, miZ¢iplo comum de ae b @ todo intetro que pertence simultaneamente aos conjuntos M(a) © M(b). 109 0 conjunto de todos os miltiplos comme de a ede b indica Se por M(a,b). Portanto, simbolicamente: M(a,b) ={x€z x e blx} ou seja: M(a,b) ={x€Z | xeM(a) e xEM(b) } e, portanto: M(a,b) = M(a)N M(b) A interesegao (N) @ uma operagao comutativa, de modo que M(a,b) nao depende da ordem dos inteiros dados a e b, isto M(a,b) = M(b,a). Obviamente, 0 € um miléiplo come de ae b: OEM(a,b). E 98 Produtos abe -(ab) também sao miltiplos comms de a eb. Exemplo 6.4 Sejam os inteiros a = 12 e b = -18, Temos: M(12) ={12q | qéz}=10, +12, +24, +36, 448,460,472, . M(-18)={-18q| qéz }= (0,418,436, +54,4+72,490,... t Portanto: M(12,-18) = M(12)N M(-18) = (0,136, +72, 110 6.3 MINIMO MOLTIPLO COMUM DE DOIS INTEIROS Definigao 6.2 Sejam a e 6 dois inteiros diferentes de ze- ro (a #0 e b #0). Chama-se minimo miltiplo comum de a e bo inteiro positivo m (m>0) que satisfaz as condigées: G@) alm e blm (2) se ale ese bic, com c>0, entao mec. Observe-se que, pela condicdo (1), m @ um miltiplo comum de ae b, © pela condigao (2), m € 0 menor dentre todos os miltiplos comuns positives de ae b. 0 minimo miltiplo comm de ae b indica-se pela notagao mnc(a,b). Pelo "Principio da boa brdenagdo", o conjunto dos miltiplos comuns positivos de a e b possui o elemento mint mo e, portanto, o mmc(a,b) existe sempre e @ imico. Além disso, por ser o produto ab um milttplo comm de aeb, se gue-se que o mme(a,b) <|ab!. fm particular, se ajb, entao o mmc(a,b) = |bj. Exemplo 6.5 Sejam os inteiros a=-12 eb = 30. Oo mill- tiplos comuns postttvos de -12 ¢ 30 sao 60, 190, 190, como o menor deles @ 60, segue-se que o mmc(-12,30)-60. 111 6.4 RELAGKO ENTRE 0 MDC E 0 MMC Teorema 6.2 Para todo par de inteiros positives a e b sub siste a relagao: mdc(a,b).mme(a,b) = ab Demonstra¢! Seja mdc(a,b) =d e mmc(a,b) =m. Como ala(b/d) e blb(a/d), segue~se que ab/d & um miltiplo comum de aeb Portanto, existe um inteiro positivo k tal que ab/d = mk, kEN o que’ implica: a/d = (m/b)k e b/d = (m/a)k isto @, k @ um divisor como dos inteiros a/d e b/d. Mas, a/d e b/d sao primos entre si (Corolario 5.1), de modo que k = 1. Assim sendo, temos: ab/d =m ou ab = dm isto é: ab = mdc(a,b).mmc(a,b) Esta importante relagao permite determinar o mme de: dois inteiros quando se conhece o seu mc, ¢ vico-versa- 112 Exemplo 6.6 Determinar o mme(963, 657). Pelo algoritmo de EUCLIDES, temos mdc(963, 657) = 9. Por- tanto: anc (963, 657) = 263-657 « 7299 Corolario 6. Para todo par de inteiros positivos a e b,o mme(a,b) = ab se e somente se o mde(a,b) = 1. Demonstragao: (===>) Seo mdc(a,b) = 1, entao: ab = l.mmc(a,b) = mme(a,b) (==>) Reciprocamente, se o mmc(a,b) = ab, entao: ndc(a,b).ab = ab ==omde(a,b) = 1 6.5 MMC DE VARIOS INTEIROS conceito de minimo multiple comum, definido para dois in teiros ae b, estende-se de maneira natural a mais de dois inteiros. No caso de trés inteiros a, b, e c, diferentes de zero, o mc(a,b,c) @ o inteiro posttivo m (m>0) que sa tisfaz as condigoes: (1) alm, blme cjm (2) se ale, se ble e se cle, com e70,enrao moo. Assim, p.ex.: mmc(39, 102, 75) = 33150 1. 4 113 EXERCICIOS Usando 0 algoritmo de EUCLIDES, determinar: (a) mdc(306, 657) (b) mde (272, 1479) (ce) mée(884,1292) (d) mde(-816, 7209) (e) mdc(7469, 2387) (£) mde(-5376, -3402) Usando 0 algorttmo de EUCLIDES, determinar: (a) mdc(624, 504, 90) (b) mdc(285, 675, 405) (c) mde(209, 299, 102) (d) mde(69, 598, 253) Usando 0 algoritmo de EUCLIDES, achar inteiros x e y que verifiquem cada uma dae coguintes igualdades: (a) méc(56, 72) = 56x + 72y (b) mde (24, 138) = 24x + 138y (c) mde(119, 272) = 119% + 272y (d) mde (1769, 2378) = 1769x + 2378y Achar inteiros x e y que verifiquem cada uma das se— guintes igualdades: (a) 78x + 32y = 2 (b) 104x + Oly = 13 (c) 31x + l9y = 7 (d) 42x + 26y = 16 (e) 288x + Sly = 3 (£) 52x + l3y = 1 (g) 145x + S8y = 87 (h) 17x + Sy = -2 114 Achar inteiros x, y e 2 que verifiquem cada umi das se guintes igualdades: (a) llx + 19y + 32 =1 (b) 56x + 6y+ 322 = 2 (c) 6x + 3y + 152 = 9 (d) 14x + 7y + 2iz = 4 Achar inteiros x, y e z que verifiquem a igualdade: 198x + 238y + 5122 = mdc(198, 288, 512) Calceular: (a) mme(45, 21) {b) mme(83, 68) (ce) mme (120, 110) (4) mme(86, 71) (e) mmo (224, 192) (£) mme(1287, 507) (g) mme(143, 227) (h) mme(306, 657) 0 mdc de dois inteiros positives ae b é 8 e na sua de terminagéo pelo algoritno de EUCLIDES os quocientes su cessivamente obtidos foram 3, 1, 1 e 4. Calculer ach Determinar os inteiros positives ae 0 sabendo: (a) ab = 4032 eo mme{a,b) = 336 (b) mic(a,b) = 8 e@0 wme(a,b) = 560 (c) a+b = 589 e meee) = 84. 115 10. Demonstrar que, se ab sao inteiros positivos tais Que o mdc(a,b) = mme(a,b), ento a= b, 11. Sabendo que o mdc(a,b) = 1, demonstrar: (a) mde(2a+ b, a+ 2b) = 1 ow 3 (>) mdc(a +b, a? +2) =1 on 2 (c) mde(a + by a? - ab +b) = 1 on 3 12. Sendo a @ b inteiros positives, denonstrar que o mdc(a,b) sempre divide o mmce(a,b). caPiTULO 7 NUMEROS PRIMOS 7.1 NOMEROS PRIMOS E COMPOSTOS Definigao 7.1 Diz-se que um inteiro positive p>1 @ um mimevo primo ou apenas um primo se e somente se le p sao os seus inicos divisores positivos. Um inteiro positivo maior que 1 e que nao & primo diz-se composto. Assim, p.ex., os inteiros positives 2, 3, 57 sao todos primos e os inteiros positives 4,6,8 e 10 sdo todos com postos. 0 inteiro positive 1 nao @ nem primo nem composto, e por conseguinte se a @ um inteiro positivo qualquer, entdo a é primo, ou a composto ou a = 1. Observe-se que 2 @ o tinico inteiro positivo par que é pri mo. Teorema 7.1 Se um primo p nao divide um inteiro a, entao ae p sao primos entre st. 117 Demonstragao: Seja do mic de ae p. Entdo dlae dp. Da relacgao d|p,re Sulta que d= 1 oud =p, porque p @ primo,e como a segun da igualdade @ impossivel, porque p nao divide a, segue-se qe d= 1, isto & o mlc(a,p) = 1. Logo, a e p sao primes entre st. Corolario 7.1 Se PB @ um primo tal que plab, entao pla ou pib. Demonstragao: Se pla, nada ha que demonstrar, e se, ao invés, p nao divi de 4, entao, pelo teorema anterior, 0 mdc(p,a) = 1. Logo, pelo teorema 5.4 (de EUCLIDES), pid. Gorolario 7.2 Se p @ um primo tal que pla. 132" existe um Indice k, com 1¢k¢n, tal’ que pla,. » entao in Demonstracao: Usando o "Teorema da indugao matematica", a Proposigao verdadeira p/ n=l (imediato) e para n= 2 (pelo Corold- tio 7.1). Suponhamos, pois, n>2 que, se p divide um Produto com menos de n fatores, entio p divide pelo monog um dos fatores (htpdtese de tndugdo). * Pelo corolario 7.1, se Playayevea,, entao pla, ou playay. 118 Se pla,s a proposicao esta demonstrada, e se, ao invés, plajay.-a, 17 entao a hipdtece de indugdo assegura que pla, com 11, porque Bé primo. Logo, p = = a: Teorema 7.2 Todo inteiro composto possui um divisor pri mo. Demonstracao: Seja,a um inteiro composto. Consideremos o conjunto A de Fodos 08 divisores positives de a, exceto os divisores tri viais le a, isto é: Astxla | léxea) Pelo "Prineipto da boa ordenagao" existe o elemento mintmo P de A, que vamos mostrar ser primo. Com efeito, se B fos~ tag se composto admitiria pelo menos um divisor d tal que l1 @ igual a um produto de fato res primos. : Demonstraga Com efeito, se n @ primo, nada ha que demonstrar, e se n é compasto, entao, pelo teorema 7.2, possui um divisor primo Py» @ temos: n= pn, 1 ny ny © como s6 existe um nimero finito de inteiros positivos me mores que ne maiores que 1, existe necessariamente um a que @ um primo p, (n, = p,,), @ por conseguinte teremos: RL" PyPoPa ++" PE igualdade que representa o inteiro positivo n?>1 como produto de fatores primes. Gorolario 7.4 A decompootggo de um inteire positivo nl como produto de fatores primos % iimtaz, a menos da oyvdom dos fatores. Demonstragao: Suponhamos que n admite duzs decomposigdes como produto de fatores primos, isto 2 = P/PoPy ; rés = 4) 4245-- onde os p; e os 45 sao todos inteiros primos e tais que 121 Como. p)la,q94, --- a,» existe um indice k, com 14,- Ana logamente, 4) = p,, com 1éhp,. Por tanto, temos py, = q), o que implica: Pghg. swag Aydgree Ve Com o mesmo raciocinio conclui-se que Py = 4g» 0 que im- plica: PsPgreePE = 43%, ++ e assim por diante. Assim sendo, se subsiste a desigualdade r1. Logo, r = s e temos: Py = ys Py = Agrees PL = 4, isto @, as duas decomposigces do inteiro positivo n>1 cu mo produto de fatores primos sao idanticas, ou seja, n ad- mite uma unica decompostgdo como produto de fatores pri~ mos. 122 Exemplo 7.1 A decomposicao do inteiro positivo n= 360 num produto de fatores primos @ dada pela igualdade: 360 = 2.2.2.3.3.5 Observe-se que os fatores primos 2 e 3 aparecem repetidos, © primeiro tras vezes e 0 segundo duas vezes. Corolario 7.5 Todo inteiro positive m>1 admite uma tind ca decomposicao da forma: ky k, n= Py Py onde, para i= 1,2,...,r, cada k, @ um inteiro positive e cada Py @ um primo,com PS Py <--- l1l. Exemplo 7.2 A decomposi¢ao candnica do inteiro positivo n= 17460 @ dada pela igualdade: Psty 17460 = 23,3 NOTA. Conhecidas as decomposigdes candnicas de dois in- teiros positives a>1 e b>1, 0 méc(a,b) @ 0 produto dos fatores primos comme is duas decomposteces candni- cas tomados cada um com o menor expoente, eo mme(a,b) & © produto dos fatores primos comms e nao comme ae dune decompostgoes candnicas tomados cada um com 0 maior ex- Ppoente. 123 Assim, p.ex., as decomposigies candnicas dos inteiros posi tivos 588 e 936 sao: 588 = 27.3.77, 936 = 27.37.13 e, portanto: mde(588, 936) = 27.3 = 12 me (588, 936) = 29.37.77.13 = 45864 Teorema 7.3 (de EUCLIDES) Ha um niimero infinito de primos. Demonstragao: Suponhamos que existe um primo Py mator que todos os de- mais "primes p, = 2, p) = 3, p, = 5, Py © 7,+++) @ conside remos o inteiro positivo: P= pyPpyP3++-P, + 1 Como P>1, o "Teorema fundamental da Aritmética" permite afirmar que P tem pelo menos um divisor primo p. Mas,Py, PooPgrre-sPy sd0 os iinicos primos, de modo que p deve, né- cessariamente, ser igual a um desses n primos. Assim sen- do: PIP epi pypyP3---P_ o que implica: pIP ~ pyp2p 124 © que @ absurdo, porque p>1 eo Unico divisor positivo de'l @ © proprio 1. Logo, qualquer que seja o primo Py existe um primo maior que P,» isto €, 0 conjunto { 2,3,5;7511,13, ws} dos primos @ infinito. Teorema 7.4 Se um inteiro positive a>1 % composto, en- tao a possui um divisor primo p< Jar Demonstragao: Com efeito, se o inteiro positive a>1 @ composto,entao: a=be, com l1, 0 "Teorema fundamental da Aritmética" asse~ gura que b tem pelo menos um divisor primo P, de modo que PSbS Ya. Ecomo plb e bla, segue-se que pla, isto 8, 0 inteiro primo p< /a~ @ um divisor de a. NOTA. 0 teorema anterior fornece um processo que permite on reconhecer se um dado inteiro a>1 @ primo eom— posto, pata o que basta dividir a sucessivamente pelos pr mos que nao excedem a Jat 125 Assim,. p.ex., no caso do inteiro a = 509, temos: 22 < 7509 < 23 de modo que os primos que nao excedem a /S0T sao Zed s5y7's 11,13,17 e 19, e como 509 nao @ divisivel por nenhum deles, segue-se que 509 @ primo. Este processo, como logo se vé, @ muito trabalhoso e, por tanto, pouco pratico, sendo até de aplicagao impossivel pa ra inteiros muito grandes. 7.3 FORMULAS QUE Dao PRIMOS Os primos aparecem com muita irregularidade na cequancta dos inteiros positivos, e por isso muitas formulas que dao primos foram construidas. Assim, p.ex., a formula de EU= LER: went a1 fornece primos para n = 0,1,2,-..,39. Entretantoy para n= 40 en = 41 os inteiros que se obtém sao compootao, pois, temos: £40 = 407 + 40 + 41 = 40(40 + 1) + dt = 40.42 + 42 = = 41(40 +1) = 41.41 = 412 126 fy, 7 41? + 41 + 41 = 41041 #1) + 41 = 41082 + 41 = = 41(42 + 1) = 41.43 Outras formulas que dao primos sao: £,=2n" +29 para 0¢n<28 f=n74+n+17 para 0<¢nelé £0 an + 30+ 23 pera Dene2l 7.4 CRIVO DE ERATOSTENES A construgao de uma tabela de Primos que néo excedem um dado inteiro n faz-se usando o processo conhecido pelo no~ me de ertvo de ERATOSTEWES, e que consiste no seguinteies crevem-se na ordem natural todos os inteivos desde 2 ate a , em seguida, eZiminan-se todos os inteiros aomoatoo que sdo miltiplos dos primos p tais que p< Jay isto ep, 3p, 4p, -.. Exemplo 7.3 Construir a tabela de todos 0S primos menores que 100. Os primos p tais que p< Y100 = 10 sao 2,3,5 07. Loge. cumpre escrever na ordem natural todos og inteiros desde 2 até 100 e, em seguida, eZiminar todos os inteiros compos tos que sao miltiplos de 2,3,5 e 7. 127 2-3 KS SF Soe pee now 13 Os we re oer po 23 hw 2 29 2 a ww On Ow 1 Be 43S ge 7 ge os ESS OS BO ge 395 61 SY Be ES oe 6) 58 oS 1 1 WO ye we 19 AL S83 BK Bk Be Br sr 99 9 SE 9 9S gk 9S 96 97 96 oF yal Os inteiros positivos nao eliminados: 2,3,5,7,11,13,17,19,23, 29,31,37,41,43,47,53, 59,61,67,71,73,79,83,89,97 sao todos os primos menores que 100. 7.5 PRIMOS GEMEOS Definigao 7.2 Chamam-se primos gémeos dois inteiros posi~ tivos impares e consecutivos que sao ambos primoo. Assim, p.ex., sdo pares de primos gémeos: 3e5, Se7, llel3, 17e€19, 29 e 31 Nao se sabe até hoje se ha um numero infinito de pares de 128 primos gémeos, mas si conhecidos primos gémeos muito gran des, tais como: 140.737.488,353.507 e 140.737.488.353.509 140.737.488.353.699 e 140.737.488.353.701 Um fato interessante @ a existéncia de apenas um terno de inteiros positivos impares e consecutivos que sao todos primos: 3, Se 7. 7.6 SEQUENCIAS DE INTEIROS CONSECUTIVOS COMPOSTOS Teorema 7.5 Existem sequéncias de n inteiros positivos consecutivos © compostos, qualquer que seja o inteiro po= sitivo n. Demonstrag. Com, efeito, @ Sbvio que aa sequénéia: (atl)! # 2, (n+l)! + 3, (nel)! + 4yeeey (ntl)! + (n+1) os seus n termos sao inteiros positivos consecutivos, e ca da um deles & compoeto, porque (ntl)! + j @ divicivel por j se 21? Ha também muitas proposicdes sobre os pYimoe cuja demons- tragdo exige recursos de indole muito elevada, isto %, pa- ra as quais nao existe uma demonstragao elementar, tals co- mo, P.eX., as duas seguintes: (i) Em toda progressao aritmatie: a, atr, at2r, at3r, .., onde a er (razio) so inteiros positivos primoo entre si, hd um niimero infinito de primoc (DIRICHLET) . (ii) Para todo inteiro n>3, entre ne 2(n-1) existe, pelo menos, um primo (TSCHEBISCHEFT) . 131 7.8 METODO DE FATORAGAO DE FERMAT Dado um inteiro positivo impar n, a decomposigao de n num produto de dots fatores distintos se pode obter pelo se- guinte método devido a FERMAT: Constroi-se uma tabela com (n-1)/2 linhas, obtidas pela adigao sucessiva de inteiros impares consecutivos a Bn. Se na r-ésima linha aparece o quadrado perfetto t*, entZo ne=(t+r)(t-r). Assim, p.ex., com n = 21, temos a seguinte tabela de (21 = 1p = 10 Linhas: gu ™a+l= % 1 22+3=25=57 25 +5 = 30 30+7=37 37 +9 = 46 46 + 11 = 57 57 +13 = 70 70 + 15 = 85 85 + 17 = 102 10 102 + 19 = 121 = 11 wan nur ene 2 Na segunda linha figura S* ¢ na déctna linha figura 14"\8e modo que temos: 132 2L = (54+ 2)(5-2) = 7x3 21 = (11+ 10)C11 - 10) = 21 x1 Sao estas as duas iimicas maneiras de decompor o inteiro po sitivo Impar 21 num produto de dois fatores distintos,pois, todas as outras sao variagdes triviais destas, tais 215 3x7 = (-7)(-3). como EXERCICIOS 1. Achar os cinco menores primos da forma n° - 2, 2, Achar trés primos impares cuja soma seja: (a) 81; (b) 125 3. Achar todos os pares de primos pe q tais que P-a=3. 4. Achar todos os prémos que séio iguais a um quadrado per feito menos 1, Achar todos os primos que $46 iguais a um cubo perfed to menos 1, 10. ll. 12. 13. 14. 15. 133, Determinar todos os inteiros positivos n tais que n, nH2 e nt so todos primes. Determinar todos os primos ptais que 3p+1 é um quadra do perfeito. Determinar se sao primos os seguintes inteiros: (a) 169 (c) 239 (b) 197 (d) 473 Achar a decomposigao candnica do inteiro 5040. Achar 0 méc(a,b) e 0 mc(a,b) sabendo: a= 239,521 19.233 2b = 29,3,74,112, 195.237 Mostrar que sdo primos gémeos: F (a) 1949 e 1951 (b) 1997 e 1999 Achar todos os pares de primos gémeos entre 400 e 500. Achar uma sequéncia de quatro inteiros positivos conse cutivos e compostos. Achar uma sequéncia de 100 inteiros positivos consecu- tives e compostos. Verificar a conjectura de GOLDBACH para os seguintes inteiros pares: (a) 32; (b) 100; (c) 456; (a) 1024. 134 16. 18. 20. 21. 22. 23. 24. 25. 26. 2. Verificar que todo inteiro par entre 4 e 100 é-a soma de dois primos. Achar 0 menor inteiro positive n tal que 2n2 +29 @ um inteiro composto. Mostrar que a soma de inteiros positivos impares e con secutivos @ sempre um inteiro composto. Usando a decomposigao canédnica dos inteiros 507 e 1287, achar o mdc(507, 1287) e o mme(507, 1287). Demonstrar que todo primo, exceto 2 e 3, @ da forma 6k-1 ow 6k+1, onde k € um inteiro positivo. Achar 0 menor inteiro positivo pelo qual se deve divi dir 3720 para se obter um quadrado perfetto. Achar todos os primos que sao divisores de 50! Mostrar que o iinico primo da forma n> - 1 67. Mostrar que todo inteiro da forma né + 4, com n>1 @ composto. Mostrar que, se n>4, @ composto, entao n divide (n= 5t Mostrar que todo inteiro da forma 841, com nl, = composto. Mostrar que, se 0°42 4 primo, entao 3(n. 28. 29. 30. 31. 135 Mostrar, mediante um exemplo, que a seguinte conjec- tura é falsa: Todo inteiro positivo pode escrever-se sob a forma 2 se a a +p, onde o inteiro a20 e p @ um primo ou l. Demonstrar as seguintes proposicoes: (a) Todo primo da forma 3n+l também @ da forma 6m+l. (b) Todo inteiro da forma 3n+2 tem um fator primo des~ ta forma. (c) Se p=5S & um primo, entdo p'+2 & composto. (d) Se p @ um primo e se pla, entdo pla. (e) Todo inteiro n>1l pode ser expresso como a soma de dois inteiros compostos. (£) Se p>q>5 ese p eq sao ambos primos, entao 2 2 ae 24] (p" - 4°). - . +, ~ 2 - (9) 8¢ p45 % um pein impor, entao p’-1 ov pst é divisivel por 10. Verificar que todo inteiro pode escrever-se sob a for- ma 2“n, onde o inteiro k20 em é um inteiro impar. Demonstrar que um inteiro positivo a>1 @ um quadra— do perfeito se e somente se todos os expoentes dos fa~ tores primos da sua decompostgao canéntca sao intei— ros pares. 136 32. Demonstrar que, se o inteiro n & composto, entao 2%! também @ composto. 33, Demonstrar que, se o inteiro n>2, entao existe um primo p tal que n Yo tais que ax, + by, = ec diz se uma solugao inteira ou apenas uma solugdo da equacao ax + by = c. Consideremos, p.ex., a equacdo diofantina linear com duas incognitas: 3x + 6y = 13 pane 3.446.118 3(-6) + 6.6 = 18 3.10 + 6(-2) = 138 Logo, os pares de inteiros: 4el, 626, 10-2 sa0 solugdes da equagao 3x + by = 1g, Existem equagoes diofantinas lineares com duas incdgnitas que nao tém solucao. linear: Assim, p.ex., a equacao diofantina 2x + dy = 7 nao tem solupdo, porque 2x + 4y @ um inteiro Par quais~ Quer que sejam os valores inteiros de x e y, enquanto que 7 um inteiro impar (observe-se que 2 = nde(2,4) nao di- vide 7), Pe modo geral, a equagdo diofantina linear ax + by =c nao tem solugdo todas as vezes que d= mdc(a,b) nao di- vide c, como @ dbvio. ° 8.2 CONDIGAO DE EXISTENCIA DE SOLUCKO Teorema 8.1 A equacao didfantina linear ax + by =e qegreme 8.1 solugao se e somente se a divide c, sendo 4 = mdc(a.p). tem Demonstragio: ( ==> ) Suponhamos que a equacdéo - ax + by =¢ Cem uma so luedo, isto &, que existe um par de inteiros Xy+¥q tais 139 que ax, + by, = c. ° Por ser o mdc(a,b) = d, existem inteiros res tais que asdr e b=ds, e temos: cm ax, + by, = drx, + dsy, = d(rx, + sy.) E como rx, + sy, @ um inteiro, segue-se que d divide c (je). (<—— ) Reciprocamente, suponhamos que d divide ¢ (dlc), isto @, que c = dt, onde t @ um inteiro. Por ser o mdc(a,b) = d, existem inteitos x, e y, tais que d= ax, + by, o que implica: c = dt = (ax, + by,) t = a(tx,) + b(ty,) isto 6, o par de inteiros: x = tx) = (c/d)x,, y = ty, = (c/d)dy, @ uma solugdo da equagao ax + by =c. 8.3 SOLUCOES DA EQUACAO ax + by = c. Teorema 3.2 Sed divide c|(d|c), sendo d=mdc(a,b), e se o par de inteiros x,y, @ uma solugdo particular da 140 equacao diofantina linear ax + by = ¢, entdo todas as ou~ tras solugées desta equacdo sao dadas pelas formulas: x= x, + (b/d)t, ys wa (a/d)e onde t & um inteiro arbitrario. Demonstraga Supohhamos que o par de inteiros x of @ uma soluedo par- tieular da equagdo ax + by = Cc, e seja Xy» Y, uma outra soluedo qualquer desta equagao. Entao, temos: ax, + sce + x, t by =e ax, + by, €, portanto: ate, - xy) = boyy ~ yy) Por ser o mdc(a,b) = d, existem inteiros Ee s tais que a=dr e¢ b=ds, com Les primos entre st. Substituin- do estes valores de a e b na igualdade anterior ¢ cance lan do o fator com 4a, obtemos: FQx, 45) = 80, - yy) Assim sendo, r[s(y, - ¥y), € comoo mde(r,s) = 1, segue— se que rly, ~ 1), isto é: Yo7¥, srt « onde t @ um inteiro. Portanto, temos as formulas: 141 =x, tst =x + (b/d)t Yp 7 ¥y 7 Key, - (a/ade Estes valores de x, e 1 ax + by = c, qualquer que seja o inteiro t, pois, temos: a[a, + @/ae]|+o[y, - taraye | = = (ax, + by,) + (ab/d - ab/a)e = y, Satisfazem realmente a equagao ax, + by) =e+Otec Como se vé, se d = mdc(a,b) divide c (d|c), entdo a equa- gao diofantina linear ax + by =c admite um namero infi- nito de solugoes, uma para cada valor do inteiro arbitra- rio Corolario 8.1 Se o mde(a,b) = Le se x,,y, é uma solugao particular da equacao diofantina linear ax + by = c, en- t@o todas as outras solugdes desta equagao sao dadas pelas formulas: x=x,+bt, y ow) onde t @ um inteiro arbitrario. NOTA. Uma solugdo particular da equdgdo diofantina lincar se obtém por tentativas ou pelo algoritmo de EUCLIDES. § em ambos os casos a solugdo geral se pode obter usando 0 teorema 8.2, conforme se vai ver nos exemplos a seguir. 142 Exemplo 8.1 Determinar todas as solugdes da equagio dio- fantina linear , 172x + 20y = 1000 Determinemos, inicialmente, o mdc(172,20) pelo algortimo de EUCLIDES: 172 = 20.8 + 12 20 = 12.1+8 12 = 8.1+4 8 = 4.2 Portanto, o mdc(172,20) = equacao dada tem solucao. e como 4{1000, segue-se que a Posto isto, cumpre obter a expressao do inteiro 4 como com binagdo linear de 172 e 20, para que o que basta eliminar sucessivamente os restos 8 e 12 entre as trés primeiras igualdades anteriores do seguinte modo: 4 = 12-8 = 12 - (20 - 12) = 2.12 - 20 = = 2(172 = 20.8) - 20 = 172.2 + 20(-17) isto é: 4 = 172.2 + 20(-17) Multiplicando ambos os membros desta igualdade por 1000/4 = 250, obtemos: 1000 = 172.500 + 20(-4250) Portanto, o par de inteiros x, = 500, y= -4250 solugao particular da equagao Proposta, e todas as solugdes sao dadas pelas formulas: x = 500 + (20/4)t = 500 + St y = ~4250 ~ (172/4)t = -4250 - 43e onde t € um inteiro arbitrario. 143 @ uma outras Exemplo 8.2 Determinar todas as solucoes inteiras e posi- tivas da equagao diofantina linear 18x + Sy = 48 Determinemos o mdc(18,5) pelo algoritmo de FUCLIDES: 18 = 5.343 stitial eee is ffs [32 fa er 3 if2 11 | 0 2 = 1.2 Portanto, o mic(18,5) = 1 e a equagao dada tem solugdo. E para exprimir 1 como combinagdo linear de 18 e 5 bas ta eli minar os restos 2 e 3 entre as trés primeiras igualdades anteriores do seguinte modo: Ll=3-223-(5-3) 227.3452 2(18 - 5.3) - 5 = 18.2 + 5(-7) isto é: 144 1 = 18.2 + 5(-7) 48 = 18.96 + 5(-336) Logo, o par de inteiros %, = 96, y¥, = ~336 @ uma solugao particular da equacao proposta, e todas as demais solugdes sao dadas pelas formulas: x= 96+ 5t, y = -336 - 18t onde t @ um inteiro arbitrario. As solugoes inteiras e posttivas se acham escolhendo t de modo que sejam satisfeitas as desigualdades: 96 + 5t>0, -336 - 182 >0 isto @: e<-ig9 i e t<-1g 2 5 3 o que implica t= -19 e, portanto: x = 96 + 5(-19) = 1, y = -336 - 18(-19) =6 Assim, o par de inteiros x= 1, y= 6a imica solucao intetra e positiva da equacéo 18x + Sy = 48. Exemplo 8.3 Resolver a equagao diofantina linear 39x + 26y = 105, 145 0 mdc(39, 26) = 13 e como 13 nao divide 105, Segue-se que a equagao dada nao tem solugdo. Exemplo 8.4 Resolver a equagao diofantina linear 14x + 22y = 50. O mdc(i4, 22) = 2 e como 2[50, a equacdo dada tem solu- gao, e@ por simples inspecdo logo se ve que 14.2 + 22.1 = 50, de modo que o par de inteiros X72, y= @ uma solugdo particular, e por conseguinte todas as demais solugdes sao dadas pelas férmulas: x=24+11t, yel-7e onde t @ um inteiro arbitrario. 146 1. EXERCICIOS Determinar todas as solugdes inteiras das seguintes equacdes diofantinas lineares: (a) 56x + 72y = 40 (b) 24x (c) 221x + ly = 117 (d) 84x (e) 48x + Jy = 5 (£) 57x (g) Lx + 30y = 31 (h) 27x (4) 13% - 7y = 21 (j) 44x (k) 21x - 12y = 72 (1) 17x + 138y = 18 ~ 438y = 156 - 99y = 77 - 18y = 54 + 66y = 11 + Say = 8 Determinar todas as solugdes inteiras e posttivas das seguintes equacdes diofantinas lineares: (a) 5x - Ly = 29 (b) 32" + S5y = 771 (c) 58x - B7y = 290 (4) 62x + Lly = 788 (e) 30x + 17y = 300 (£) Séx + 2ly = 906 (g) 123K + 360y = 99 (h) 158x - 57y = 7 Determinar 0 menor inteiro positivo que dividido por 6 @ por 15 deixa os restos 6 e 13, respectivamente. 147 Exprimir 100 como soma de dois inteiros positivos de modo que o primeiro seja divisivel por 7 e 0 segundo seja divisivel por ll. Determinar as duas menores fragoes positivas que te- sham 13 e 17 para denominadores e cuja soma seja igual a 305/221. . Demonstrar que, se a e b sao inteiros positivos primos entre si, entao a equacao diofantina ax - by = c tem um nimero infinito de solugdes’ inteiras e positivas.. cAPITULO 9 CONGRUENCIAS 9.1 INTEIROS CONGRUENTES Definigdo 9.1 Sejam a e b dois inteiros quaisquer e seja mum inteiro posttivo fimo. Diz-se que a é congruente a b méduio m se e somente se m divide a diferenca a-b. Em outros termos, a € congruente a b modulo m se e somen- te se existe um inteiro Kk tal que a- b= kn Com a notagao a =b (mod.m) indica=se que a & congruente a b médulo m. Portanto, sim bolicamente: a= b (mod.m) <==) m|(a = b) ou seja: a => (mod.m) <== > Jkézia-ps= km 149 Assim, p.ex.: 3.524 (mod.7), porque 7|(3 - 24) -31 = 11 (mod.6), porque 6] (-31 - 11) -15 = -63 (mod.8) porque 8|(-15 -(-63)) Se m ndo divide a diferenga a-b, entdo diz-se que a in- congruente ab modulo m, o que se indica pela notagao: a fd (mod. 2) Assim, p.ex.: 25 712 (wod.7), porque (25 - 12) -21 #10 (mod.5), porque 5-21 - 10) 16 jo (mod. 4), porque 4(16 - 9) Note-se que dois inteiros quaisquer sao congruentes modulo 1, enquanto que dois inteiros sao congruentes modulo 2 se ambos sao pares ou se ambos sao impares. Em particular, a = 0 (mod. m) se e somente se o modulo m divide a (mia). Exemplo 9.1 Mostrar: n= 7 (mod.12) — > n = 3 (mod.4), Vn€Z Com efeito: 150 n= 7 (mod.12) —> n= 7 = 12k ==> n- 3 = 4C3ke1) 4| (a - 3) n = 3 (mod. 4) Exemplo 9.2 Mostrar: q n” = 0 (mod.4) ou n° = 1 (mod.4), Ynez Com efeitos Z G) om pars nm eam nt - ant ao Gln? — 2? = 0 tod. 4) Gi) on impar: n= 2+ => m2 2 44K) 41 — (nt) — n= 1 God. 4) 9,2 CARACTERIZAGAO DE INTEIROS CONCRUENTES Teorema 9.1 Dois inteiros a e b sao congruentes modulo m se e somente se a e b deixam o mesmo resto quando dividi- dos por m- Demonstragao: (=>) Suponhamos que a = b (mod.m). Entao, por defini gio: a-b= km, com k€Z 151 Seja ro resto da divisao de b por m; entao, pelo algorit= mo da divisdo: b=mq+r, onde O a = b (mod. m) Exemplo 9.3 Sejam os inteiros -56 e -1l. Pelo algoritmo da divisao: “56 = O(-7) #7 “elk = 9E2) #7 isto @, -56 e -11 divididos por 9 deixam o mesmo resto 7. Logo, pelo teorema anterior: -56 = -11 (mod. 9). Sejam, agora, os inteiros -31 e ll. Temos a congruencia: -31 = 11 (mod. 7) 152 de modo que, pelo teorema anterior, -31 e 11 divididos por 7 deixam o mesmo resto. Realmente, @ o que mostram as igual dades: “31 = 7(-5) +4 e 1L=7.1+4 9.3 PROPRIEDADES DAS CONGRUENCIAS Teorema 9.2 Seja mum inteiro posttivo fizo (m>0) e se- jam a, b e cinteiros quaisquer. Subsistem as seguintes propriedades: (1) a =a (mod. m) (2) Se ab (mod..m), entao b =a (mod. m) (3) Se ab (mod.m) e se bc (mod. m), ent@o a =c (mod. m). Demonstragao: (1) Com efeito: m0 ou mi(a- a) —= a Za (mod. m) (2) Com efeito, se a= b (mod. m), entao a- b = km, com k€2, Portanto: b =a = -(km) = (-k)m => b = a (mod. m) 153 (3) Com efeito, se a =b (mod. m) ese bic (mod. m),en tao existem inteiros hek tais qe a-b=hm e b-c=km Portanto: a-ec= (a-b) + (b-c) = hm + m= (h+ k)m e@ isto significa que a = c (mod. m). NOTA. Consoante este teorema, a relacgao R no conjunto Zdos inteiros definida por aRb <=> a = b (mod. m) & veflexiva, simétrica e transitiva, ou seja, R @ uma relaeao de equivaléncia em 2. Esta relagao de equivaléncia R em Z @ denominada "congruen eta modulo m". Teorema 9.3 Seja mum inteiro positivo fimo (m>0) e se- jam a eb dois inteiros quaisquer. Subsistem as seguintes proptiedades: (1) Se a=» (mod. m) ese nim, com n>.0, entao a = b (mod. n) (2) Se a 2b (mod. m) e se c>0, entao ac F be (mod. me). 154 (3) Se a = b (mod. m) e se a, b, m 8a0 todos divisivets pelo inteiro d>0, entao a/d = b/d (mod. m/d). Demonstracao: (1) Com efeito: ab (mod. m) => a-b-=km e n[lm— > m= nq onde ke q>0 sao inteiros. Portanto: a-b = (kq)n ==> a = b (mod. n) (2) Com efeito: a=» (mod.m) => a - b = km==> ac - be = k(me) ==> ac = be (mod. me) (3) Com efeito: B® " = b (mod.m) ==> a-b km => a/d - b/d = k(m/d) —— a/d = b/d (mod.m/d) Assim, p.ex.: a -15 = 9 (mod.8) ==> -15 = 9 (mod. 4) 7 -8 (mod. 3) > 35 = -40 (mod. 15) 36 = -24 (mod. 12) => 9 = -6 (mod. 3) 155 Teorema 9.4 Seja mum inteiro posttivo fro (m>0) e se- jam a, b, ¢, d inteiros quaisquer. Subsistem as seguintes propriedades: (1) Se a =b (mod.m) e se c =d (mod. m), entao at+c=b+d (mod. m) e ac = bd (mod. m) (2) Se a= b (mod. m), entdo atc b+c (mod. a) e ac = be (mod. m) (3) Se a = b (mod. m), entao b™ (mod. m) para to- P do inteiro positivo n. Demonstrag (1) Com efeito, se a= b (mod. m) ese ¢ d (mod. m), entdo existem inteiros he k tais que a - b = hm e ¢- d= km. Portanto: (ate) - (b+d) = (a-b) + (c-d) = bm + km = (h + k)m ac-bd = (b + hm)(d + km) - bd = (bk + dh + hkm)m o que implica: atec=b+d (mod.m) e ac = bd (mod.m) (2) Com efeito, se a =b (mod.m), como cic (mod.m), temos, pela propriedade anterior: at+c=bt+c (mod.m) e ac = be (mod.m) 156 (3) Usando o "Teorema da indugao matematica”,;. a-proposi~ cao @ verdadeira para n= 1, e suposta verdadeira para um inteiro positivo k, temos: a* = B® (aod.m) e a = b (nod.m) Portanto, pela propriedade (1): k a aktls vb (mod.m) ou bel nod.) isto @, a proposicao @ verdadeira para o inteiro positivo k+l. Logo, a proposicao @ verdadeira para todo inteiro po. sitivo n. Assim, p.ex.: (4) 12° 22 (mod.5) e 8 £13 (mod.5) implica: 12 + 8 5 22 + 13 (mod.5) ou. 20 = 35 (mod. 5) 12.8 = 22.13 (mod.5) ou 96 = 286 (mod.5) (ii) 12 = 5 (mod.7) implica: 12+ 6=5 +6 (mod.7) ou 18 = 11 (mod.7) 12(-9) = 5(-9) (mod.7) ou -108 = -45 (moa.7) (iii) -5 = 2 (mod.7) implica: (-5)3 (mod.7) ou 125 = 8 (mod.7) 157 Exemplo 9.4 Mostrar que a= b (mod.m) implica -b (mod.m). Com efeito, multiplicando ordenadamente as congruéncias: b (mod. m) e -1 = -1 (mod. m) obtemos: a(-1) = b(-1) (mod.m) ou -a = -b (mod.m) Exemplo 9.5 Mostrar que a+ bc (mod.m) implica ac -b (mod. m). Com efeito, somando ordenadamente as congruéncias: a+ bic (mod.m) e -b = -b (mod.m) obtemos: a+b + (=b) 3c + (-b) (mod.m) ou a = c-b (mod. m) Portanto, numa congruencia pode-se passar um termo de um membro para o outro trocando-lhe o sinal. Teorema 9.5 Se ac = be (mod.m) e se,o mdc(c,m) = d, en- tao a = b (mod.m/d). Demonstrag. Com efeito, se ac = be (mod. m), entao: ac- be = (a-b)c = km, com kEZ 158 E se o mdc(c,m) = d, existem inteiros res tais-que ° ce =dr em=ds, onde r e s sao primos entre st. Portantot (a- b)dr = kds ou (a - b)r =ks © que implica que s|(a - b)r, como mde(r,s) = 1, Logo, pelo teorema 5.4 (DE EUCLIDES): s|(a-b) @ ab (mod.s)ou, por ser s=m/d, a (mod.m/d). Corolaric 9.1 Se ac = be (mod. m) e se o méc(cym) = len > (mod.m). tao a Esta propriedade mostra que @ permitido cancelar fatores de ambos os membros de uma congruéncia que sao primos com o modulo. Corolario 9.2 Se ac = be (mod.p), com Pp primo, ese p nao divide c ok), entao a ='b (mod.p). Demonstragao: Com efeito, as condigoes: p nao divide c (pl<) e p & primo, implicam que o mdc(c,p) = 1. Exemplo Consideremos a congruéneia: 3321S (mod. 9) ou 3,11 5 3.5 (mod. 9) Como o mdc(3,9) = 3, pelo teorema 9.5, temos: 5 (mod, 3). 159 Exemplo 9.7 Consideremos a congruéncia: -35 = 45 (mod.8) ou 5(-7) 5.9 (mod.8) Como o mdc(5,8) = 1, podemos cancelar o fator 5 de ambos os membros da congruencia, o que da a nova congruéncia: -7 = 9 (mod.8). - Na congruéncia 4.11 = 4.15 (mod.8) nao podemos cancelar o fator 4, porque o mdc(4,8) = 4 #1, Realmente, 11 71s (mod. 8). Mas, temos 11 = 15 (mod. 2) 9.4 SISTEMAS COMPLETOS DE RESTOS Definigao 9.2 Chama-se sistema completo de restoe modulo m todo conjunto $ ={rysty,..eor, } de m inteiros tal que um inteiro qualquer a @ congruente modulo m a um intco elemento de S. Assim, p.ex., cada um dos conjuntos: €1,2,3}, {0,1,2} , {-1,0,1} , {1,5,9} 8 um sistema completo de restos médulo 3. Teorema 9.6 O conjunto s ={0,1,2,...,m-I} @ um sistema completo de restos modulo m. 160 monstracaot Seja a um inteiro qualquer e sejam qe t 0 quoctente eo resto na divisao de a pelo inteiro positivo m, isto é: a=mq+r, onde O¢r ) Suponhamos que a congruéncia linear ax = b (mod. m) tem como solugao o inteiro xy isto @, que ax, = b (mod.m). Entao, existe um inteiro Yq tal que ax, - b= my, ou ax, — my, = b ecomo dja e d|m, porque d= mdc(a,m), segue-se que dl (ax, - my,) e, portanto, d|b. 168 ( <==) Reciprocamente, suponhamos que d|b,"isto @% que b= dk, onde k @ um inteiro. Como o mde(a,m) = d, existem inteiros x, e Yq tais que ax, + my, = 4 ou, multiplicando ambos os membros desta igualdade por k: a(kx,) + m(ky,) dk =b ou a(kx,) - b = m(-ky,) © que implica: a(kx,) = b (mod.m) Portanto, o inteiro kx, @ uma solugdo da congtuéncia 1li- near ax = b (mod.m) 10.3 SOLUGOES DA CONGRUENCIA ax = b (mod.m) Teorema 10.2 Se d divide b (d|b), sendo d = mdc(a,m),en tao a congruéncia linear ax = b. (mod.m) tem precisamente d solugdes mutuamente incongruentes mddu lo m. 169 Demonstragao Foi mostrado anteriormente que a congruéncia ax = b (mod. m) @ equivalente 4 equagao diofantina linear ax - my = b, equacao esta que tem solugdo se e somente se d= mde(a,m) divide b (d|b). Além disso, como @ sabidb, se djb e seo par de inteiros x,y, @ uma solugdo par- ticular da equagao ax - my = b, entao todas as outras so. lugdes desta equagao sao dadas pelas formulas: x= x, + (o/d)t, yey, + (a/de onde t @ um inteiro arbitrario. Entre o numero infinito de intéiros dados pela primeira dessas formulas consideremos sémente aqueles que resultam de atribuir a t os valores: 0,1,2,...,d-1, isto @, os d inteiros: Xo» Xt md, x, + 2(m/d), +s Xp + ED (w/a) Posto isto, vamos mostrar que estes d inteiros sao mutua- mente incongruentes modulo me que todos os outros intei- ros dados pela formla x =x, + (m/d)t sao congruentes médulo m a algum desses d inteiros. Com efeito, se fosse x + (/d)t, =x, +(m/d) ty (mod.m) onde O y = 5+ 9t onde t @ um inteiro arbitrario. Substituindo este valor de y na equagao diofantina linear dada, obtemos: 9x + 16(5 + 9t) = 35 => x = -5 - Lot 178 10.5 INVERSO DE UM INTEIRO Definigao 10.2 Seja a um inteiro. Chama-se imverso de @ modulo mum inteiro a* tal que aa* = 1 (mod.m). Nem todo inteiro tem um inverso médulo m. Assim, p.ex.,2 nao tem um tmerso médulo 4, porque a congruencia linear 2x = 1 (mod.4) nao tem solugao. Teorema 10.3 Seo mdc(a,m) = 1, entdo a tem um iinico in verso modulo m. Demonstragao: Com efeito, seo mdc(a,m) = 1, entao a congruéneia 1i- near ax = 1 (mod.m) tem uma tinica solugdo x, (mod.m), isto é: ax, = 1 (mod.m) de modo que o inteiro a tem um iintco imerso médulo m: atex, 0. Exemplo 10.9 Por ser 5.5 25 = 1 (mod.8), segue-se que o inverso de 5 modulo 8 & 0 proprio 5. Note-se que 5(-3) = 1 (mod.8) e 5.13 1 (mod.8) 179 de modo que -3 e 13 também sao inversos de 5 modulo 8, mas isto nao @ inconsistente com a unteidade do inverso de .5 modulo 8, porque “3 = 5213 (mod.8) Exemplo 10.10 Achar: (a) o inverso de 2 médulo 5: (b) 0 inverso de 7 modulo 9: Temos as congruéncias lineares: (c) o inverso de 12 modulo 17. 2a* = 1 (mod.5) => a* = 3 Jax = 1 (mod.9) ==> at = 4 “12a* = 1 {mod.17) —> a* = 10 EXERCICIOS 1, Resolver as seguintes congruéncias lineares: (a) (c) (e) (g) (i) 2x = 1 (mod.17) 3x = 6 (mod.18) Sx 2 2 (mod.26) 36x = 8 (mod.102) 8x = 16. (mod.12) (b) (a) (é) (h) @ 3x = 1 (mod.17) 25x = 15 (mod.29) 6x = 15 (mod.21) 34x = 60 (mod. 98) 14x = 36 (mod.43) 180 2. Resolver por congruéncias as seguintes equagdes- fantinas Lineares: (a) &x + 5ly =9 (6) 12x + 25y = 331 (ce) 5x - 53y = 17 (4) 7x + by = 9 (2) Lix + 27y = 4 (£) 75x - 131y = 6 (g) 39x + 26y = 104 (h) 61x - Ly = 81 (i) 65x + 77y = 200 Gj) 51x + B5y = 1037 Determinar o nimero de solugdes de guintes congruéncias lineares: (a) 3x = 6 (mod.15) (b) 4x {c) Sx = 10 (mod.15) (d) 6x cada uma das = 8 (mod.15) = 11 (mod.15) dio se~ Deterninar o ninero de solugdes que pode ter uma con- gruéncia linear cujo modulo @ 20. Demonstrar que, se d = mdc(a,m) e se d|b, entdo as congruéncias lineares: ax b (mod.m) e (a/d)x = (b/d) (mod.m/d) tém precisamente as mesmas solugdes. capttuco 11 SISTEMAS DE CONGRUENCIAS LINEARES 11.1 GENERALIDADES Um ststema de duas ou mais congruéncias lineares nao tem necessariamente solugdo, mesmo que cada uma das congruén- cias do sistema, isoladamente, tenha solugao. Assim,p.ex., nao existe inteiro algum x que verifique simultaneamente as congruéncias lineares: x = 1 (mod.2) e x =0 (mod.4) embora cada uma delas, isoladamente, tenha solugao. Obviamente, um sistema de congruéncias lineares ndo tem so lugdo se alguma das congruéncias do sistema nao tem solu gio: Exemplo 11.1 Mostrar que todo inteiro x = 52 (mod.105) 8a tisfaz cada uma das trés seguintes congruéncias lineares: 182 1 (mod. 3) x = 2 (mod. 5) x = 3 (mod. 7) A primeira congruéncia da-nos x = 1 + 3a, onde a @ um in- teiro. Substituindo este valor de X Na segunda congruén- cia, obtemos: 1 + 3a = 2 (mod. 5) 3a = 1 (mod. 5) a = 2 (mod. 5) Portanto: a=2+5b e x=1 + 3(2 + 5b) = 7 + 15b onde b @ um inteiro, Substituindo este valor de Xma_ ter- ceira congruéncia, obtemos: : 7 + 15b = 3 (mod.7) : 1Sb = -4 (mod. 7) b = 3 (mod.7) Portanto: - b=3+7e e@ x= 7 + 15(3 + Je) = 52 + 105¢ onde c @ um inteiro, e isto significa que todo inteiro uw x = 52 (mod.105) satisfaz as trés congruéncias lineares dadas. 183 Note-se que os modulos 3, 5 e 7 sao primos entre si dois a dois e que o mme(3,5,7) = 105. 11.2 TEOREMA DO RESTO CHINEZ Sejam My sMyy +++ 5m, inteiros positivos primos entre si dots a dots, isto @, tais que o ndé (m; sm;) =1se i # j. Nestas condigoes, o ststema de congruéncias lineares: x= ay (mod.m,) - x =a) (mod.m,) ” 2 = Fa. (mod.m_) tem uma iorica solugio médulo m = n,m, Demonstrag: Para cada k = 1,2,...,r, seja: Mm mf, = mye My isto é, M, € 0 produto de todos os inteiros m; como fa~ tor m, ométido. Por hipotese, os m; sao primos entre si dots a dois, de modo que o ndc(tt,,m,) = 1 e, portanto, a congruéncia linear: Me tem uma tmica solucgao x 1 (mod.m,) qa) 184 Posto isto, vamos demonstrar que o inteiro: Ks aMx, + ajiyx, +... + a Mx, satisfaz cada uma das congruéncias do sistema considerado, ou seja, que X é uma solugdo deste sistema. Com efeito, se i #k, entao a, |My e M; = 0 (mod. m,),0 que implica: X= ayMx, + aMoxy +..-+ aM x = aM x, (mod.m,) E como x, @ a solugdo da congruéncia (1), temos Mo, = 1 (mod. m,) © que implica: a.1 = a, (mod.m,) e isto prova que X @ uma solugdo do sistema de congruén- cias lineares considerado. Para demonstrar a untctdade desta solugdo, suponhamos que KX) @ uma outra solugao qualquer do sistema de con- gruencias considerado. Entao: Kea 2X (modm), k= 1,2,...,7 de modo que a (& ~ X,) para cada valor de k. E como o mde (m, m;) = 1, segue-se que aym,.-.m,| (X - X), isto &: ml (XK - %) e X =X; (mod.m) 185 com o que termina a demonstracgao do "Teorema do resto chi- nea", Exemplo 11.2 Resolver o sistema de congruéncias lineares: x = 2 (mod.3) x = 3 (mod.5) x = 2 (mod.7) Os médulos 3, 5 e 7 das congruéacias do sistema dado sao primos entre si dois a dots: mde (3,5) = mde(3,7) = mde(5,7) =1 de modo que, pelo teorema anterior, o sistema tem uma iint- ca solugao modulo m= 3.5.7 = 105. Temos aqui: M, = m/3 = 35, M, = m/5 = 21, =m/7 = 15 As congruencias lineares: 35x = 1 (mod.3), 21x = 1 (mod.5), 15x = 1 (mod.7) tém como solugdes respectivas: x 2, x, #1, 1 2 3 Portanto, o inteiro: X = 2.35.2 + 3.21.14 2.15.1 = 233 Como 233 = 23 (mod.105), segue-se que X = 23 (mod.105) @ a intea solugdo do sistema de congruéncias lineares dado. 186 Noté-se que este sistema corresponde ao seguinte problema: achar um inteiro que deixa os restos 2, 3 e 2 quando divi- dido por 3, 5 e 7, respectivamente. Exemplo 11.3 Resolver o sistema de congruéncias lineares: = 8 (mod.5) = 5 (mod.3) 11 (mod.7) = 2 (mod. 4) ae) A Os médulos 5,3,7 e 4 das congruéneias do sistema dado sio primos entre si dois a dois, de modo que o sistema tem uma linica solugdo midulo m= 5.3.7.4 = 420. Temos: Ms m/5 = 84, M) = m/3 = 140, M, = 0/7 = 60 M, = m/4 = 105 As congruéncias lineares: 84x = 1 (mod.5), 140x = 1 (mod.3), 60x =1 (mod.7) 105x = 1 (mod.4) tem como solugdes respectivas: x, = 4, x)= 2, x, = 2, x7 1. Portanto, o inteiro: X = 8.84.4 + 5,140.2 + 11.60.2 + 2.105.1 = 5618 Como 5618 2 158 (mod.420), segue-se que X = 158 (mod.420) @ a tnica solugdo do sistema de congruéncias lineares da- do. Teorema 11.1 Sejam m,,m),...,m_ inteiros positivos pri- mos entre st dots a dois, isto é, o nde(m; ,m;) el se i#j, e sejam a 12895 23, inteiros tais que o mdc(a, jm.) = 1 para k= 1,2,...,r7 Nestas condigoes, o sistema de congruéncias lineares: x = by (nod-m,) ox = by (mod.m,) tem uma tinica solugdo médulo m= mm,.. Demonstragac Como mdc(a,,m,) = 1, a congruéncia Linear a,x = 1 (mod.m,) tem uma unica solugao modulo my: aks, de modo que ayak = 1 (mod. m,) 188 Logo, a congruéncia a,x = b, (mod.m,) @ equivalente a con gruéncia: =x © bat (mod .m,) © por conseguinte o sistema dado é equivalente ao sistema de congruéncias lineares: * = bat (mod..m, ) x = byag (mod.m,) 3 bax (mod om.) x © qual tem, pelo "Teorema do resto chinez" » uma unica solu ¢do médulo m= Mym)++-m. Exemplo 11.4 Resolver o sistema de congruéncias lineares: 2x = 1 (mod.5) 3x = 2 (mod.7) 4x = 3 (mod.11) Os médulos 5, 711 das congruéncias do sistema dado sao primos entre si dots a dois e, além disso: mde(2,1) = mdc(3,2) = mdc(4,3) = 1 de modo que, pelo teorema 11. 1, o sistema tem uma unica sO lugao médulo m= 5.7.11 = 385. 189 As congruéncias lineares: 2x = 1 (mod.5),. 3x = 1 (mod.7), 4x = 1 (mod. 11) = . * * * tem como solugdes respectivas: a) = 3, a, = 5, ayo3,e por conseguinte o sistema dado @ equivalente ao sistema: - 3 (mod.5) x = 10 (mod.7) x = 9 (mod.11) para o qual X = 108 (mod.385) @ a iinica solugdo. Exemplo 11.5 Resolver a congruéncia linear: 13x 217 (mod.42) Por ser 42 = 2.3.7, a congruéncia linear dada @ equtvalen- te ao sistema de congruéncias lineares: 13x = 17 (mod.2) [ x = 1 (wod.2) 13x = 17 (mod.3) ou x = 2 (mod.3) 13x = 17 (mod.7) Le 4 (mod.7) A primeira congruéncia da-nos x = 1 + 2a, onde a @ um in- teiro. Substituindo este valor de x na segunda congruencia obtemos: 1 + 2a = 2 (mod.3) =a = 2 (mod.3) 190 Portanto: a=2+3b e x-=1 + 2(2 + 3b) =5 + 6b onde b @ um inteiro. Substituindo este valor de x na ter~ ceira congruéncia, obtemos: 5 + 6b = 4 (mod.7) ==> b = 1 (mod.7) Portanto: bel+7e e x=5+ 6(1+ 7c) = 11 + 42c onde ¢ @ um inteiro, e isto significa que x = 11 (mod. 42) @ a uiniea solugao da congruéncia linear dada 13x = 17 (mod.42). 191 EXERCICIOS Resolver os seguintes sistemas de congruencias res: linea~ (a) x = 1 (mod.2), x 21 (mod.3) (b) x = 5 (mod.12), x = 7 (mod.19) (c) x = 8 (mod.26), x = 11 (mod.33) Resolver os seguintes sistemas de congruéncias linea~ rest i (a) x =3 (mod.5), x =5 (mod.7), x 27 (mod.11) (b) x = 1 (mod.3), x = 2 (mod.5), x = 3 (mod.7) (c) x = 5 (mod.6), x = 4 (mod.11), x = 3 (mod.17) (d) * n = 5 (mod.11), x = 14 (mod.29), x = 15 (mod.31) (e) x 7 (mod.9), x = 10 (mod.4), x = 1 (mod.7) (£) x = 28(mod.29), x = 30(mod.31), x = 10 (mod.11) (g) x a (mod.3), x = b (mod.5), x Zc (mod.8) Resolver os seguintes sistemas de congruencias linea- rest 192 5x = 11 (mod.17) 3x = 19 (mod.32) 11x = 6 (mod.37) 2x = 1 (mod.5) 3x = 9 (mod.6) 1 (mod.7) 5x = 9 (mod.11) 4, Resolver, os seguintes sistemas de congruencias linea- 1 res: (a) = 8 (mod.9) 2 (mod.3) = 5 (mod.7) um = 4 (mod.6) 13 (mod.15) = 8 (mod.14) = 1 (mod.7) ae veo (c) (mod. 3) = 1 (mod.4) 17x = 9 (mod.23) x O# ui ° caPITULo 12 TEOREMAS DE FERMAT E WILSON 12.1 TEOREMA DE FERMAT Se 24 um primo ¢ se p nio divide o intetro a (va), en- tao: a 2 1 Guod.p} Demonstragao: Consideremos os p-1 primeiros miiltiplos positivos de a, isto é, os inteiros: a, 2a, 3a, ..., (p-l)a Nenhum desses inteiros @ congruente a 0 (mod.p), além dis- so, dois quaisquer deles sao tneongruentes (mod.p),pois, se fosse: ras=sa (mod.p), 1l<¢r1 é primo se e somente se (n - 1)! = +1 (oden). Entretanto, para inteiros grandes este critério é imprati- cavel, e por isso de interesse apenas tedrico. Exemplo 12.6 'Verificar o teorém de WILSON com p=7. Temos: “OQ -bei= 641 = 7206127212 7,103 Portanto: (7 = 1)!+ 150 (mod.7) f (7 = 1)! = -1 (mod.7) Exemplo 12.7 Reconhecer se o inteiro 11 & primo. Temos: (11-1)! +2 = 102 +1 = 1.2.3...10+1 = 3628801 = 11.329891 &, portanto: (11 - 1)! = -1 (mod.11) Logo, 11 @ primo. EXERCICIOS 1. Verificar o teorena de FERMAT com a = 2 p = 13, 2. Verificar o teorema de WILSON com p = 5. 3, Mostrar que 8 @ composto usando o teorema de WILSON. 4. Mostrar que 19 @ primo usando 0 reetproco do teore ma de WILSON. 5. Reconhecer se 17 @ primo usando o teorema de WILSON. 6. Verificar: (a) 18° = 1 (mod.49) —(@)_ 185 = 1 (mod. 343) 204 7. Achar o resto da divisao de 15! por 17. 8. Mostrar que, se o mdc(a,35) = 1, entao a? = 1 (mod.35). 9. Demonstrar que, para todo inteiro a, se tem: (a) al3 = a (mod.7) (6) a3? = a (mod.13) (e) a2} = a (mod.15) (a) a7 = a (mod. 42) 10. Demonstrar que, para todo inteiro positivo n, se tem: (a) 2" = 1 (mod. 3) () 23" = 1 (od.7) 11. Mostrar que 18! + 1 = 0 (mod.437). 12. Mostrar: (a) 561] (258! - 2) () 561] (3°! - 3) 13. Formar com os inteiros 2,3,4,...,21 todos os pares a, btais que ab = 1 (mod.23). caPituLo 13 DIVISORES DE UM INTEIRO 13.1 DIVISORES DE UM INTEIRO F & ia Teorena 13.1 Se n= p, Py «+. p,” & a decomposigao cand nica do inteiro positive’ a>i, entao os divisores positi vos de n sao precisamente os inteiros d da forma: hy h, 1 d= py Py onde 0 l e a) >1 Exprimindo d e dj como produtos de (nado necessariamente distintos) primos: d= IQ +++ Ags a4 = tjt,. Ee obtemos: _ ky ky T= Py Py wee rr ~ TWy%g++-ytjt5-- que sao duas decomposigées do inteiro positivo n num produ to de primos, e como @ imica uma decomposigao de ade tal natureza, entao cada primo q, coincide com um P5» de modo que, substituindo os ecaageae de primos iguais ao potén- cias de expoente inteiro, teremos: hy hy h t ds 1 42---4, = Py Py +++ P, 2 onde @ possivel algum h = 0. L Reciprocamente, todo inteiro by hh h d= py'Py + P, xh 1, entao: dim) = (ky + Lk, + De (k, + 1) 209 Demons trac: Consoante o teorema 13.1, os divisores posttivos de n sao precisamente os inteiros d da forma: h, h ap lt 4 =p, Py onde Ogh, Sky, Oh 1_@ dado pela férmula: a(n) = (ky + Dek, + 1). FD ou sejat r d(n) = TM (k, + 1) E i isl Note-se que k k. k aa) = a(n, )a0o,” -4(p,") 210 Exemplo 13.2 0 nimero de divisores posttivos ~do~intéiro n= 756 = 27.33.7 a 4(756) = (2 + 1)(3 + 1). + 1) = 3.4.2 = 24 Estes 24 divisores positivos de 756 sdo os inteiros d da forma: onde bh, = 0,1,2, hy = 0,1,2,3 © by = 0,1 0 caleulo para.a determinagdo efetiva destes 24 divisores Posttivos de 756 pode dispér-se do seguinte modo: sg 2 4 3}] 3 6 12 of] 9 + 48 36 271 | 27 54 108 77 14 28 21 42 84 63 126 252 139 378 756 Na primeira linha, do lado direito, escrevem-se as potéen- cias do primeiro fator primo 2, isto é: 1,2 e 4. Na segun- da, terceira e quarta linhas, do lado esquerdo, escrevem- 211 se, por baixo uma das outras, as diversas poténcias do se- gundo fator primo 3, a partir da primeira poténcia, no ca- so presente 3, 9 e 27, e 4 direita de cada uma delas es crevem-se os produtos. dessa poténcia por todos os inteiros da primeira linha. A seguir, do lado esquerdo, escrevem-se as poténcias do terceiro fator primo 7, a partir da primei ra (no caso 7), e a direita escrevem-se os produtos dessas poténcias por todos os inteiros que do lado direito figu- ram nas linhas anteriores. Se houvesse mais fatores primos procedia-se com eles como para o caso do terceiro ator primo. Exemplo 13.3 Achar o menor inteiro positive n que tem 10 divisores positivos. Como 10 = 10.1 = 5.2, temos: d(n) (ky + 1) (ky +2) = 10.1 ou d(n) (ky + 1k, + 2) = 5.2 Portanto, os expoentes, k, ek, dos fatores primos de Ip sao 90, ou4e1. como 2%.3 6 menor que 2°, 0 menor se Beene eee inteiro positivo com 10 divisores positives @ 27.3 = 48. Ha ae m Exemplo_L Achar o inteiro positivo da forma 9.10 e que admite 27 divisores positivos. 212 Por. swe, 9510" « 9°.2 5", deveuon cars (2 + 1)(m+ 1)(m@ +1) = 27 ou (+1? =9—>m+1=3 © mH2 Logo, o inteiro positivo procurado é 9.10 = 900. 13.3 SOMA DE DIVISORES Seja n um inteiro positivo. A soma dos divisores posttivos de n (inclusive 1 e n) indica-se por s(n). Assim, p.ex., os divisores positivoe de 12 sio 1,2,3,4,6 e 12, de modo que s(12) =14+#2+3+4+6+4+12 = 28 Se p @ um primo, entao s(p) = 1 + p, porque os inicos di- visores positivos de p séo le p. E como os divisores posi tives de p* sao 1, pe p*, temos: 3 = sp) #14 p+ ph sR at De modo geral, os divisores posttivos de p™ sao 2 n 1, Ps Pos ears e, portanto: 213 sip) = 1+ pt pe + Em particular: atl | s(2") = 2 e s(t) = £ 3m — 1) A tabela abaixo dé a. soma dos divisores positivos dos in- teiros de 1 até 10: afl 2 3 4 5 6 7 8 9 WwW s(n) ff 1 3 4 7 6 12 8 15 13° 1B Teorema 13.3 Se hk mee Py Pg se PL @ a decomposigao canénica do inteiro positivo n>1, entao kal +1 k+l poo-d pees ret s(n) = + pic aE haa Py od Py -t bie L Demonstraga Consideremos o produto: k, k. 1 2 2 C+ py + Pp tet Dy DL + py + ppt eee ty?) vee k t 2 GQ+pitpe tee. +p 214 Pelo teorema 13.1, cada divisor positivo den é um termo do desenvolvimento deste produto e vice-versa, de modo que k k, s(n) = (1 + p. 4+ on, + 61 + oe pee -. ae 1 2” P23 2 2 kK (tpt pete +R). Aplicando a cada paréntese do segundo membro desta igualda de a fOrmula que da a soma dos termos de uma progressao geométrica finita, temos: kel k,+L ne s(a) == 1 Pom 1 ou seja: Note-se que : ky ky kK. s(n) = s(p,") S{p)°) ++. s(p) Exemplo 13.5 A soma dos divisores posttivos do inteiro n= 180 =°27,32.5 Ceckr ¢ 215 3 2 sis) = 2-1 : «Sat = 113.6 = sus Realmente, os divisores positivos de 180 sao: 1,2,3,4,5,6,9,10,12,15,18,20,30,36,45,60,90,180 ea soma destes 18 divisores @ 546, 13.4 NOTAGHO Seja n um inteiro positivo. com 0 simbolo E £(d) dln : indica-se a soma dos valores de £(d) quando d @ sucessiva- mente igual a cada um dos divigories posttivos de n. Assim, peex.: E £(d) = £(1) + £(2) + £(4) + £(5) + £(10) + £(20) 420 Eat 12 4 22 4 52 4 107 1 6 4 4 25 + 100 = 130 aj10 Com esta notacao, temos: d()= £1 e s(n) = Ed dln din. 216 Assim, p.ex.: d(l0) = E lei+iti+iss a] 10 s(10) = Dd=1+2+5+10=18 4)10 Analogamente, o produto dos valores de f(d) quando a é su- cessivamente igual a cada um dos divisores posttivos de a indica-se pelo simbolo: T £(4) dln Assim, p.ex.: TM £(d) = £(1)£(2)£(3)£(4)£(6)£(12) ayji2 Td = 1,2.4.5.10.20 = 8000 a]20 13.5 PRODUTO DE DIVISORES Teorema 13.4 0 produto dos divisores positivos de um in- . ae 2¢ d 2 teiro positive n>1 @ igual a n@(™)/?, 217 Demonstrag: Sejam Ay ody sere ydy todos os divisores positivos de n, de modo que existem os inteiros q,,q,,--.,q, tais que edy eee q Bedi, a= dq, 0.0 da Multiplicando ordenadamente todas essas k igualdades, obte mos: = (aja, aes 4) (4449 sr Como d)d)...d, = aya) +++ 4, porque cada um dos inteiros Qyo4y> +++) Gy também 3 um divisor de n, temos: a(n). 2 n (dyd5.--d,) donde d(n)/2 did, . a ou seja: na 2 nf@/2 dln Exemplo 13.6 0 produto dos divisores postttvos do inteiro n= 16 @: Ta = 168 (16)/2 © 65/2 2 (42)5/2-. 45. 1028 aj16 Realmente, os divisores posttivos de 16 sao 1,2,4,8,16 eo Produto destes 5 divisores @ igual a 1024. 218 EXERCICIOS 1, Sendo p e q primos, caleular: d(pq), d(p2q) e d(p?a)- 2. Caleular: (42), d(240), d(420) e a(10115). 3. Achar o niimero de divisores positivos de cada um dos seguintes inteiros: (a) 1445 (b) 360; (ec) 1009; (d) 6534, 4. Achar o niimero de divisores positivos do produto 9.26.42. 5. Achar o menor inteiro positivo com sets divisores posi tivos. 6. Achar o menor inteiro positivo natal que d(n) = 8. 7, Achar o inteiro positivo da forma 28.15" e que admite 54 divisores positivos. 8. Achar o inteiro positivo da forma 2.15".7" e que admi- te 36 divisores positivos. 9. Sendo p e q primos, calcvlar: s(pq), s(p?) e s(p%q). 10. Calculat: s(240), s(420), s(1008) e s(10115). € CCC ce ¢ ¢ 17. 18. + Verificar que s(n) = 2n para 219 + Achar a soma dos divisores positivos de cada um dos se guintes inteiros: (a) 144; (b) 360; (c) 1009; (d) 6534, + Calcular: d(s(360)) e s(d(180)). = 6, 28, 496, 8128, 4. Verificar que s(n) = 3n para n= 120 en = 672, Yerificar que s(n) = s(ntl) para n= 957 en =14206. + Verificar as seguintes relagoes: (a) 5(8)s(3) = s(24) (6) s(8)s(9) = s(72) (c) s(4)s(27) = s(108) Verificar que d(m) = d(ntl) = d(nt2) = d(n+3) para n= 3655 e n = 4503. : Achar os inteiros positivos menores que 10000 e com 60 divisores positives. + Achar todos os primos com 90 que dividem o produto 90.1274. 220 20. Achar todos os inteiros positivos tais que-na~ divisao de 167 por cada um deles se obtenha o mesmo resto 17. 21. Determinar o inteiro n = 2*.3%, sabendo que n/6 e n/9 tém, respectivamente, 8 divisores positivos e 10 divi- sores positivos a menos que n. 22. Determinar o inteiro n = 2".5¥.77, sabendo que os pro- dutos Sn, 7n e 8n tém, respectivamente, 8,12,e 18 divi sores positivos a mais que n. 23. Resolver as equagées: (a) s(x) = 36; (b) s(x) = 60. 24, Mostrar que nao existe inteiro positivo n tal que s(n) = 10. 25. Demonstrar: (a) d(m) ¢2¥n 5 () ngs(a) 1, como os fatores ice so primos entre ei dots a dots, tenos: Ae ‘. £(n) = £(P) E09"). 00,7) . Esta igualdade mostra que uma funcdo aritnética mltiplica tiva fica completamente determinada quando os seus valores para poténcias de primos. sao conhecidos. Importa ainda notar que, para toda fungdo aritmética multi plicativa nado identicamente nula se tem £(1) = 1, pois existe necessariamente um inteiro positivo r tal que f(r) #0 ecomoo mde(r,1) = 1, temos: f(r) = £(r.1) = £(r)£(1) —> £(1) = 1 Teorema 14.1 As fungdes d(n) e s(n) sao ambas fungoes aritméticas mltiplicativas. Demonstragao: Se ue v dois inteiros positivos tais que o mdc(u,v) = 1. Se u = 1 ou v = 1, entao, obviamente: 224 d(uv) = d(u)d(v) e s(uv) = s(u)s(v) Suponhamos, pois, u>1 e v>1, e sejam k, k, t by hy hy BO PL Py vee Pee VEG ND eee as decomposigées candnicas de u e v. e y= 1,2,...,5, visto Como Py # dy para x = 1,2,.00y4 que o mdc(u,v) = 1, segue-se que a decomposigao candnica do produto uv @ dada pela igualdade: _ ae Kk, hy hy hy wre Py Py tee PE By Wy os e, portanto: atu) [oe + DeeGg + u [eas + Deeeth; + v] Z = d(u)d(v) = s(u)s(v) de modo que d(n) e s(n) sao fungdes arttméticas miltipli- cativas. 225 Exemplo 14.1 Verificar que a fungao d(n) @ uma fungao aritmérica mltiplicativa para n= 144, Temos 144 = 24,32 e o mdc(16,9) = 1. Portanto: d(144) = (4 + 1)(2 +1) = 5.3 = 15 416) = 44125 d(9) =2+1= isto é: x d(144) = d(16)d(9) Exemplo 14.2 Verificar que a funcao s(n) @ uma fungao arit mética miltiplicativa para n = 63. Tenos 63 = 37.7 e 0 mdc(9,7) = 1. Portanto: 13.8 = 104 isto é: 8(63) = 5(9)s(7) 226 Teorema 14.2 Se £ @ uma funpao aritmética maltiplicativa © se g & a fungao ardimética assim definida: a(n) = 5 £(a) dln entao g também @ uma fungao arttmética multiplicative. Demonstragao: Sejam re s dois inteiros positivos tais que o nde(r,s)=1. Como © conjunto dos divisores positivos de rs consiste de : ze todos os produtos xy, onde xlr, yls eo mde(x,y) = 1, temos: girs) = Ef) = £ £(xy) da[rs xlr,y]s Pela definicgao de fungdo aritmética multiplicative: f(xy) = £(&)£(y) e, portanto: g(rs)= Oe) “{ r a ( E £(y) ) xlr,y|s x|r yls = g(r)g(s) isto 8, a fingao avitmtica g & muiltiplicativa. 227 Exemplo 14.3 Verificar que a fungdo aviimética g(a) @ mil tiplicativa para n = 24. Temos 24 = 3.8 e 0 mdc(3,8) = 1. Pottanto: g(3.8) = £ £(d) = al24 = £(1) + £(2) + £(3) + £(4) + £(6) + £(8) + + £(12) + £(24) ou seja: (338) £(1.1) + £(1.2) + £(1.3) + £(1.4) + + £(2.3) + £(1.8) + £(3.4) + £(3.8) = = £C.)£(1) + £CL)E(2) + (1) (3) + £01) £(4) + + £(2)£(3) + £CL)E(B) + £(3)E(4) + £(3)£(8) isto é: £0.8) = [ay + £03) [E@ + £0) + ea) + £()] = Z£(d) . £ £(d) = g(3)g(8) a{3 als 14.3. FUNGAO DE MOBIUS Definigao 14.3 Chama~se fungdo de MOBIUS a fungao aritmé tiea u assim definida: 228 1 se n=l u(a) = | 0 ‘se p"|n, sendo p um primo PyPo+++P, onde os Py sao primos distintos Assim, peexe: (6) = u(2.3) = (+1)? = 1 u(12) = u(27.3) = 0 (42) = (2.3.7) = (-1)3 = -1 (250) = u(2.5°) = 0 Note-se que, se p @ um primé, entao: up) = -1 e u(pt) -0 para k>2 A tabela abaixo d4 os valorés de u(n) para os dez primei- ros inteiros positives: a (4, 464-458 (ie 8 ua) La Sees eo Ue Ct Teorema 14.3 A fungdo u de MOBIUS © uma fungao arttméti- ca mltiplicativa. Demonstrat Sejam re s dois inteiros positives tais que o mdc(r,s)=1. 229 Cumpre demonstrar que u(rs) = u(r)u(s). Se r=1, entao: u(rs) = u(s) = l.u(s) = u(1)u(s) = u(r)u(s) Analogamente, se s = 1, entdo u(rs) = u(r)u(s). Suponhamos, ‘agora, r>1es>1. Se p-|r ou p“|s, sendo p um primo, entao p*|rs, e tems: u(rs) = 0 = u(r)u(s). Se, ao invés, re s nao sao divisiveis pelo quadrado de qualquer primo, entao: T= PyPoee © 5 = gyda, onde os primos Pred sao todos distintos, e temos: urs) = u(pyPp+++Padydye+-4,) = = (-D™™ = (-17(-1)" = ulr)u(s) © que completa a demonstracao do teorema. Teorema 14.4 Se n @ um inteiro positivo, entao: se n=1 geo 0 se n>1 Demonstragao Consideremos a funedo aritmética g assim definida: g(n) = £ u(d) d[n 230 Se n= 1, entao: sg) = Dua) = ul) #1 aj Suponhanos, agora, n>1. Sens p*, sendo p um primo e k>1 um inteiro, entao os epractes positivos de Pe sao os k+l inteiros: 1, p, P* ’ 2390 » © temos: a") = E uC) = ul) + ulp) + ulp’) + ee + up®) dip ou seja: get) = 5 fia = u(1) + u(p) = 1 + (-1) = a|p* kok Re ie a Finalmente, se n= p,'p,* ... Py” @ a decomposigdo candni ca de n, como a fungao ude xostus € multiplicativa (Teorema 14.3), entdo a finpdo artimética g também 3 mul- tiplicativa (Teorema 14.2), ¢ temos: Kk k 8a) = (Py) )apy") ... (+) = 0 Suponhamos, p.ex., n= 10. Como os divisores positivos de 10 sao 1, 2, 5 e 10, temos: E u(d) = u(1) + u(2) + u(5) + u(10) = af10 1+ (1) + (-1) +120 231 14.4. FUNGOES ARITMETICAS MULTIPLICATIVAS COMPLETAS Definigdo 14.4 Uma funcao aritmética f: N + Z diz-se uma fungdo artimética miltiplicativa completa se £(rs) = £(r)£(s) para todo par de inteiros positivos re se Assim, p.ex., a fungao aritmética f definida por £(n) @ uma fungdo aritmetica mltiplicativa completa, pois, quaisquer que sejam os inteiros positivos r e s, temos: 3 £(rs) = (rs)? = 1's? = £(r)£(s) As fungoes aritmética d(n), s(n) e u(n) ndo edo fungdes avitméticas mltiplicativas completas, pois, temos: (2.10) = d(20) = 6 # 2.4 = d(2).d(10) s(2.10) = s(20) = 42 # 2.18 = s(2).s(10) u(2.10) = u(20) = 0 # (-1).1 = u(2).u(10) 14.5 FUNCAO MAIOR INTEIRO Definicao 14.5 Para todo niimero real x indica-se por [x] o mator inteiro que nao supera x, isto é, [x] @0 imteo | inteiro que satisfaz 4 condigao: x - 1 < [x] R definida por £(x) = [x] chama-se "fungao maior inteiro". Nao € uma fungao aritmética, por- que o seu dominio @ 0 conjunto R #N dos nimeros reais, mas desempenha papel importante no tratamento dos proble- mas de divisibilidade. Observe-se que Eud) = [1m | ajn = onde u @ a fungao de MOBIUS. A "fungdo maior inteiro” também € denominada "fungao col- chéte" ou "funedo escada" (por virtude do seu grafico car tesiano). Também se costuma consideraraa fungdo {x}= x - [x], de- nominada parte fraciondria de x. Assim, p.ex 233 {7}=0, {2,6}=0,6, {-4,75}= 0,25 14.6 FORMULA DE INVERSAO DE MOBIUS Teorema 14.5 Sejam £ e g duas fungdes aritnéticas relacig nadas pela igualdade: f(m) = £ g(d) jn Entao: g(a) = F u(d)£(n/d) din Demonstragao: Obviamente, temos: Tu@)e(n/d) = Llad) = gle) d|n | dln | (i/d) =f Zo u(d)g(c) dln \ ¢| (n/a) A Ultima soma dupla @ sobre todos os pares de inteiros po- sitivos (c,d) tais que din e c|(n/d). E como din e cl(n/d) se e somente se c|n, e d|(n/c), temos: 234 z E u(d)g(c)\ = £ E u(d)g(e) } = d[n | | (n/ay eln \ al (n/c) = = fg) £ utay eln | (n/c) Consoante o teorema 14,4, a soma = ua) d| (n/e) temo valor 0 se n/e>1 e temo valor 1 se n/e=1 ou n= c. Assim sendo, temos, finalmente: E u(d)f(n/d) = Z g(c).1 = g(a) dln c=n Note-se que E u(d)f(n/d) = £ u(n/d)£(d) = g(n) aln jn Assim, p.ex., no caso das fungoes aritméticas d(n) e s(n) temos, por definigao: d@y= Zl e s(n) = Ed eln din e, portanto, pela formula de inversao de MOBIUS: l= 2 ule)d(n/e) = 2 u(n/e)d(e) eln eln n= £ u(d)s(n/d) = © u(n/d)s(a) d[n d{n formulas validas para todo n>1. 1. EXERCICIOS Mostrar que @ multiplicativa a fungao aritmética £(n) = n¥, onde k @ um inteiro fixo positive. Mostrar que, se f(n) e g(n) sao fungoes aritméticas multtplieativas, entao a funeao aritnética h(n) = £(n)g(n) também @ multiplicative. Mostrar que, se f£(n) e g(n) sao fungdes arttméticas muitipticativas tais que £(p*) = g(p*) para todo p primo e k2>I1, entao f=, Sendo u a fungdo de MOBIUS, demonstrar: (a) u(n)u(m + L)u(n + 2)u(n +3) = 0, Yael (b) ul!) + .u(2!) +...4 u(nt) = 1, Yu23 236 5. Resolver as duas seguintes equacées: (a) [e+ 3] =x+3 (>) + 3} =3+ Ey 6. Resolver a equacgao: tl + [x] = (2x). 7. Sejam x ¢ y nimeros reais. Denonstrar as seguintes pro priedades da "fimgdo mor intetro": (2) [s+] = [x] +0, Vnez @®) + Eel 0 owl, consoante x€Z ou xfz (©) B+ b] < keys B+ f)ea @ & f) < by] e caPituto 15 FUNCAO E TEOREMA DE EULER 15.1 FUNGAO DE EULER Definigdo 15.1 Chama-se fungdo de EULER a fungao aritméti ca (fi) assim definida para todo inteiro positivo n: $(n) = niimero de inteiros positivos que nao superan ne que sao primos com n+ Em outros termos: $(n) = numero de elementos do conjunto {xE€N] l¢gxgn e mdc(x,n) = 1} Em particular, $(1) = 1, porque o méc(1,1) =1,e para n>1, 0 mde(njn) =n #1, de modo que 4(n) = niimero de inteiros positivos menores que n e primes com n. Esta importante funcao aritmética 4$(n) também @ denomina da o indicador de ne, menos frequentemente, o totalizador den. 238 Exemplo 15.1 Se n = 30, entao os inteiros positivos steno ves que 30 e primos com 30 sao 1,7,11,13,17,19,23 e 29,de modo que $(30) = 8. Exemplo 15.2 Se n = 12, entZo 0 conjunto { wEN| L1 e s>1. Neste caso os inteiros de 1 a rs podem ser dispostos em r columas com s inteiros em cada uma delas, do seguinte modo: z ee « “h aie . meer r+2 rth 2x ar4+1 an +2 arth 3r (s-l)r +1 (s-l)r 2 (s-1)r+h sr Por ser o mdc(qr + h,r) = mdc(h,r), os inteiros da h-ési- ma coluna sao primos com r se e somente se 4h @ primo com r E como na primeira linha o nimero de inteiros que sao pri- mos com rx @ igual a $(r), segue-se que existem somente $(r) colunas formadas com inteiros que sao todos primos com r. Por outro lado, em cada uma destas 9(r) colunas 240 existem precisamente $(s) inteiros que sao primos com s, porque na progressao aritmética: h, rth, 2rth, ..., (s-L) rth onde o mdc(h,r) = 1, 0 numero de termos que sao primos com s @ igual a $(s). Assim sendo, o niimero total de in- teiros que sao primos com re coms, isto @, que sao pri- mos comrs, @ igual a (r) 6(s), e isto significa que o(rs) = 6(r) o(s). 15.2 CALCULO DE (n) Teorema 15 Seo inteiro n>1, entao $(n)=n-1 se e somente se n @ primo. Demonstragao: (——) se n>1 @ prim, entao cada um dos inteiros po sitivos menores que n é primo com n e, portanto, o(n) en - 1. (<==) Reciprocamente, se $(n) =n- 1, com n>1, en~ tao n @ primo, pois, se n fosse composto, teria pelo menos um divisor d tal que 11, entao: 242 kK kl &, ky-1 k 9 Gl = Goes Py) @,y ou seja: den I GL = Vp.) tet i Demonstragao: * Usarenos o "Teorena dz indupiio matenitica” sobre x, niimero de fatores primos distintos de n. A proposicgao @ verdadeira para r = 1 (Teorema 15.3). Supo- ~ nhamos, entao, a proposicao verdadeira para r =i. Como o em) Fe eat nde(p;'p,” ... pt, pit) e $(n) @ uma fmgio ariimética multiplicativa (Teorema 15.1), temos: ke KAI] kk ke k +1 slog FA seePy Pel iss 40," Py oo By) OCP ) za ou seja kel] k, kk, ¢ (,) 29 see k, Piet |r 4p, "p,2 243 ou, 4 vista da hipdtese de indugdo: k, k, k, k.+1 k, k,l k. hae dig t 2 Op Pg PE Pigg) = (Py dee, : ky kyl e (; Py e isto significa que a proposicdo é verdadeira para r=i+l: Logo, a proposigao @ verdadeira para todo inteiro positivo ne Exemplo 15.4 Calcular $(7865). ; a Por ser 7865 = 5,117.13, temos: (7865) = (5 ~ 1)(11? - 11) (13 - 1) = 4,110.12 = 5280 Exemplo 15.5 Calcular (1350). Por ser 1350 = 2.37.57, temos: (1350) = 1350(1 - 1/2)(1 - 1/3)(1 - 1/5) = = 1350.1/2.2/3.4/5 = 360 264 15.3 PROPRIEDADES DA FUNCAO DE EULER Teorema 15.5 Para todo inteiro positivo n>2, o(n) é um inteiro par. Demonstragao: Se n @ uma poténcia de 2, isto é, n k22, en- tao, pelo teorema 15,3: 1 (mn) = (2) = 2k - aya) = que @ um inteiro par. Se, ao invés, n nao @ uma poténcia de 2, entao n @ divisi- vel por um primo impar p, isto é: ne pn, onde k21 © 0 mac(pYm) =1 E,como $(n) € uma fumgdo artimética mitiplicativa (Teore ma 15.1), temos: om) = a0) o¢m) = 2h - 1) om) que @ também um inteiro par, porque 2|(p - 1). . Assim, $(n) @ um inteiro impar somente para n=1 e n=2: @Q) = $(2) = 1 245 Teorema 15.6 Se p @ um primo, entao: ‘ i = op) = isd Demonstragao: Pelo teorema 15.3, temos: 0p) = p= o(p?) op?) = cae 2. » s ' + cy} + ~ + + ~ 7 i ee ou seja: Portanto: k : k k E o(p') = 61) + pk - 11+ pe -1sp i-o 248 Teorema 15.8 Para todo inteiro positivo m,>1, a somtdos inteiros positiyos menores que ne que sio primos comn & igual a L Fa. ola) Demonstragao: Sejam AV 829 > Apcay os inteiros positivos menores que n e que sao primos com R. Como o mdc(a,n) = 1 se e somente se o mdc(n-a, n)=l, os inteiros positivos memores que ne que sao primos com n podem ser expressos pelas diferencas: Rays May, vey MAB) Portanto: ay + ay st bq) 7 (nra,) + (nray) +e. 64 (a-ag (gy) “ = a.¢(n) - (yt ay t+ ay cy) o que implica: ay tay tet Og CQ) t a. b(n) oa Exemplo 15.8 Verificar o teorema 15.8 com n = 15. Por ser 15 = 3.5, temos: 905) = G-G-1) = 2.428 247 @ igual a $(n/d). E como cada um dos n inteiros 1,2,3,... +eeyM pertence precisamente a uma unica classe Sq» temos:. Z $(n/d) =n, d|n Mas, quando d percorre todos os divisores positivos de n, © mesmo ocorre com n/d e, portanto: E o(n/d) = £ $(n) =n dln dln * Exemplo 1 Verificar o teorema 15.7 (de GAUSS) com n=10. Os divisores positivos de 10 sao 1,2,5,10 e, portanto, as classes $4 sao: {m|1l1 inteiros tais que o mdc(a,n) = 1. Se Bye Bpy wees Agen 8a0 08 inteiros positivos menores que n e que sao primos com n, entao cada um dos inteiros: p> AA), ve+y 48g ¢Q) & congruente médulo na um dos inteiros Ayr Aye ser Agcy) (nao necessariamente nesta ordem), Demonstrag i i intei a, a8 sao in- Dois quaisquer dos inteiros aay, 2a,,..., aay.) congruentes médulo n, pois, se fosse: 250 2a ¢ = aa; (mod.n), com 1 n)=1 e, portanto, podemos cancelar o fator comum aya, 192 «°° oq)? © que da: a9) = 1 (mod.n) Note-se que, se p @ um primo, entao $(p) = p-l, e seo mic(a,p) = 1: aPl = 1 (mod.p) que & 0 teorema de FERMAT. Assim, o teorema de EULER é uma generalizagdo do teorema de FERMAT. Exemplo 15.9 Verificar 0 teorema de EULER com n=9 e a=-4, 0 mdc(-4,9) =1 e (9) = 6. Portanto: (-4)PO) = (-4)% = 4096 253 E como 9/ (4096 - 1), segue-se que 4096 = 1 (mod.9), isto 8: (4%) = 1 (moa.9y Exemplo 15.10 Usando o teorema de EULER, resolver a con- gruéncia linear ax = (mod.m), onde o mde(aym) = 1, Pelo teorema de EULER, temos: 2). om) 1 (mod.m), ou: = (mod.m) Portanto: ax a? (a) b (mod.m) Como o mde(a,m) = 1, podemos cancelar o fator comum a, 0 que da: x= aP™"ly (nod nm) Consideremos, p.ex., a congruéncia linear 3x = 5 (mod.8), onde o mdc(3,8) = 1. Temos: 30-E 5 2 hs = 135 (mod.8) x 15.5 RELAGAO ENTRE AS FUNGGES $ Eu’ Teorema 1 Para todo inteiro n>1: o(n) =n £ u(d)/d dln 254 Demonstragao: Pelo teorema 15.7 (de GAUSS), temos: n= 2 $(da) ala @, portanto, pela forma de inversao de MOBIUS: om) = EF u(d)(n/d) =n £ uld)/d d[n dja fSrmula importante que relaciona as fungoes 6 de EULER e u de MOBIUS, Assim, p.ex.: 9(8) = 82 uld)/d = B(u(1)/1 + u(2)/2 + u(4)/4+u(8) /8)= als = (1/1 + (-1)/2 + 0/6 + 0/8) = 8M. - 1/2) = 4 10. 255 EXERCICIOS Caleular: (420), 91001), 9$(5040) e (8316). Caleular: d($(15)) e 9$(d(15)). Calcular: ($(p)s(p) + 1)/p, sendo p um primo. Verificar que $(n+2) = $(n) + 2 para n = 12,14,20. Verificar que $(n) = $(n+1) = $(n+2) para n = 5186. Verificar que 9(3*.568) = (3.638), onde o inteiro k20, Verificar que $(n) @ uma funedo aritmética multiplica tiva para n = 144. Mostrar que $(n) ndo & uma funedo aritmétion mltiplé cativa completa. Verificar 0 teorema de EULER com n=10 e a= 3. Usando 0 teorema de EULER, resolver as seguintes con- gruéncias Lineares: (a) 5x = 7 (mod.12) (b) 2x = 3 (mod. 9) (c) 7x = 1 (mod.10) (4) 8x = 4 (mod.5) (e) 2x = 1 (mod.17) (£) 5x = -3 (mod.8) 256 ll. 12, 13, 14. 15. 16. 17. 18. 21. : : Demonstrar que (n°) = n.¢(n) para todo inteiro-posi- tivo n. Demonstrar que, se n é um inteiro positivo impar, en- tao: (a) $(2n) = o(n) . (b) $(4n) = 2¢(n) Mostrar que 0 mdc(m,n) = 2 implica (mm) = 29(m)o(n). Demonstrar que o niimero de fragoes positivas irreduti- veis menores ou iguais a 1 com denominador n @ igual a o(n). Verificar o teorema 15.6 com p=3ek=4. Verificar o teorema 15.8 com n= 17 en = 28. Verificar o teorema 15.7 (de GAUSS) com n = 36,105,147. Mostrar que, se n @ um inteiro par, entao: $(2n) = 26(n). . Demonstrar que $(3n) = 30(n) se e somente se 3|n. Demonstrar que $(n) = se e somente se n= 2*, onde k21. Demonstrar que Yn/2< $(n).1 temr fatores pri- mos distintos, entao (a) 20/2", Demonstrar que, se o inteiro n>1 @ composto, entao o@) 1), en- um nimero perfet- to. Demonstragao: seja 2*-1 = p(k>1) um primo. Consideremos o inteiro posi tivo n= 215, como o mic(2*!,p) =1e s(n) & uma fungdo aritmética multiplicativa, temos: 260 s(n) = s(2®1y) = 521) scp) = (2-1) pei) = 12 225 Logo, por definicao, n @ um nimero perfetto. Assim, todas as vezes que se conhece um inteiro k>1 tal que 2" - 1 @ primo pode~se construir um nimero perfetto. Para k= 13, p.ex., tems 2)? - 1 = 8191, um primo,o que di 0 5 mimero perfeito: 120913 _ 1) 2 33550336 = Ps NOTA, Os quatro primeiros nimeros perfeitos se obtem pela fSrmula de EUCLIDES atribuindo a k os valores 2,3,5 e 7,is to é: P= 2(27 = 1) = 6, P, = 27(23 - 1) = 28 Pp, = 2425 = 1) «496, p, = 2627 ~ 1) = size Para k=17 e k= 19, a formula de EUCLIDES da-nos o sexto e 0 sétimo mumeros perfeitos: a 2i6 2l7 Shy ee ePa ia 2}8(919 =) ’, 7 ¢ os quais foram determinados, pela primeira vez, pelo mate- matico italiano PIETRO CATALDI em 1603. Os outros 17 mimeros perfeitos conhecidos se obtem pela 261 formula de EUCLIDES atribuindo a k os seguintes valores: 31, 61, 89, 107, 127, 521, 607, 1279, 2203, 2281, 3217, 4253, 4423, 9689, 9941, 11213 e 19937 Teorema 16.2 (de EULER) Se n @ um niimero perfeito par,en- tao: eel alt ns 2° “(2" - 1) onde 2* - 1 @ primo. Demonstra¢a Suponhamos que n @ um nignero perfetto par. Entao, n pode k-1 ee escrever-se sob a forma: n = 2“ 'm, onde m @ um inteiro impar e k>2. Como o mdc(2*"!, m) =1 © s(n) uma fungdo aritmética maltiplicativa, temos: s(a) = 8(2® 4a) = 5(2* ly 6¢m) = 2 - 1)8(m) Por outro lado, como n @ um nimero perfeito, temos: s(n) = 2n = 2%q Portanto: 2a = (2 - 1)s(m) (1) 262 de modo que (2* - 1)|2"a. was, 0 nde(2* - Lkeay ‘6 que implica: QF 24) |a; Fate fm =k 2 bs Substituindo este valor de mem (1) e cancelando o fator comum 2 - 1, obtemos: s(m) = 2% Como me M sao ambos divisores positivos de m (com M2) & primo, entao k também é primo. Demons tragao Suponhamos o inteiro 2* - 1 (k>2) primo. Seo inteiro k fosse composto, entao teriamos k= rs, com r>1l e s>l, © que implica: 2-1 227-12 @tye-1 ou seja: gee a = at - Qt) sD 4s oF 1) Como r>1, os dois fatores do segundo membro sao ambos maiores que 1, isto é, 2-1 @ um inteiro composto, o que contraria a hipdtese. Logo, k @ primo. NOTA. 0 reetproco deste teorema 16.3 @ falso, isto@, k primo nao implica 2-1 também primo. Assim, p.ex., 11 primo e no entanto 2/1 ~ 1 & composto, pois, temos: ght - 1 = 2047 = 23.89 264 Teorema 16.4. Todo nimero perfeito par termina em 6 ou 8. Demonstraga Seja n um numero perfeito par. Pelo teorema 16.2 (de EULER), temos: angel gk ay, ante! 2 = 18 pea E, pelo teorema 16.3, sendo 2 - 1 primo, k também @ primo. Se k=2, entao n=6,ea proposigao @ verdadeira. Supo nhamos, pois, k>2. Todo primo mator que 2 € da forma 4m+1 ou 4m+3. Se k @ da forma 4m+1, entao: ia 2am antl 3}; a 2omtl - 24m. 2.162" ~ 167 Por ser 16° = 6 (mod.10), qualquer que seja o inteiro posi tivo t, segue-se que n= 2,6 - 6 = 6 (mod.10), isto &: n= 10h + 6 @, portanto, n termina em 6. Analogamente, se k @ da forma 4m+3, entao: gimme? cgtm+3 _ 4) | 8mt5 _ bm? _ n = 2,162 - 4 gt Por ser 16° = 6 (mod.10), qualquer que seja o incairo posi 265 tivo t, segue-se que 2.6 - 4.6 = -12 = 8 (mod.10) isto @: , portanto, n termina em 8. Gorolario 16.1 Todo nimero perfeito par n @ congruente a 6 médulo 10 ou congruente a 8 midulo 10, isto é: n = 6 (mod.10) ou n= 8 (mod.10) Teorema 16.5 Um primo nao pode ser um nimmero perfetto. Demonstracao: Seja p um primo qualquer. Entao, s(p) =p +1. Sep é@ um nimero perfeito, entao s(p) = 2p. Portanto: p+l=2p e p#=1 Como p22, segue-se que p nao @ um mimero perfetto. Logo, um primo nao pode ser um nidmero perfeito. 16.2 NOMEROS MULTIPERFEITOS Definigao 16.2 Um inteiro positivo a diz-se que um nimero 266 muitiperfeito de ordem k ou um k-niimero perfeito see “so- mente se s(n) = kn, onde k23 @ um inteiro. 5.39.5.7 @ um nimero multiperfeito de ordem 4, pois, temos: Assim, p.ex., 0 inteiro positive 30240 = 2 sn-1 7-1 a-k eee 6 s(30240) = 2 = = 63.40.6.8 = 120960 = 4.30240 16.3 NUMEROS AMIGOS Definigao 16.3 Dois inteiros positivos me n dizem-se mii- meros amigos se e somente se a soma dos divisores positi- vos de m, exeeto o divisor m, é igual an, e a soma dos di visores positivos de n, eaceto o divisor n, @ igual a m. Em outros termos, dois inteiros positivos m e n dizen-se miumeros amigos se e somente se s(m)-m=n e s(n)-n=m ou seja, o que @ equivalente: s(m) =m +n = s(n) 267 Exemplo 16.1 Mostrar que os inteiros 220 e 284 sao nime- ros amigos. Por ser = 2 220 = 2°.5.11 e 284 = 2°.71 temos: 8(220) - 220 = s(27)s(5)s(11) - 220 = = 7.6.12 - 220 = 504 - 220 = 284; 8(284) - 284 = s(27)s(71) - 284 = = 7.72 ~ 284 = 504 - 284 = 220 isto é@: 8(220) = 220 + 284 = s(284) Logo, por definigdo, os inteiros 220 e 284 sao nimeros amt gos. 16.4 NOMEROS DEFICIENTES E ABUNDANTES Definicgao 16.4 Um inteiro positivo n diz-se um mimero de- ficiente se e somente se s(n) -nn ou s(n)>2n Assim, p.ex., os inteiros 15 e 18 sao respectivamente um nimero deftetente e um niimero abundante, pois, temos: (15) = 8(3-5) = s(3)s(5) = 4.6 = 24<2.15 8 (18) = s(2.37) = s(2)s(3*) = 3.13 = 39>2.18 16.5 NOMEROS DE MERSENNE Definigdo 16.5 Chama-se nimero de MERSENWE todo inteiro positivo da forma: uM = 2"- 1 (a2) a Se M, @ primo diz-se que @ um primo de MERSENNE. A tradicional denominagao de “niimeros de MERSENNE" dada aos inteiros M, @ uma homenagem ao matematico do século XVII MARIN MERSENNE (1588-1648) que, interessado em encoa trar nimeros perfeitos, estudou os inteiros M,. Como re- sultado das suas pesquisas MERSENNE fez em 1644 a seguinte afirmagdo: Todo inteiro 4° 2? - 1 @ primo para p = 2,3,5,7513,17,19,31,67 127,257 e@ & composto para todos os outros primos p< 257. 269 Esta afirmagao, entretanto, @ incorreta, pois, sabe-se ho- je que sao primos de MERSENNE os 24 seguintes inteiros: My aM 9B 9My 99.7 9My goM3 1 Mey Mag ot g7 9M 27 Msi Meo7i279> “o203>M22¢1 -M3217°Mu253Maa23Mg6g9° Myou1 11213-19937 € que para todos os demais valores de p<257 os inteiros 4, sa0 compostos, exceto possivelmente para p = 157, 167, 193, 199, 227, 229 porque, para estes 6 valores de p nao se sabe ainda se M, @ primo ou se @ composto. Como se vé, MERSENNE cometeu dois enganos: inclutu Mey e Mj57 na sua Lista de primos e exclutu dessa lista ¥, Mag © Mig7- E interessante notar que os quatro primeiros primos de MER SEWNE sao: 61’ Mn3, Mo 7, y= 31, M, = 127 e que =M., =M, ha, =M. Ma Mae Mast Mae Ma” Mar 4° M27 também sao primos de MERSEWNE. Entretanto, o quinto primo de MERSENNE @ M3 = 8191, mas 7 270 oo oat Ya" “eri nio“é wn primo de MERSENNE, pois, & um inteiro composto. com 2466 algarismos. Assim, @ falea a conjectura: Se M, @ um primo de MERSENNE, entao My, também é um primo de MERSENNE. 5 Teorema 16.6 Se peq-= 2p +1 sao primos, entao: IM IM +2 aM, ou als, Pp mas q nao divide conjuntamente MeM, +2. Como q @ primo eo mdc(2,q) = 1, temos, pelo teorema de PERMAT: 20-120 (mod.a) ou, fatorando o 1? membro: qrl) /2 1)/2 «2 - pF? 41) = 0 (moda) ou seja: + 2) = 0 (mod.q) Mery /2 “eq-1)/2 ou, por ser (q-1)/2 = MAM, + 2) = 0 (mod-q)- 271 Assim, q @ um primo que divide o produto MoQt, + 2) e,por tanto, alm, ou aM, +2 (orolario 7.1). Mas, q ndo di- vide conjuntamente My e M, +2, porque, entao, q{|2,o que @ impossivel. Exemplo 16.2 Demonstrar que o nilmero de MERSENNE M, composto. ary Como os inteiros p= 23 e q = 2.23 +1 = 47 so primos, Segue-se que 47|My3 ow 47|My4 + 2 © cumpre demonstrar que se verifica a primeira alternativa Com efeito, temos: 23 0 23(25y4 = 23¢-15)4 (moa. 47) e (-15)* = (225)? = (-10)? = 6 (mod. 47) Portanto: 1 (mod .47) isto @: 223 ~1=4M,, = 0 (mod.47) e isto significa que 47|/M,,, de modo que M,, @ composto - 272 16.6 NOMEROS DE FERMAT Definigao 16.6 Chama-se nimero de FERMAT todo inteiro po- sitivo da forma: +1 (n20) Se F, & primo diz-se que @ um primo de FERMAT. Observou FERMAT, em 1640, que os inteiros positivos: Fo=3, Fy 2d, Fy 217, Fy = 257, Fy = 65537 S40 todos primos e conjecturou: Fé primo para todo in- teiro n20. Entretanto, EULER, em 1732, derrubou a conjectura de FERMAT, mostrando que F, @ composto - precisando que 641/F5, isto @: = 2°° + 1 = 4294967297 = (641). (6700417) Até hoje nao se conseguiu encontrar um primo de FERMAT dis tinto dos cinco primeiros. Realmente, para 5F, sao inteiros compostos. Teorema 16.7 Se Fe F, sao dois nimeros de FERMAT, com m>n20, entaoo mdc(F_,F_) = 1. nm Demonstragao: Seja d= mdc(F,,F,). Como os niimeros de FERMAT so intei- ros impdres, segue-se que d tambem @ um inteiro impar. Te- mos: F-2 m a 2 me a -pie += ‘a earn 2 -[ 0? far | 1? #1) a Denotando, para simplicidade de notagao, 27 por xe 2m por k, temos: Fr? yk F x*t m de modo que F.|(F, - 2). Como a|F,, segue-se que a] CF, - 2). Mas, d|F_e, portanto, d|2. E como d @ um inteiro émpar, temos d=l1. EXERCICIOS Mostrar que o inteiro 130816 nao @ um numero perfeito. atte 10,11 Sauk a Mostrar que o inteiro n= 2° (2°" - 1) nao @ um nime- ro perfeito. Demonstrar as seguintes proposigoes: (a) Nenhuma poténcia de um primo @ um niimero perfetto. (b) Um quadrado perfetto nao pode ser um ntonero perfet to. (c) 0 produto de dois primos imares nunca € um niime ro perfeito. Demonstrar que, se n @ um nimero perfetto, entao: Li/d = afi Demonstrar que todo nimnero perfeito par @ um ninero triangular. Demonstrar que se n é um nigmero perfetto par: 2-1), a entao: (a) nel+2+3e...4 (2-1 (b) O(n) = 2 QE - 1y 10. ll. 12. 13. 275 Mostrar que, se n>6 € um nigmero perfetto par, entao a soma dos seus algarismos @ congruente a 1 (mod.9). Mostrar que nenhum divisor de um nigmero perfetto pode ser um nignero perfeito. Achar os dois Gltimos algarismos do mimero perfeito: n = 219936 (519937 _ ¢ D Mostrar que, se n @ um ntmero perfeito par, entao 8n+l & um quadrado perfeito. Mostrar que os inteiros 120 e 672 sao nimeros multiper feitos, Mostrar que o inteiro: (a) 523776 & um 3-nimero petfeito; (b) 2178540 € um 4-niimero perfeito; (c) 14182439040 € um 5-niimero perfeito. Demonstrar as seguintes proposicoes: (a) Se n @ um 3-niimero perfeito e se 3m, entao 3n @ um 4-niimero perfeito. () Se n @ um S-niimero perfeito ese 51a, entao 5n é um 6-numero perfeito. (c) Se 3n @ um 4k-nuimero perfeito e se 3 entao o @ um 3k-nimero perfeito. 276 14, Mostrar que 120 e 672 sao os imicos 3-niimeros perfei— 15. 16. 7 18 19 20. 21 22 tos da forma n= 2.3.9, onde p @ um primo impar. Demonstrar que, se n>6 @ um niimero perfeito par,en- tao n @ congruente a 1 (mddulo 6). Mostrar que sao mimeros amigos os inteitos: (a) 2620 e 2924 (b) 17296 e 18416 Classificar os inteiros 2,3,4,...,21 como niimeros defi ctentes, perfeitos ou abundantes. Mostrar que o inteiro 945 = 39.5.7 @ um mimero abun- dante e determinar um outro nimero abundante da forma ke 3.5.7, Demonstrar que todo miltiplo de um niimero perfeito @ um mtmero abundante. Mostrar que, se n>3 ese 2n+l @ primo, entao 2n(2n+1) & um nimero deficiénte. Mostrar que, se n & um niimero perfeito par e se d di- vide n (d|n), onde 1n, entao m@ um nimero deficiente. a (>) Se mé pare sen perfetto. impar, entdo n é um quadvado Determinar 0 primo de MERSENVE M, 9. Mostrar que os miwneros de MERSENNE M11 & Mog sao in- teiros compostos. Um inteiro positivo n diz-se um nimero superperfetto se e somente se s(s(n)) = 2n. Mostrar que, se n= 2* + Bi eerie mantis ese 2T 1 & primo, entdo n um nimero ouperper- fetto. Mostrar que 5 e 7 sao os imteos primos gémeos cuja se- mi-soma @ um munero perfeito. 7 K-1 (ok a : Seja n= 2" (2° - 1) um niimero perfeito par. Demons trar que o produto dos divisores positivos de n @ igual a n*, caPituLo 17° NUMEROS DE FIBONACCI 17.1 SEQUENCIAS RECORRENTES Consideremos a sequéncia de inteiros: Bye yy Uy seers Cys Uy eee ay na qual cada termo, a partir do tercetro, @ a soma dosdois termos precedentes, isto @, para todo n>3: Q) Tais sequéncias, nas quais a partir de um determinado ter- mo, cada um dos seguintes @ uma combinagao linear de ter- mos anteriores, ocorrem com frequencia em Matematica e re- cebem o nome de sequencias recorrentes. 0 processo pelo qual se determinam sucessivamente os termos particulares destas sequéncias chama-se processo recorrente e uma igual dade da forma (2) @ uma formula recorrente. 279 E Sbvio que se podem construir arbitrariamente um niimero infinito de sequéncias de inteiros satisfazendo 4 condigao (2), tais como, p.ex.: 2,5,7,12,19,31,50, +. 1,3,4,7,11,18,29, +++ -1,-5,-6,-11,-17, Como se vé, a condigao (2) nao basta para determinar univo camente a sequéncia (1), sendo necessario conhecer os seus dots primetros termos para poder calcular todos os outros, isto 8, conhecer ou fixar os termos u, ¢ uy. 17.2 SEQUENCIA DE FIBONACCI A sequéncia de inteiros: Fy Fos Fyy veer F Fin oo no caso em que isto é, a sequéncia: 1,1,2,3,5,8,13,21,34,55,89,144,233,377,.++ cecebe o nome de sequéncia de FIBONACCI e os seus termos chamam-se nimeros de FIBONACCI. 280 Os mimeros de FIBONACCI ‘ou LEONARDO DE PISA, famoso mate- mAtico italiano do século XIII (1180-1250) possuem muitas propriedades importantes e aqui vamos estudar algumas de- las. 17.3 SOMAS DE NOMEROS DE FIBONACCI Teorema 17.1 A soma dos n primeiros nimeros de FIBONACCI iguala Fy )-1. Demonstragao: Com efeito, temos: Somando ordenadamente todas essas n igualdades e simplifi- cando, obtemos: FL +F,+F3 + 281 Assim, p.ex.: Lele2*3e5+04134 Mary 1 = 55-1 = 54 Teorema 17.2 A soma dos n primeiros nimeros de PIBONACCT com indices impares @ igual a Foat Demons tragao: Com efeito, temos: Fon-3 7 Fon-2 ~ Fang Font * Fon ~ Fon-2 Somando ordenadamente todas essas n igualdades e simplifi- - cando, obtemos: FL + Fy + Fy t 2.4 F, =F, 2a-3 * Font * Fon Assim, p.ex.: L+245 +413 + 34+ 89 4 233 =F y= 377 282 Zeorema 17.3 A soma dos n primeiros nimeros de’ FIBONACCI com indices pares é igual a F, = 1s 2n+1 Demonstragao: Com efeito, temos: 7 Fone 7 Fon-3 an ~ Fanet ~ Fon-1 Somando ordenadamente ‘todas essas n igualdades e simplifi- cando, obtemos: Fe Fol Bak S06 ts -* Prog) Ton — Pont 15 Panel Assim, p.ex.: L+ 348+ 21455 + 144 =F, - 1 = 233-1 = 232 Teorema 17.4 A soma dos n primeiros nimeros de FIBONACCI tomados alternadamente com os sinais + e - @ igual a u Come ea. 283 Demonstraga Com efeito, subtraindo ordenadamente as igualdades corres- pondentes aos teoremas 17.2 e 17.3, temos: +F - F, ant ~ Fan * Fan” Pome * 2 = Fuigta * E; ~ 84 = “Fon tl (3) Somando F,,,; 4 ambos os membros desta igualdade, temos: By EEaPES treet Bod Yow Fone = Ebg atl + * For 7 Fog tl (4) Consoante as igualdades (3) e (4), temos: mL, papyntl, TEST Ry tet GDMIB = GME et Assim, p.ex.: P-L 2-345 8 413 2 = = CDF, +1 2-13 41512; Lrl+27345-84+13-214 4% = = (le +1 = 2141 = 22 284. 17.4 SOMA DOS QUADRADOS DE NOMEROS DE FIBONACCT: ~~ Teorema 17.5 .A soma dos quadrados dos n primeiros mimeros de FIBONACCI @ igual a Fel’ Demonstragao: 2 Por ser F,) =F) = 1, temos (F,)° = FFy, @ para k>1, temos: - = 2 Ftd ~ Fea Fe 7 Fe Fear” Fay) = Fd Fazendo nesta igualdade k = 2,3,...jn, temos: 2 @y" = Fy 2. - (B)* = FOF, - FF, 2 (8y)" = FAR, ~ ELF 2 yD” = FoeaFa 7 Faa2Fnet @* FFoe 7 Fiea’a somando ordenadamente todas essas n igualdades e simplifi- sando, obtemos: 2 2 2 2 Pow By" + Ey)" + Fy)” tet (Fi * (B) = 285 Assim, p.ex.: vit? 22 4 3? a 5? 4 8? 6 23? » = FUR, = 13,21 = 273 Realmente: L+1+4+9+ 25 + 64 + 169 = 273 17.5 IDENTIDADES ENTRE NOMEROS DE FIBONACCI Para cada valor de m usaremos o "Teorema da indugGo matemi tica" sobre n. A identidade subsiste para n= 1, pois, te mos: FP + Fuky * Pee +2, Fuel 1 m1 m Suponhamos, agora, que a identidade subsiste para n = 1,2, seesk, @ demonstremos que também subsiste para n= k+ 1. Entao, temos (hépdtese de indugao): Pek 7 Fate * FaPicet Fat¢e-1) 7 FaaPe-1 * FaPk 286. Somando ordenadamente estas duas igualdades, obtemos:- Farle * Fe(ket) * Fart Fe * Puen) * FaFysat Fi) ou seja: Fatcket) ~ Facet * FaFee2 que @ precisamente a identidade (I) PARA n= +1. Logo, a identidade (I) subsiste para todos os valores inteiros e positives deme n. Assim, p.ex.: F °F, = FF, + F, 643 SF 3 6F4 = 5-2 + 8.3 = 34 9 2 1 (1) uy)” = FFnag * CD ntl mnt Demonstragao: Usaremos o "Teorema da indugdo matemitica". A identidade subsiste para n= 1, pois, temos: a 2 = S , =1 =1.2-1=* FF, + (-1) Suponhamos, agora, que a identidade subsiste para o intei- ro positive k (hipdtese de indugdo), isto é: 2 k a)" Mean * OU 7 287 Somando aos dois menbros o inteiro positive F.4jF,,., tere mos, sucessivamente: Fy)? + FeetFaz 7 Fifce2 * MeaFeag + Cae Fen Pet * Fara) 7 Fea + Fay) + CD* Fens ~ Faz)? + CDE Fao) = Facey t CIMT isto @, a identidade (II) subsiste para o inteiro positivo k+l e, portanto, subsiste para todo inteiro positivo n. Assim, p.ex.: Bee 4 5 (Fg)" = 8° = 13.5 L= FoF) + (-1) oan 6 (Ey)" = 1S" = 21.8 + 1 = FEF, + (1) 17.6 PROPRIEDADES DOS NOMEROS DE FIBONACCI Teorema 17.6 Dois nimeros de FIBONACCI consecuttvos sao primos entre st, isto é, 0 nde(F,, Fi,,) = 1 para todo n+l inteiro n21. Demonstragao: Determinando o mic(F,, F.,,) pelo algorttmo de BUCLIDES, 288 temos: By = 2.F, +0 Portanto: mdc(F_, F a an) =, baie WOTA. Os nimeros de FIBONACCI: Fz=2, Fo +5, F) 13, Fy = 89 sao todos primos, e por conseguinte é natural conjecturar que FL & primo para todo indice n>2 que é primo. Mas esta conjectura @ falsa, porque Fig @ um inteiro compos- to: Fig = 4181 = 37.113 Teorema 17.7 Fay @ divistvel por F, (myn 21). Demonstragao: Para cada valor de m usaremos o "Teorema da indugdo matema 289 tica" sobre n. Como a proposigao @ verdadeira para n=l, suponhamos que F, @ divisivel por F, para nel,2,...,k (hepotese de indugao), e demonstremos que Fyij,1) & divi stvel por F_. Temos, pela identidade (I): Facet) * Ficem ~ Fk-1¥m * Fuente Como F divide Fuel © Fak (hipdtese de indugdo), segue-se que F divide o Gltimo membro das igualdades acima e, por tanto, F,, divide F,(,,1)- Logo, F.,, @ divisivel por F, pa ra os valores positivos de me n. LEMA. Se m= nq + r, entao o mde (F,sF,) = mde (F,,F,). Demons trag: Usando a identidade (I), temos: nde (FF) = mde(F, Fw mde Fg Ft F, ngtr? “nv fet Faghrsa’?n? Mas, se bic, o mdc(atc, b) = mdc(a,b), ‘e como Talbaa (Teorema 17.7), segue-se que o nde(F,,F,) = mde (F, ) = mde(F Fs) fnq-ite * Fag’ret* Fa: anger? Posto isto, vamos demonstrar que o mdc(F,q_),F,) = 1. De fato, seja d= nde (F, FA). As relagoes: iq-1? ale, e Flag implicam alg de modo que d @ um divisor positivo comum dos niimeros con- seeutivos Fe Fg ‘¢ FIBONACCI. Logo, pelo teorena 17.6, d=1, isto 0 méc(Fy, F) = d= 1. Finalmente, como o mdc(a,c) = 1 implica mde(a,bc) = 7 mdc(a,b), temos: S mdc(F.yF,) = mc aq eFn) = ndc(F, 5F_) e a proposicao fica demonstrada. Teorema 17.8 0 mdc de dois nigneros de FIBONACCI também @ um numero de FIBONACCI: ~ mdc(F a’ Fy) "Fy, onde d = mde(m,n) Demonstragao: Suponhamos m>n. Determinando o mdc(m,n) pelo algoritmo de EUCLIDES, obtemos as seguintes igualdades: meh try , O O O Seguerse que F, |F, (Teotema 17.7)e, por a tanto o a méc(F 4 FL) nl oTa 1 Mas, sendo r_ 0 iiltimo resto diferente de zero no algorit mo de BUCLIDES, 0 mde(m,n) = r,, 0 que implica: nde (Fy» Fy) = Fade (m,n) e o teorema fica demonstrado. Assim, p.ex.t nde (F, 65 Fy) = Fuge (16,12) * Realmente: ndc (F, Fh) = mdc(987, 144) = 3 16? Teorema 17.9 Se F |F , entao mln. SOE ay Demonstragao: Com efeito, se FijF., entaoo mdc(F,, Fy) = 292 lo teorema 17.8, 0 mdc(F_, F mS yo- y+ Assim~ serfdo, Fade (m,n) o mdc(m,n) =m e, portanto, Note-se que este teorema 0 reciproco do teorema 17.7, de modo que na sequencia de FIBONACCI, FIP se e somente se njn. EXERCICIOS 1. Determinar os nidmeros de FIBONACCI: Fuss Fig © Fao, 2, Calcular a soma dos 15 ptimeiros mimeros de FIBONACCI. 3. Calcular a soma dos 12 primeiros “mnimmeros de FIBONACCI * com indices impares. 4. Calcular a‘soma dos 10 primeiros niimeros de FIBONACCI com indices pares. 5. Calcular a soma dos 17 primeiros nimeroe de FIBONACCI tomados alternadamente com os sinais +e ~. 6. Calcular a soma dos quadrados dos 14 primeiros nimeros de FIBONACCI. 10. 11. 12% 13. 14, 293 Caleular o mde(F,76, F177) © 0 méc(Fyo¢5 Fes.) Caleular o mdc(F)5, Fyq) eo méc(Fy,, F¢). eF,. Achar os nimeros de FIBONACCI que dividem Poy 36 Foi conjeeturado que ha somente cinco nimeros de FIBO- NACCI que também sao nigmeros triangulares. Achar esses cinco niimeros de RIBONACCI. Representar os inteiros 50, 75 e 100 como somas de dis tintos mumeros de FIBONACCI. Verificar que 5(F,)> + 4(-1)" @ sempre um quadrado perfeito para n= 1,2,...,10. Demonstrar: = 2 2 (a) Se 2\r, entao 41 ay Fev x 3 3 (b) Se 3|F,, entao Na re) Demonstrar: (a) Fi, =F, (mod.2), isto a, Fy, Fes Foo -++ sa0 to- dos inteiros pares; (b) Fis = SF, (mod.5), isto 8, Fy, Fiy, Fys,--- so todos inteiros divisiveis por 5. 294 15.Levando em conta que Ful, se e somente se -m|n,-mos- 16. 17. 18. 19. ‘trar: (a) 2|F.. se ° somente se 3|n; (b) 3|F, se e somente se 4|n; (c) 4|F_ see somente se 6|n; (4) 5|F, see somente se 5|n. Demonstrar que, se o mdc(m,n) = 1, entao FAP alam (mn>1). Usando o "Teorema da indugao matemaitica", demonstrar: FL + 2F) + 3F, +--+ oF, = (n+ DPA) - Fy, * 2 Demonstrar: - 2 GY Fo, = Ft Yur () Fy yy) - gy) Wn e2 2 2 a ) Fy * Fy) 7 yy) + CD", Wn22 Demonstrar: cee = N @ Gy) 4F Fy 7 Fay) > n>3 a ©) FogaFoea 7 FoeaFana 7 20D", W083 ie Mey \ CGD a Jor, Suet) Yne3 2 ea eth \ (4) (F) FiyaFag CD, Yn>4 4 (0) FuFnerFaea neg 7 Fy) 7 1s View 295 20. Demonstrar: 2 n (mod.5), para todo inteiro n>L nly = 2 21. Usando o "Teorema da indugao matematica" demonstrar a forma de BINET: FL = (a = b/s i ; e se 2 onde ae b sao as raizes da equacdo quadratica x = =x +1, isto é: a= (1+ /5)/2 e b=(1 -/5)/2. 22. Demonstrar que, se r @ 0 resto da divisao de FL por F, (n>m), entdo r ouF -r um nimero de FIBO- WACCI, Exemplificar ambos os casos. 23. Demonstrar a identidade: > Fa) + (QF ye, “Wnt ae at ne? ~ Fones 24, Demonstrar a identidade: » VWarl a. a a — CDF t GF, + GP3 te2* QOFA & Fag caPituLo 18 TERNOS PITAGORICOS 13.1 CONCEITO DE TERNO PITAGORICO Definicao 18.1 Chama-se terno pitagérico todo terno de in teiros positivos (a,b,c) tais que Em outros termos, terno pitagorico @ toda solugao inteira e positiva da eqyagao diofantina: 2 etek Assim, p.ex., sao ternos pitagdricos: (3,4,5), (6,8,10), (5,12,13), (12,35,37) pois, temos: 297 Se (a,b,c) @ um terno pitagérico, entao (ka,kb,ke), onde k>1 @ um inteiro positive qualquer, tambem @ um terno pi- tagérico, pois, temos: (kay? + (eb)? = 2a? + eo? = e2(a2 + 02) = =e? = (ke)? 18.2 FORMULAS QUE DAO TERNOS PITAGORICOS As formulas: andk+l, bam 4 mw, ca + ee onde k € um inteiro positivo qualquer, atribuidas a PITAGO RAS, dao uma infinidade de ternos pitagéricos, pois, te- mos: ae bt = (ak + 1)? + (22 + 2K)? = 3 = ak + 8x + 8k2 + Bk + 1 = = (22 + 2k +1)? # 2 Assim, p.ex., para k = 7, temos: a=2.7+12=15, b e=2.77 42.7412 113 de modo que (15,112,113) @ um terno pitagorico. 298 Analogamente, as formulas: a-=2pq, b= onde p e q (p>q) sao dois inteiros positivos quaisquer, atribuidas a PLATAO, tambem dao uma infinidade de ternos pitagoricos, pois, temos: ne a? +b? = (pq)? + (p? - gs phe apg? + gt = = te yt 2 Assim, p.ex., para p= 5 e q= 3, temos: a=2.5.3=30, b=5'-3? = 16 ce Ste gee 3g de modo que (30,16,34) @ um téermo pitagérico. As formulas: asf@-, Ben, c= FG +D fornecem ternos pitagdricos mediante a substituicao de n por um inteiro positivo impar maior do que l. Assim, p.ex., fazendo n= 7, temos: a=FP-1) =m, bH7, cx br +1) #25 de modo que (24,7,25) @ um terno pitagdrico. 299 £ imediato que a todo terno pitagorico (a,b,c) esta asso- ciado um trtangulo retdngulo cujas medidas respectivas dos catetos e da hipotenusa sao a, bec, denominado trian— gulo pitagérico. 18.3 TERNOS PITAGORICOS PRIMITIVOS Definicao 18.2 Chama-se terno pitagdrico primitivo todo terno pitagdrico (a,b,c) tal que o mdc(a,b) = 1. Em outros termos, terno pitagdrico primitivo é todo terno pitagdrico (a,b,c) em que os inteiros positives ae b sao primos entre st. Assim, p.ex., sao ternos pitagéricos primitivos: (5,12,13), (8,15,17), (9,40,41) pois, sao ternos pitagdricos nos quais o mdc(5,12) = mde(8,15) = mdc(9,40) = 1. Note~se que, se (a,b,c) @ um termo pitagdrico primitivo, isto é: 2 2 2 eo mdc(a,b) = 1 entao, se tem: mdc(a,c) = mde(b,c) = mdc(a,b,c) = 1 300 Porque todo divisor d de dois quaisquer dos trés. inteiros positivos a, b, ¢ divide também o terceiro. Se (a,b,c) @ um terno pitagorico nao primitive, isto é, tal que o mdc(a,b) =d #1, entao dlc, e os quocientes: ay ald, by = b/d, c) = e/a formam 0 terno pitagérico primitivo (a,,b,,¢,), pois, te- mos: 2 2 2 2 pte 24 2a Mtl 2, 2 ale middle oigedbygzone eae hielay eo mdc (a, ,b,) =l. Portanto, qualquer terno pitagérico ndo primitive se pode obter de um terno pitagdrico primitive multiplicando-se os seus elementos por um conventente inteiro positivo maior do que 1, isto @, todas as solucdes de x s =a? re sultan daquelas de xj + yz = 24, onde o mle(x,,y,) = 1. Assim, p.ex., os ternos pitagoricos nao primitivos: (15,36,39) e (25,60,65) resultam ambos do terno pitagérico primitivo (5,12,13) ml tiplicando-se os elementos deste respectivamente pelos in- teiros 3e 5. 301 NOTA, Com todos os elementos menores que 1000 sao conheci © dos 158 ternos pitagoricos primitivos, e aquele que tem os maiores elementos @ (372,925,997). 18.4 PROPRIEDADES DOS TERNOS PITAGORICOS Teorema 18.1 Para todo inteiro positivo a>2 existem in- teiros positivos b ec tais que (a,b,c) & um terno pitagé- rico. Demonstragac Suponhamos, primeiro, que o inteiro a>2 @ par. Entao, 2la e 4la%, de modo que b= (a= 4/6 @ c= (a + A/S sao dois inteiros positivos. E como = 8a" + 16 _ 16 segue-se que (a,b,c) @ um terno pitagorico. Suponhamos, agora, que o inteiro a>2 @ {mpar. Entao, a= 2k+1e as formlas de PITAGORAS: 302 an2k+1, b= 2k? + 2k; c= 2+ 2 E-—-7 onde k @ um inteiro positivo qualquer, dao o terno pitag rico (a,b,c). Teorema 18.2 Um terno (a,b,c) @ um terno pitagérico se e somente se existem inteiros u ¢ v que verificam as seguin- tes condigoes: Gi) urv>0 (ii) =u = v (mod .2) (iit) uv @ um quadrado perfeito (iv) asVYuv, b= (u-v)/2, c= (ut v)/2 Demonstrag ¢ que verificam as quatro condicoes enumeradas no enunciado >) .Suponhamos, primeiro, que ue v sao dois inteiros do teorema. Entao, por (i) ¢ (iv), a, be ¢ sao positives, por (iii) e (iv), a @ um intetro, e por (ii) e (iv), bec também sao intetros (a condicado (ii) @ um modo abreviado de dizer que os inteiros u 6 v sao ambos pares ou ambos impares). Finalmente, pela (iv), temos: w= 2uv + v? ata bt = ut - e, portanto, (a,b,c) @ um térno pitagdrico. 303 (<—— ) Reciprocamente, suponhamos, agora, que (a,b,c) & um terno pitagdrico, Cumpre achar inteiros u e v que ve~ rifiquem as quatro condigces enumeradas no enunciado do teorema. Seja u=c+b evec-b Como b e ¢ sao inteiros positivos e c>b, temos u>v>0, de modo que (i) é satisfeita. Por ser u-v = 2b, segue-se que 2|(u - v), isto é: u =v (mod.2) e a condica0 (ii) @ satisfeita. A condigdo (iii) @ satisfeita, pois, temos: uv = (c + b)(c - b) = co = Finalmente, a condicgao (iv) também é satisfeita, pois, te- mos: Yuva, (u-v)/2 = 2b/2=b, (u + v)/2 = 2c/2 0 ° Exemplo 18.1 Mostrar que (360, 319, 481) @ um terno pita gorico. Os inteiros: u = 481 + 319 = 800, v = 481 - 319 = 162 304 satisfazem as quatro condicgoes enumeradas no enunciado do teorema 18.2: (4) u>v>0, porque 800>162>0 (ii) uv (mod.2), porque u-v = 638 e 2/638 (444) uv = 129600 = 360%, quadrado perfeito (iv) B= 360, (u - v)/2 = 638/2 = 319 (u + v)/2 = 962/2 = 481 Logo, (360, 319, 481) @ um terno pitagérico. Teorema 18.3 Se (a,b,c) & um terno pitagérico primitivo, entéo c é imar e af b (mod.2). Demonstrag: Se a= (mod.2), entao 4 e b sdo ambos pares ou sao am bos tmpares. Se a e b sao ambos pares, entao 2|mdc(a,b), o que @ impossivel, porque o mdc(a,b) = 1. E se, ao in- 2 eS pee e, portanto, c @ par, isto @, se a=2h+le b = 2k+l, entao c = 2n, e temos: Zz & az x aug vés, a e » sao ambos impares, entao c =a a2 = Gh? + Gh +1 = 1 (mod.4) po = 4k° + 4k + 1 = 1 (mod.4) e2 = in? = 0 (mod.4) © que implica: 305 Pg +b 22 (mod.4) ou seja: c =a +b = 2 (mod.4) e, portanto, 2 = 0 (mod.4), o que @ absurdo. Assim sen- do, sO pode ser a 2 b (mod.2), de modo que ae b sdo de paridade diferente, e como a’ +b’ = c%, segue-se que c” @ impar, o que implica c também impar. NOTA. De conformidade com este teorema, num terno pitagért co primitivo qualquer (a,b,c), ha exatamente wn elemento que @ par (a ou b) e dots elementos que sao imares (a e cou bec), de modo que a soma at+b+c @ sempre um inteiro positivo par. EXERCICIOS 1. Achar os ternos pttagéricos dados pelas formulas de PITAGORAS para k = 3,5,8. 2. Achar os ternos pitagéricos dados pelas formas de PLATAO para ‘os seguintes pares de valores de p e @! (a) p=6eq=4; (b) p=9eq7 306 10, ll. 12. 0 termo pitagérico (19,b,c) € dado pelas fémitilas de PITAGORAS. Determinar b e c. 0 terno pitagérico (20,b,c) @ dado pelas férmulas de PLATAO. Determinar be c. Construir trés ternos pttagdricos (a,b,c) tais que os elementos b e ¢ sejam inteiros consecutivos. Achar trés ternos pttagdricos distintos da forma (16,b,c). Achar todos os ternos pttagoricos da forma (a,30,c). Achar todos os ternos pttagdricos primitivos da forma (40,b,c). Achar todos os ternos pitagéricos primttivos da forma (60,b,c). Mostrar que (3,4,5) @ 0 imtco terno pitagérico primt: vo formado por inteiros positivos consecutivos. Demonstrar que num terno pitagérico primitivo (a,b,c) © produto ab é divisivel por 12 e o produto abe é di- visivel por 60. Demonstrar que, se n # (mod.4), ento existe um terno pitagérico primttivo (x,y,z) no qual x=n ou y=n. Mostrar que (3n,4n,5n),onde n=1,2,3,...,340 0s iimicos 14. 15. 16. 17. 18. 19. 307” ternos pitagéricos cujos elementos estao em progres- sao arttmética. Demonstrar que,se (a,b,c) @ um terno pitagorico primi tivo, entao a ou b @ divisivel por 3. Mostrar que, se a € um inteiro positivo impar, entdo existe um terno pitagorico (a,b,c) tal que c=b+l. Demonstrar que num terno pitagérico (a,b,c): (a) Um dos elementos a, b ou c @ divisivel por 5. (b) a, by atb ou arb @ divisivel por 7. Seja (a,b,c) um terno pitagorico primttivo no qual b & impar. Mostrar que 4{a. Seja (a,b,c) um terno pttagérico, Mostrar: Mostrar que o numero de ternos pitagéricos (x,y+2) cujo primeiro elemento x = a, sendo a um inteiro po~ sitivo dado, é igual a 2 = te (@@2) - /2 se a & dmpar (a(a2/4) -1)/2 se aé par capituto 19 CLASSES RESIDUAIS 19,1 CONCEITO DE CLASSE RESIDUAL Definicao 19.1 Chama-se classe residual médulo m de um in teiro a o conjunto de todos os inteiros que sao congruen- tes a a mddulo m, Este conjunto representa-se por a+ Portanto, simbolica- mente: a, ={ xez | x = a (mod.m)} = ={ xéZ | mj(x - a) } ou seja: {x|x=sat+kn, kez } ™ Em particular, se o médulo m = 1, entao: a, =(xéZ | 1]@e-a)}=2 309 isto @, a classe residual modulo 1 de um inteiro qualquer a @ o conjunto 2 dos inteiros. As classes residuats médulo m também sao denominadas in- tetros médulo m. : Note-se que a= a (mod.m), porque m/0, de modo que a€a,, isto é, a classe residual modulo m de qualquer in- tetro a nunca é vazia: a, # > para todo a€Z, Exemplo 19.1 A classe residual médulo 3 do intetro 2 0 conjunto: {3]x=2+4 3k, Kez}e ={..., -7,-4,-1,2,5,8,11,...} Observe-se que 245 (-7)4 =. 11, = 353 =... isto @, os simbolos distintos C7)5, a5 ll, 3545 see der signam todos um mesmo conjunto - @ classe residual médulo 3 do inteiro 2. Exemplo 19.2 A classe residual modulo 5 do intetro -4 & © conjunto: (A)o atx | x= -4+ 5k, kez} = = {...,719,-14,-9,-4,1,6,11, ...} 310 Observe-se que 4a 4), = 19.2 PROPRIEDADES DAS CLASSES RESIDUALS Sejam a, e b, as classes vesiduats mddulo m de dois intei- ros quaisquer a e b. Teorema 19,1 As classes residuais a, © b, sao tguate se e somente se a = b (mod.m). Demonstragao: (= ) Suponhamos que a = b (mod.m), e seja x um elemen to qualquer de a (x€a,). fatad: * = a (mod.m) e a = b (mod.m) => x = b (mod.m) @ isto significa que x€b,. Logo: a.cb, (1). Seja, agora. y um elemento qualquer de b,(y€b,). Entao: y = b (mod.m) e a= b (mod.m) => y = a (mod.m) e isto significa que yo a+ Logo: bea, (2). Das inclusdes (1) e (2), resulta: a, = b,. 311 (< a€a,, segue-se que ab, e, portanto: a = b (mod.m). ), Reciprocamente, suponhamos que a, = . Como Teorema 19.2 Se as classes residuais a, e b, ndo sido dis juntas (a,b, #9), entao coineidem: a, = b,- Demonstragao: Com efeito, se a eb, nao sao disjuntas, entao existe ao menos um inteiro ¢ tal que cé a, @ ce€b,, isto é, tal que c =a (mod.m) e c = b (mod.m) => a = b (mod.m) Logo, pelo teorema 19.1: a,=b. im On ‘ Teorema 19.3 Se as classes residuais a,e ba sao distin— tas (a, # b,), entdo sao disjuttas: a fb, = ¢. Demonstragao Com efeito, se aye by nao sao disjuntas, entao existe um inteiro c tal que c€a, e céb,, isto é, tal que ¢ =a (mod.m) e c= b (mod.m) => a = bd (mod.m) Logo, pelo teorema 19.1: a, = b,, 0 que & impossivel, vis to que, por hipdtese, a, # be 312 NOTA, A classe residual a, diz-se determinada ow-defthida pelo inteiro a, o qual, por sua vez, chama~se un represen tante de a. Dois inteiros sao representantes de uma mesma classe rest dual mbdulo m se e somente se sao congruentes médulo m. 19.3 CONJUNTO DAS CLASSES RESIDUATS 0 conjunto de todas as classes residuais modulo m indica se por ay isto @, simbolicamente: z, ={ay Jaez} Em particular, se o moduld m = 1, entao a, = 2 para todo a€Z, de modo que 2, = {Z}. Note-se que, na Teoria dos Conjuntos, 2, outra coisa nao @ que o conjunto-quociente de Z pela congrudncia modulo m Teorema 1 0 conjunto 2, encerra m classes residuais modulo m distintas. Demonstragao: (i) "As classes residuais modulo m dos inteiros xX que sa tisfazem 4 condigao x€[0,m-1] sao todas distin~ 313 tas, porque, se a e b sao dois inteiros tais que a,b€[0,m-1], entao: ja-bl a2 b (odin) > a, ¥b, (ii) Por outra parte, dado um inteiro qualquer ye[0o, wi], temos, pelo algoritmo da divisao: yrrsmq, com O¢r y= 41, com r€[0, m1] Como existem apenas m inteiros x¢[0,m-1], que sao: 0,1,2,...,m-1, segue-se que o conjunto Z, en~ cerra m classes residuais modulo m distintas: 2, =40,. Lys Que oes G1), } Observe-se que Zz, @ um conjunto finito, embora Z nao o se- ja. NOTA. Para achar a classe residual mddulo m de um inteiro qualquer y basta determinar o resto r da divisao de y por m, pois, temos Yq = Fr COD O 75g = ={x|x=3+8k, kez}. = «,713,75,3,11,19,. Exemplo 19.3 Se o modulo m = 2, temos duas classes resi- duais nédulo 2 distintas: {x|]x=0+2k, kez} = = (24+ 576,74,-2,0,254,6, oo} = (-4)y = 49 = 8p tee 1, s{x|x=1+ 2k, kEZ}= = (.40575,7357151,3,5 46 = 3p 25.27) ee A classe residual 0, @ formada por todos os inteiros pares ea classe residual 1, @ formada por todos os inteiros im- pares. Portanto: 315 Note-se que 0,1) = @ 0,U1, = 2. Exemplo 19.4 Se o médulo m = 5, temos cinco classes resi duais modulo 5 distintas: Os six|x= 04 5k, kEz}= = 1... ,°15,-10,-5,0,5,10,15, ... = = (10), = 5, = 1555... 1, ={x|x=1+ 5k, kezbe ={...,°14,-9,-4,1,6,11,16, ... }= Msg 7a65.7 Zl gpa 25 ={x|[x=2+ 5k, kEz}= =(...,-13,-8,-3,2,7,12,17,... }= * (3), = 25 = 17, a, 3, ={x|x=3+ Sk, kEZ}= =4...,712,-7,-2,3,8,13,18,...} = = (Wg 2 8, = 2358... 316 4, atxlx=4+ 5k, kez}= +y711,-6,-1,4,9,14,19, = FCs #452 195%. Portanto: ={0,, 15, 25, 355 4g} Esta representacao de Z, no & imica, pois, temos: Bs = (555 (5s 755 (-2)55 95} Note-se:- MMISNZN3N4s= 4 © OY IGU2U3U 4, = Ze NOTA, Em cada um destes dois exémplos observe-se que as classes residuais obtidas possuem as tres seguintes pro- priedades: (1) no. sdo vazta’;(2) edo disjuntas duas a duas; (3) sua renido 0 conjunté Z dos inteiros. Expri- me-se este fato dizendo que 6s conjuntos 2, 25 sao par tigdes do conjunto Z dos inteiros. De modo geral, as m classes residuais 0 lor Sou s++s (rl), que formam 0 conjunto Z, possuem estas mes- mas trés propriedades, de modo que o conjunto 2, 3 uma partigdo do conjunto Z dos inteiros. Importa ainda notar que um conjunto de m representantes, 317 um de cada.uma das classes residuais 0,142 ,...,(a-l)_, 2 wn m @ um sistema completo de restos médulo m, Assim, p.ex., 0 conjunto: {-5,11,-8,13,-6 } @ um sistema completo de restos modulo 5. EXERCICIOS 1. Achar a classe residual médulo 8 de -5. 2. Achar a classe residual modulo 9 de 1913. 3. 0 inteiro 17 pertence 4 classe residual médulo m de 24, Determinar m. 4, 0s inteiros 29 e 41 pertencem a uma mesma classe resi dual médulo m. Determinar m. 5. Mostrar: 10, ly, 841g = Big (-8), = 205. caPituto 20 REPRESENTACAO DOS INTEIROS EM OUTRAS BASES, CRITERIOS DE DIVISIBILIDADE 20.1 REPRESENTAGAO DOS INTEIROS EM OUTRAS BASES Teorema 20.1 Dado um inteiro qualquer b>2, todo intei- ro positivo n admite uma imiea representagio da forma: sagt m1 2 n= ab +a. > Feeet apd tayb+a, onde os a; sao tais que O¢a:q, >a, >qz>-++ e cada q,20, esta sequéncia decrescente dos quocientes qy @ fintta, isto é, existe um indice m tal que ae oe ba, +a Osa = 1, 5 + (bq, + ba te + 2 (bq, + B’ay +... +d G1) * +altapbtab + ree tay, pelea ou, finalmente: ety m-1 2 neat a bh +..4 ab? + ab +a) Assim, dado um inteiro qualquer b >2, todo inteiro posi- tivo n pode ser representado por um polindmio inteiro em b do grau m (porque a, # 0), ordenado segundo as potén- ctas decrescentes de b, e cujos coeficientes a, sio in- teiros que satisfazem as condicées: Osa; ) +...4 aj.10 + a, a representagao no sistema decimal (base 10) do inteiro po sitivo n, onde cada a, @ um inteiro tal que O¢a,<10, ¢ seja S a soma dos seus algarismos: = + Seajta ts. tata, Entao, o inteiro positivo n @ divisivel por 9 (9|n) se e somente se S @ divisivel por 9 (9]S). Demonstragao Consideremos o polindmio em x com coeficientes inteiros: P(x) 326 Por ser 10 = 1 (mod.9), temos (lema 20.1): Pore P(10) = P(1) (mod.9) Como P(10) =n e P(1) = S, segue-se que nis (mod.9). Assim sendo n=0 (mod.9) se e somente se § = 0 (mod.9), isto €, 9|n se e somente se 9/5. Assim, p.ex., 0 inteiro n = 26356734 @ divistvel por 9, Porque a soma dos seus algarismos é S#2+6+34+5+64+74+34+42 36 e 36 @ divisivel por 9. Realmente: 26356734 = 9 x 2928526, Exemplo 20.4 Determinar os algarismos x e y do inteiro n= 75x4y de modo que 2B seja divisivel por 5 e por 9, Belo critério de divisibilidade por 5 sd pode ser y =0 ou y = 5, e pelo critério de divisibilidade por 9 deve ser: T+54x4+4+y 20 (mod.9) ou x + y 2-16 = 2 (mod.9) Como x e y sao ambos menores que 10, sd pode ser: x+y=2 on x+y=1l 327 Se y= 0, entao x = 2 ou x = 11 (impossivel), o que da 0 inteiro 75240, e se y = 5, entao x = -3 (impossivel)ou x = 6, 0 que dd o inteiro 75645, isto @, o problema admi- te duas solucdes. 20.4 CRITERIO DE DIVISIBILIDADE POR 11 Teorema 20.3 Seja n n= nm ay.10" + a) ).10N" 4...4 ay 10 + a) a representagao no sistema decimal (base 10) do inteiro Po sitivo n, onde cada a, @ um inteiro tal que Osa <10, e seja n Tina, 7 2) * dp Saye © GL) Entao, o inteiro positivo n @ divisivel por 11 (11|n) se e somente se T @ divisivel por 11 (11}T). Demonstracat Consideremos o polinémio em x com coeficientes inteiros: m P(x) = £ a, = k=0 k Por ser 10 = -l (mod.11), temos (Lema 20.1): 328 P(1O) = P(-1) (mod.11) a Como P(10) =n e P(-1) = T, segue~se que n = T (mod.11). Portanto, n= 0 (mod.11) se e somente se T= 0 (mod. 11), isto @, 1l]n see somente se 11|T. Assim, p.ex., 0 inteiro n = 1571724 @ divisdyel por li, porque a soma alternada: Te4-2+7-147-S5+1e]n é divisivel por 11. Note-se que este inteiro também @ divisivel por 9, porque a soma dos seus algarismos @ 27 e 27 @ divisfvel por 9. EXERC ictos 1. Escrever (31322), e (5al9))) no sistema de base 10. 2, Escrever no sistema de base 10: GL4D., (3141), € G14) 4). 3. Escrever 132 e 285 no sistema de base 8. 4, Escrever 1492 nos sistemas de base es3yi75 Ie We 10, ll. 12. B.. 329 Escrever (7645), no sistema de base 5 e (a3b) 15 no sistema de base 7. Resolver as seguintes equacoes: (123), = Xs, (234) 5 = Xs (123) (2002) , Escrever 2417 no sistema de base 5 e 1639 no sistema de base 7. Determinar a base do sistema no qual 73 se escreve 243. Determinar a base do sistema no qual (346), se escre ve 91, Escrever 3ab7) 9 no sistema de base 10. Construir as tébuas de adigao e de multiplicagao no sistema de base2. ; Usando 0 critério de divisibilidade por 9, determinar se os inteiros 374518 e 1520397 sao divisiveis por 9. Usando 0 eritério de divisibilidade por 11, determinar se os inteiros 370546 e 22481907 sao divisiveis por ll. Usando os critérios de divistbilidade por 9 e por 11, determinar se os inteiros 176521221 e 149235678 sao divisiveis por 9 ov 11. 330 15. 16. 7. 18, Pemonstrar que um inteiro @ divisivel por 3 sé <~ so mente se a soma dos seus algarismos @ divisive] por 3, Pemonstrar que um inteiro & divistvel por 4 se @ so- mente seo numero formado pelo seu algarismo das deze~ Has @ pelo seu algarismo das unidades @ divisivel por 4 Determinar o algarismo X de modo que o inteiro: (a) 5x9 seja divisivel por 9 (b) 45x3x seja divisivel por 3 (c) 7x6 seja divisivel por 3 e por 4 (4) 934x1x seja divisivel por 2 e por 3 (e) 50x2x seja divisivel por 2, por 3 e por 11 Demonstrar que um inteird @ divisivel por 8 se e somen te se o niimero formado pelos seus trés dltimos algaris mos da direita @ divisive por 8. 19. Determinar os algarismos x e y de modo que o inteiro: - (a) 67xy seja divisivel por 5 e por 11 (b) Sx6y seja divisivel por 3 e por & (c) 56x2ly seja divisivel por 9 © por 10 (4) 3éxxS8y seja divisivel por 9 e por 11 (e) 231xy seja divisivel por 5 e por 9 (£) Sl4xy seja divisivel por 8 e por 9 331 20, Mostrar que a diferenca entre os inteiros abe e cba (a>c) @ um miltiplo de 11. 21. Mostrar que todo inteiro com trés algarismos, todos iguais, & divisivel por 37. 22, Escreve-se, 4 esquerda de um inteiro com dois algaris mos, 0 dobro desse inteiro. Mostrar que o novo intei- xo que assim se obtém @ divisivel por 67. 23. Demonstrar que um inteiro € divisivel por 4 se e so- mente se a soma do algarismo das umidades como do- bro do algarismo das dezenas @ divisivel por 4. 24, Determinar os algarismos x e y de modo que o inteiro: (a) x0y seja divisivel por 12 (b) 5x2y seja divisivel por 15 (c) 28x75y seja divisivel por 33 (d) 1234xy seja divisivel por 72 (e) 3262xy seja divisivel por 36 (£) 7x36y5 seja divisivel por 1375 25. Mostrar que todo inteiro formado de seis algarismos, todos iguais, @ divisivel por 7, 11, 13 e 37. 26. Mostrar que a soma dos seis inteiros de dois algaris- mos que se podem formar com trés algarismos diferentes @ igual a 22 vezes a soma desses algarismos. 332 27, 28, 29. 30. 3L. Demonstrar que um inteiro é divieivel por 8’se-e~ ‘so~ mente se a soma do algarismo das unidades, mais 0 do- bro do algarismo das dezenas, mais o quadruplo do alga rismo- das centenas é divisivel por 8. 5 Demonstrar que um inteiro @ divistvel por 6 se e so- mente se a soma do algarismo das unidades como qué- druplo da soma de todos os outros algarismos @ divisi- vel por 6. Demonstrar que, para todo inteiro a, o algarismo das unidades de at @0, 1, 5 ou 6. Demonstrar que o algarismo das unidades de um nigmero triangular @ 0,1,3,5,6 ou 8 0 inteiro xy243z € divisivel por 396. Determinar os al garismos x, ye z do dividendo. CAPITULO 21 RAIZES PRIMITIVAS 21.1 ORDEM DE UM INTEIRO MODULO N Sejam ae n>1 inteiros tais que o mdc(ayn) = 1. elo teorema de EULER: a) = (uodsn) aba) mas podem haver poténcias de a menores que e que sao congruentes a 1 modulo n. Assim, p.ex.: 3 33.56 30013) 3° = 1 (mod.13), 3°21 (mod.13) e Definicao 21.1 Sejam ae n>1 inteiros tais que o mdc(a,n) = 1. Chama~se ordem de a mddulo n o menor intei- ro positive k tal que = 1 (mod.n) 334 Indica~se que a ordem de @ médulo n é k com a nétagao: ” o,(a) =k. Assim, p.ex., com a=2en-= 7, temos: 2) 2 (mod.7), 27 = & (mod.7), 23 1 (mod.7) 2 (mod.7), 2° = 4 (mod.7), 2 = 1 (mod.7), ... Portanto, o inteiro 2 tem ordem 3 modulo 7, isto é: 0, (2) = 3. NOTA. Nao existe inteiro positive k tal que = 1 (mod.n) seo mde(a,n)>1 Porque, se existisse, entad x = a) seria uma solugdo da congruencia linear ax (Teorema 10.1). 1 (mod.n), 0 que & imposstvel Cumpre ainda observar que, se dois inteiros a e b sio con- gtuentes modulo n, entéo a e b tem a mesma ordem modulo n, porque k a =b (mod.n) e a 1 (mod.n) implica: = b’ = 1 (mod.n) Exemplo 21.1 Achar a8 ordens médulo 11 dos inteiros posi- tivos menores que 11. 335 a vig wy a” elie ae re) 10 1 L 1 1 1 1 < 1 1 A. 2 4 8 5 10 9 7 3 6 1 3 9 5 4 1 3 a = 4 1 4 5 9 z 1 4 4 9 3 i 5 3 4 9 1 2 3 4 S 1 6 3 7 9 10 5 8 4 2 1 ? 3 2 3 10 4 6 9 8 i 8 9 6 4 10 3 2 5 7 1 9 4 3 5 1 9 4 3 5 1 10 1 10 1 10 1 10 1 10 1 Consoante a tabela acima, temos: 0441) = Ly 04402) = 04166) = 045 (7) = 04,(8) = 10 94; (3) = 04, (4) = 04 (5) = 04,09) = 3 01; (10) a2. Observe-se que as ordens 1, 2,510 sa0 divisores de o(1L) = 10. Teorema 21.1 Seja a um inteiro que tem ordem k modulo ne Entao: a= 1 (mod. n) se e somente se k divide h (k|h). 336 Demonstrag: 3 (=>) Suponhamos que k\h, isto 8, h = kq, onde a éum inteiro. Por ser o,(a) = k, temos: 31 (modem) ——> (a®)% = ah 21 (mod.n) 7 he (<==) Reciprocamente, suponhamos que a" = 1 (mod.n), sendo h um inteiro positivo. Pelo algoritmo da divisao,te- mos: he=kq+r, com O¢rcek Portanto: he gke eS Glyagr Mas, por hipdtese: = 1 (modin) e a = 1 (mod.n), 0 que implica: a” = 1 (mods) 2. r A ‘ok x... Como k € 0 menor inteiro positivo tal que a* = 1 (nod.n) © O€ri inteiros tais que o mdc(a,n) = 1, e sejam Ays Ags vets Bycgy os inteiros positivos menores que ne que sao primos com n Se a & uma ratz primittva de n, entdo as poténcias: so. congruenter modulo n aos intéiros Arr S29 89 Soca)? nao necessariamente nesta ordem. Demonstrag: Como a e n sao primos entre si, 6 mesmo ocorre com toda po téncia de aen. Portanto, cada a* @ congruente médulo na algum dos a;. E como as $(n) poténcias de a: a, a7, ..., a9 gio incongruentes médulo n (Corolario 21.2), segue-se que estas $(n) poténcias de a sao con- gruentes midulo n aos O(n) inteiros a). +++4 ay¢qys ecc € 343 ndo necessariamente nesta ordem, Assim, p.ex., 2 € uma raiz primitive de 9, @ as poténcias de 2: 4 6 Deez ean Sao congruentes mdulo 9 aos inteiros menores que 9 e@ pré mos com 9: 2,4, 8, 7, 5,1 Corolario 21.4 Sen tem uma ratz primittva, entdo n tem forolario 21.4 sen a Precisamente $((n)) ratzes primitivas. Bemonstracao: Suponhamos que a é uma raiz primttiva de n. Pelo teorena anterior, qualquer outra raiz primitive de n se encontra entre os elementos do conjunto: 2, ao) Mas, o nimero de poténcias a, onde 11l tem uma naiz primitiva se e somen- te se n= 2, 4, p*, apf onde p é@ um primo impare k2l. EXERCICIOS 1. Achar as ordens dos inteiros 2, 3 e 5: (a) modulo 17; (b) modulo 19; (c) médulo 23. 2. Achar as ordens dos inteiros 2,4,7,8,11,13 e 14 (mod.15). 3. Achar os inteiros que tei ordem 6 (mod.31). 4, Demonstrar as seguintes proposicoes: (a) Se a tem ordem hk (mod.n), entdo a" tem ordem k (mod.n). 10. i. 12. 345 (b) Se a tem ordem 2k modulo o primo impar RB, entao a = 1 (med.p). (c) Se a tem ondem n-1 (nod.n), entdo n @ um primo. As ordens respectivas dé 2b mdulo n sao he k. Mos trar que a ordem de ab (mod.n) divide bk. Em particular, Seo mdc(h,k) = 1, ent&o a ordem de ab (mod.n) é bk. Mostrar que, se a tem ordem 3 (mod.p), ent@o atl tem ordem 6 (mod.p). Mostrar que 2 @ uma rats primtiva de 19, mas nao de 17. Mostrar que 15 nado tem ratzes primitivas caleulando as ondens dos inteiros 2,4,7,8,11,13 e 14 (mod.15), Achar duas ratzes primitivas de 10. Sabendo que 3 @ uma ratz primitiva de 17, achar todas as outras raizes primtivas de 17. Sabendo que 2 & uma raiz primitiva de 19, achar todas as outras raizes primitivas de 19. Achar todas as ratzes primitivas de 5, 9, lle 13. 346 EXERCICIOS SUPLEMENTARES 1, Demonstrar que, se ae b sao inteiros, com 6>0, en- tao existem e sao iinteos os inteiros gexrtais que a= bq +r, onde ber <3. 2. Verificar que, se um inteiro n @ a0 mesmo tempo um qua dado perfeito e um cubo perfeito (como 64 = 82 = 43), entao n & da forma Tk ou 7k+l. 3. Demonstrar que, se ae b sao inteiros, comb #0, en- tao extstem e sao iicos os inteiros Gexrtais que a=bq+r, onde =/b//21. 7. Demonstrar que, se a € um inteiro tal que 2}/a e 344, entdo 24 divide a2 - 1. cccc¢ SCRER ¢ ( 10. ll. 12, 13. 14. 15. 347 Demonstrar que a soma dos quadrados de dois inteiros impares nao pode ser um quadrado perfetto. Demonstrar que o produto de-trés inteiros consecutivos @ divisivel por 6 e que o produto de quatro inteiros consecutivos & divisivel por 24. Demonstrar que 360|a2(a? - 1)(a? - 4), qualquer que seja o inteiro a. Demonstrar que o produto de cinco inteiros consecuti- vos @ divisivel por 120. Sejam ae b inteiros ndo nulos. Demonstrar: (a) Se k>0, ent@o o mme(ka,kb) = k.mme(a,b). (>) Se m @ um miltiplo comum de ae b, entao o mne(a,b)|m. Sejam p e p+2 primos gémeos. Demonstrar que p(p+2)+1 & um quadrado perfeito e que pt(p+2), com p>3, @ dé visivel por 12. Mostrar que, se n>3, os inteiros n, nt2e nth nao podem ser todos primos. um Demonstrar que, se o conjunto {a sat 2a sistema completo de restos modulo n e se o mdc(a,n) = = 1, entao o conjunto {aa,,aa,,...,aa,} também @ um stetena completo de restos modulo n. 348 16. Verificar que o conjunto {0,1,2,27,23,...,27) a) um ststema completo de restos modulo 11, mas que oo con- junto {0,17, 22, 32, ..., 102} no o &. 17. Demonstrar as seguintes proposigées: (a) Seo mdc(a,n) = 1, entdo o conjunto de inteiros: fe, cha, ct2a, ..., c#(n-1)a} & um sistema completo de restos modulo n, qualquer que sejac. (b) Todo conjunto de n inteiros consecutivos & um sis- tena completo de restos midulo n. 18. Verificar que, se a=b (mod.m) ese a=b (mod.my), entao: a =b (mod.m); onde m= mame (m) ,m>) Em particular, se m, em, so primos entre si, entao a 22 (mod.m,my). 19. Demonstrar que, se ab = cd (mod.m) e se b= (mod. m), como mde(byn) = 1, entao a = c (mod.m). 20. Demonstrar que, se a= b (thod.m) ese atc (mod.m,), entao: e 2 ¢ (mod.m), onde m ='mde(m, jn») 9 21, Achar os dois iiltimos algarismos do inteiro 9 22. 23. 24, 25. 26. 27. 349 Usando os oritérios de divisibilidade por 9 ou por 11, determinar os algarismos x e y nas duas seguintes igualdades: (a) 2x99561 = [3(523 + oy] (b) 52817 x 3212146 = 169655y15282 Achar o menor inteiro a>2 tal que 2[a, 3| (atl), 4) (a2), 5|(a#3) © 6|(a+4). Determinar o menor inteiro positivo que deixa os res- tos: (a) 1,2,55 quando dividido por 2,3,6,12, respectivanen te; (b) 2,3,4,5 quando dividido por 3,4,5,6, respectivamen te; (c) 3,11,15 quando dividido por 10,13,17,respectivamen te; Mostrar que, se o mdc(a,42) = 1, entao 168 = 3.7.8 di vide a® - 1. Mostrar que, seo mdc(a,133) = mdc(b,133) = 1, entao 133|(al® - 418), Mostrar que 2° e a tém o mesmo algarismo das unidades, qualquer que seja o inteiro a. 350 28 29, 30. 31. 32. 33 34. 35. 36. + Demonstrar que a‘? = 13 = a (nod.3,7.13), qualquér“que’ se- ja o inteiro’a, Usando 0 teorema de FERMAT, achar o algarismo das uni- 100, dades de 3 Demonstrar que d(n) é um inteiro impar se e somente se n @ um quadrado perfetto. Demonstrar: El/d = s(n)/n ajn Verificar que s(n+2) = s(n)+2 para n = 434 ¢ n=8575, Demonstrar que, se n e n+2 sao primos gémeos, entao: s(n+2) = 8(n)42 Demonstrar que s(n) & uni inteiro impar se e somente se fn @ um quadrado perfeito ou 0 dobro de um quadrado per feito. Demonstrar que, se n>1 & um inteiro composto, entao s(n) >n+¥n, Sejam me n inteiros positivos. Demonstrar: (a) $(m)o(n) = $(mm)$(d)/d, onde d = mde(m,n) (©) $(m)4(n) = $(mde (m,n) )¢ (mmc (m,n) ) 351 37, Mostrar que a equagao $(n) = 14 nao tem solugao. 38. Usando o teorema de EULER, demonstrar: (a) a°? 2 a (mod.1729 = 7.13.19), qualquer que seja o inteiro a. (b) al? = a (mod.2730 = 2.3.5.7-13), qualquer que seja © inteiro a. (c) 2°? = a (mod.4080 = 15.16.17), qualquer que seja o inteiro impar a. 39, Sejam men inteiros positivos primos entre si. Demons trar: 2h 4 9) = 1 (mod) 40. Achar o ménor inteiro impar i>3 tal que 3(n, Si(m+2) e 7|(n+4). 41. Achar o algarismo das unidades de 78, 42, Achar o resto quando 314°? & dividido por 163. 43. Seja £(n) = (n + $(n))/2. Mostrar que £(£(n)) =6(n) se » onde k = 2,3,... 44. Achar todas as solugoes da equacao >(n) = 4. ee - Mostrar que pq (pq # 6) & deficiente. 352 46. 47. 48. 49, 50. 51. 52. 53. 34. 55. Mostrar que p®, k>1, € deficiente. a Achar os valores de k para os quais 2*.11 abundante. Mostrar que, se o mdc(a,b) = 1e se a @ abundante, en- tao ab @ abundante. Mostrar que, se 6|n, entao $(n)) n= p, sendo p um primo impar. (ce) n= 2, sendo- k>1. (4) n= pF, sendo k>1e p um primo impar. 68. 69, 70. Tle Hn. 73. 74, 75. 76. = = 2 Mostrar que, se atb @ par, entao 24Jab(a? - b*), Demonstrar que, se 210m +n @ primo, onde O¢n<121, entaoné primo, Mostrar que todo divisor proprto de um niimero perfeito par @ um numero deficiente. Mostrar que, se n = (6m+1)(12m + 1)(18m+1), entdo a-l 8 divisivel por 36m. Demonstrar que a soma 2p+2 de primos gémeos p e p+2, onde p>3, é divisivel por 12. Seja p um primo impar. Determinar o niimero de elemen- tos em cada uma das sequencias: 1222357354, 25.2, p(ptl) 2,2 Tidy 2635 Shy swear b: (PAL) que so prinos com p. Mostrar que a media harmonica dos divisores de um nime ro perfeito par @ um inteiro. Mostrar que 2n” - 3 nunca @ um quadrado perfettc, on- de n = 2,3,4,... Demonstrar por indugao matematica que E107 ait linen Os152 cee 356 77. Seja n um niimero perfeito par. Demonstrar qué ~- Td & uma poténcia de n. ala 78. Mostrar que a sequéncia 5,12,19,26,... nao contém ter mo algum da forma 2" ou 2" 1, 79. Mostrar que o inteiro 111...11 comn algarismos 164 composto se n @ composto. 80. Mostrar que, se a @ a soma de dois mimeroe triangula- ves, entdo 4n+1 @ uma soma de dois quadrados. 81. Mostrar que a° = a (nod.10), qualquer que sejao in teiro a. 82. Demonstrar que todo inteiro compreendido entre dois primos gémeos, exceto, 3;5 e 5,7, & abundante. 83. Sejam men nimeros amigos. Demonstrar: = * fl/d =1 84. Um inteiro diz-se sem quadrados se nao é divisivel pe- lo quadrado de qualquer inteiro maior que 1. Demons- trar: (a) um inteiro n>1 @ sem quadrados se e somente se n @ um produto de primos distintos, isto é: n= ppp, tintos; +Py» onde os p; sao primos todos dis- 85. 86. 87, 88. 89. 90. 357 (b) todo inteiro n>1l @o produto de um quadrado Perfeito por um sem quadrados, isto é: ‘a = a“b,on- de b @ sem quadrados. Demonstrar que o cubo de qualquer inteiro pode ser ex- Presso como a diferenca de dois quadrados. Seja mum inteiro qualquer e seja no inteiro que se obtém invertendo a ordem dos algarismos de m (p.ex.,se m= 7925, ento n= 5297), Mostrar que a diferenca m-n @ divisivel por 9. Mostrar que os inteiros: ALLL, 111111, 11111111, ..., 111...11, ... cada um dos quais @ formado por um niimero par de alga~ rismos 1, sao todos compostos. Calcular a soma: 1+ 11+ 111+... + 111...1, onde a Gltima parcela € formada de n algarismos 1. Demonstrar que o produto P = (n+1)(n#2)...2n de n in- teiros consecutivos é divisivel pelo produto P, = 1.3.5...(2n-1) dos n primeiros inteiros impares e achar 0 quociente. Um inteiro positivo dividido por 5 da resto 3 e dividi do por 9 da resto 4. Determinar o resto da divisao des se inteiro por 45=5 x9, 358 91. Calcular o-resto da divisdo por 8 de 43653 x 793787. 92. Mostrar que sao falsas as Proposicdes (a) e (b), dando um contra-exemplo para cada uma delas: (a) se 47/67, entio afb (b) seb] (a7#1), entdo b | (a1), 93. Demonstrar que, se o mde (a,b) = 1, entao o mde(a +b, ab) = 1, 94. Achar as solugdes inteiras e 2 positivas da equagao x - y? 2 499, 95. Determinar o inteiro i = 2*.3%.57, sabendo que dividi do por 12, por 18 e por 90 perde, respectivamente, 24, 27 e 30 dos seus diviscres. 96. Achar os inteiros de trés algarismos divisiveis ao mes mo tempor 14 e 34, $7. Sabendo que o mdc(n,54) = mde(n,126) = 6, calcular o inteiro n. 98. Sabendo que o mmc(35,n) = 180 e que o mme(84,n)=1260, calcular o inteiro n, 99. Determinar o niimero de inteiros compreendidos entre 100 e 1000 que sao divisiveis por 7. 100. 101. 102. 103, 104, 359 Sejam p e q primos distintos. Demonstrar: q-l , op po +g?) = 1 (mod.pq) Mostrar que os inteiros 1729 = 7.13.19 e 1905 = = 35.127 sao ambos pseudo-primos. Seja n um inteiro sem quadrados. Demonstrar que d(n) = a ‘ area aff 5 2°, onde r @ 0 nimero de divisores primos de Determinar quais dos seguintes conjuntos sao sistemas completos de restos modulo 11: (a) (0,1,2,4,8,16,32,64,128,256,512 } (b) €1,3,5,7,9,11,13,15,17,19,21 } (c) {2,4,6,8,10,12,14,16,18,20,22 } (d) {-5,-4,-3,-2,-1,0,1,2,3,4,5,} Determinar quais dos seguintes conjuntos sao sistemas reduztdos de restos médulo 18: (a) {1,5,25,125,625,3125 } (b) {5,11,17,23,29,35 } (c) {1,25,49,121,169,289 } (4) {1,5,7,11,13,17 } 105. Sabendo que k = 16 (mod.11), achar um inteiro x, com O1 @ impar, entao 3[(21), isto é, 21 & composto. 113. Demonstrar que para n>1: (a) GO1 @ primo se e somente se (n-2)! = 1 (mod.n). Mostrar que, se n é um inteiro composto, entao (n ~ 1)! = 0 (mod.n),-exceto quando n = 4, 362 122 123. 124, 125, » A sequéncia de inteiros Lys Lys Lge voey Lyigs Lge eee no caso em que Ly =i, Ly =3e Ly = Ly + L-2 (n 23) isto @, a sequéncia: 1, 3, 4, 7, 11, 18, 29, ... chama-se sequéncia de LUCAS. Demonstrar: (a) Ly, € pare Ly, € Le, _, sa0 impares, para todo n2l. @) LF ee Fon a-L (e) Ly + 2by + Mb, + Bby test ML = FL = 1 Determinar a mais elevada poténcia de 7 que divide o produto dos 1000 primeiros inteiros positivos. Demonstrar que 60 divide o produto (a°=1)n2(n2#1), qualquer que seja o inteiro positivo n. 0 inteiro n= 2*.3¥.5%, Calcular os expoentes x, y e 2 sabendo que o quociente n/2 tem 252 divisores e que os quocientes n/3 e n/5 tem, respectivamente, 45 divi sores a menos que n, ‘ 126. 127. 128, 129. 130. 131. Achar o menor inteiro positivo que tem 20 divisores positivos, sendo primos apenas 3, 5 e°7. Determinar o inteiro n = 2%.3¥, sabendo que o namero aul 2- -. = tot de divisores de n° @ 0 triplo do numero de divisores den. Determinar a mais elevada poténcia de 11 que divide o produto: P = 100 x 101 x 102 x ... x 1000 Os inteiros a e b sao formados: o primeiro de 2m al- garismos iguais a1, e o segundo de m algarismos iguais a 4. Demonstrar que a soma a+b+1 6 um inteiro quadrado perfeito. Verificar as igualdades: 10 221041 (a) 100001” = 10 (b) 999997 = 1019 - 2 x 19 +1 (c) 100001 x 99999 = 102° eae Demonstrar que o inteiro n = 111...12888...896, que tem n algarismos iguais a 1 e n-1 algarismos iguais a 8, @ um quadrado perfeito. 364 132. 133. 134. 135, 136. 137, Achar 0 menor inteiro positivo que admite: (a)-9 divi sores positivos; (b) 12*divisores positivos: (c) 15 divisores positivos; (a) 18 divisores positivos; (e) 20 divisores positivos; (£) 24 divisores positivos. Mostrar que um inteiro da forma 42"! mmoa é prino. Determinar o inteiro cujo produto de todos os seus di visores & igual a 3° y 540, Forma-se a sequéncia de inteiros: 49, 4489, 444889, 44448889, .. na qual cada termo resulta de intercalar 48 no contro do anterior. Mostrar que todos os termos desta sequén cia sao quadrados Perféctoe e determiner a raiz qua- drada do n-@simo termo. Determinar a base do sistema de numeracdo no qual 4 inteiro 16000 se escreve 1003000. sey 2 5 Achar um inteiro Atal que n° + 49 seja um quadrado perfeito. 138. 139. 140. 141. 142. 143. Li 4 145. 1 4 6. 365 Achar um inteiro de tres algarismos que seja quadrado perfeito e divisivel por 3 e por 5. Demonstrar que, se os inteiros a e b sao impares, en- PE = tao a + v mao pode ser um quadrado perfeito. Demonstrar que, se os inteiros a e b sao primos com 3, entdo a” + b* nao pode ser um quadrado perfeito. Demonstrar que z 1/j nao @ um inteiro para todo n>l. ai i a id Demonstrar que Z 1/(2j-1) nao @ um inteiro para to- do a>l. om Demonstrar que, se os inteiros ae b sao impares, en 2 a 2 « a ee tdo a“ +b° @ par, mas nao é divisivel por 4. Demonstrar que, se 0 mdc(n,7) = 1, entdo 7{ (n° - 1) e 7I(al? - 1), Achar o menor inteiro positivo n tal que 13|(n2 + 1). Caracterizar os inteiros positivos n para os quais se tem: (a) $(2n) = o(n)s (b) $(2n) > O(n) 366 147. Demonstrar que, se ne k sao inteiros Positivos, en+ tao 0 niimero de inteiros positives < nk que sio pri mos com n @ igual a ké(n). 148. Demonstrar que, se $(m) = $(mn) e n>1, entao n=2 em é impar, 149. Demonstrar que, se d|n, com 0d - $(d). 150. Achar 0 resto da divisdo de (1237158 + 34)?8 por 111, 151. Demonstrar que $(3n) = 26(n) se e somente se 3 nao divide n (34). 152. Mostrar que todo inteiro composto menor que 1000 tem um fator primo menor que 37. RESPOSTAS DOS EXERC{CIOS CaPETULO 1 i. 17. 18. 19. 20. 21. 22, 23. n(n + 1)/2 o=2 n=3 n= 4,5,7 Falsoz (3.2)! #312! @ (3 +2)! £3! 421 565 = 109 + 111 + 113 + 115 + 117 x=2, y= X55, y=3; 0 x23, ys4 . 12, 24, 36 e 48 107 ¢ 19" - 1 _x =F -xel ou x=2 +x =23,y=21; x= 13, y=9 (a) 3976 + 2934 + 5928 (b) 5623 + 4035 + 1269 = 11927 (a) 13256 - 4310 = 3946 (b) 6321 - 435 = 5886 234 9835 (a) 435. 36 (b) 2403. 62 110997 883889 368 24, 690 tm 25. (a) 73 x32 +.45; — (b) 257 x 6 + 59 27. 14 e 56 2 CAPITULO 3 1. (a) 363 (b) 30 (c) 68 (d) 746 2. (a) 336 (6) 125 (ce) 302400 (a) 315/222 4 k : 3. (a) eT abe yy (b) na i(itL) (i+2) 5 ) a 2553 4 fv (>) F (@v @F 7. (a) te= 2b e ts = 15 (©) ty = 6 ty = 10 e ts = 15 (ec) ts +15, te = 21 e t, = 28 9. 672 10,n=9 CAPITULO 4 3. Falso: 6] (57), mas 645 e ot 14. 18, 38, 60 e 84 15.a56, b=10 e e221 369 16. Falso: 6/18 e 9/18, mas 61% 19. b= 33, 34, 35 e r= 31, 19, 7 20. 249 € 2 166e 3; 8306 21, 322 e 19 22, 40, 80 e 120 CAPITULO 5 Ty Cail (>) 7 (e) 1 251,35 3. © = {1,5} 4. a=-13 oa=13 5. ¢ = mde(22, 55) = 11 6 1 7. (a) 2 (b) 1 8. 1,2, 5.e10 9% Low3 13. 1 ou 2 14. 10, 50, 70 ou 110 15. mdc(160 - 7, 198 ~ 11, 370 - 13) = 17 16. (a) a=9e b= 54; a= 18 eb = 45; a= 27 eb = 36 (b) a=6eb=126; a=18eb= 42 17. n = 48 ou n = 96 CAPITULO 6 1. (a) 9 (6) 17, (ce) 68 (4) 3 (e) 77 (£) 42 2. (a) 6 (b) 15 () 1 (a) 23 is 3. (a) x24, y= -3 (b) x= 6, y= -1 (c) x= 7, y= -3 (4) x = 39, y = -29 4. (a) x=7, y=-17 (b) x= 1, y =-1 (c) x = 56, y = -91 (4) x = 40, y = -64 (e) x= -3, y= 17 (£) nao existem (g) x = 3, y = -6 (h) x= 4, y = -14 5. (a) x= 28, y=-16, 22-1 (b) x= 17, y= -153, 2 =-1 (c) x= 18, y=-18, z= -3 (a) nao existem 6. x=171, y=-144, 2 = -2 7. (a) 315 (6) 5644 (ce) 1320 (d) 6106 =~ (e)-1344 (£) 16731 (g) 32461 (h) 22338 8. a= 184 eb = 72 9. (a) a= 12 eb = 336; a= 48 eb = 94 " (b) a= 8 eb = 560; a= 16 eb = 280 a=40eb= 112; a= 56e¢D = 80 (c) a= 57 e b= 532; a= 217 eb = 372 CAPTTULO 7 1. 2, 7, 23, 47 e 79 2, 81=34+5473; 125=5+13+ 107 3, 2¢5 i 3) 371 5.7 6. n=3 7. p25 8. (a) 169 € composto —(b) 197 primo (e) 239 & primo (4) 473 & composto 9. 5040 = 24.32.5,7 10, mde(a,b) = 2°.19,233 ame (a,b) = 230.3.521 74,14? 195,937 12, 419 e 421; 431 © 433; 461 e 463 13.522, 513 seers: GS ST Ws 14, 101! + 2, 101! +3, 101! + 4, ..., 101! + 101 15, (a) 32 #3429 413419 (b) 100 = 3 +97 =11+89+17+83=.., (c) 456 = 7 + 449 = 13 + 443 = 174439 =... (a4) 1024 = 3 + 1021 = 5 + 1019 = 11 + 1013 =... 17. n = 29 19, mdc(507, 1287) = 39; mmc(507, 1287) = 16731 - - 21. 930 i 22. Todos os primos ¢ 47. CAPITULO 8 Le (a) x= 204-9t, y=-15-7¢ (b) x = 18 + 23t, y = -3 -dt 372 (c) (4) (e) (£) (g) (h) (i) q@) (k) ql) 2. (a) (b) (c) (a) (e) (£) (g) “(hy 3. 118 x= — = Na 8 x= x ae Nao ia es xs Nao x« x= Nao. x= Nao x= “18+ 7t, y= 45 -17¢ 54 - 73t, y= 10 - Lat “3+ 7t, y = 35 = 48t tem solugoes inteiras ML + 30, y=-3- 4. 2+ 2, y= 3e Wt, y=-3- 13 tem solugoes inteiras 4t, ys-6 4 7t 10+ Sat, ys -3-a7t 8+lit, yole 5t, onde t>0 tem solugdes inteiras e positivas 8 + 3t, y=2+2t, onde t20 1, y= 66; x=12, yes tem solugdes inteiras e positivas 2, y= 383 x= 9, y= 20; x=16, y=2 tem solugoes inteiras e positivas 17 - Sit, y = 47 - 158t, onde to 4. 100 = 56 + 44 5. 8/13 e 13/17 CAPITULO 9 1. (a) 2. (a) v v (b) Vv (c) F (@)v Ke) FloiGeyrr. ‘ (b) v 373 3. (a) 5 (b) 1 4. 1,2,5,10,71,142,355,710 5. n= 1 (mod.2); n= 2k + 1; n deixa o resto 1 na divi- sao por 2 6. 2,7e12 7. 7, 26,41, 58, 75 e 92 9. 7 = 5* (mod.8) © 7 2 5 (mod.8) 4. 4e6 19. (a) e (c) (b), (c) e (a) Rp 8 21. { 2,3,5,7,11,13,29} ; existem outros. * 22. { 0,4,8,12,16,20,24 } CAPTTULO 10 1. (a) x = 9 (mod.17) (b) x = 6 (mod.17) (c) x = 2,8,14 (mod.18) (4) x = 18 (mod.29) (e) x = 16 (mod.26) (£) x = 6,13,20 (mod. 21) (g) Nao tem solugoes (h) x = 45,94 (mod.98) (i) x = 2,5,8,11 (mod-12) (j) x = 6,30 (mod. 48) 2, (a) x= 15+ Sit, ys-1+ Gt (b) x= 13+ 25t, y=7- lat (c) x= 14+ 53t, ysl+ St (d) x 234+ 6t, y=-2-7t 374 (e) x= 20+27t, y=-8-1lt (f) x = 42 4 131t, y= 24 + 75¢ (g) x= 2t, ys4n-3t (h) x= 8+ 11t, y = 37 + 6lt (i) x= 9+ 77t, y=-5 - 65t (j) x= 2+ 5t, y=1l-3t 3. (a) 3 (b) 1 (c) 5 (a) 0 4. 0, 1, 2, 4, 5, 10 ou 20 CAPITULO 11 1. (a) x = 1 (mod.6) (b) x = 197 (mod.228) (c) x = 242 (mod.858) 2. (a) x = 348 (mod.385) (b) x = 52 (mod.105) (c) x = 785 (mod.1122) (4) x = 4944 (mod.9889) (e) x = 106 (mod.252) (£) x = -1 (mod.9889) (g) x = 40a - 24b ~ 15¢ (mod.120) 3. (a) x = 12113 (mod.20128) (b) x = 1423 (mod.2310) 4. (a) x = 26 (mod.63) (b) x = 148 (mod.210) (c) x = 33(mod.276) 375 CAPITULO 13 10, il. 12. 18. 19, 20. 21, 22. 23. 4,6e8 8, 20, 24 e 12 (a) 15 (b) 24 (c) 2 (d) 24 50 12 24 6300 3150 l+p+qtpqy, lep+pe+p? 2 2 l+ptaq+p +pq+ pq 744, 1344, 3224, 14736 (a) 403 (b) 1170 (c) 1010 = (a):15960 24 e 39 5040, 7920 e 9360 1, 7, 13, 49, 91 e 637 25, 30, 50, 75 e 150 n = 432 n = 1400 (a) x = 22; (b) x = 38, 24, 59 376 CAPITULO 15 1. 96, 720, 1152 e 2160 2 &e2 3. P 10. (a) x = 11 (mod.12) (b) x = 6 (mod.9) (c) x = 3 (mod.10) (4) x = 3 (mod.5) (e) x = 9 (mod.17) (£) x = 1 (mod.8) CAPETULO 16 1. 130816 = 28(29 - 1), e como 2% 1 © 7.73 nio é um pri- mo, segue-se que 130816 nao & um nimero perfeito. 2, 21 1 = 23.89 ndo é um primo. Logo, a ndo @ um niime- ro perfeito. 9. 56 11. s(120) = 3.120 e s(672) = 3.672 17. 12, 18 e 20 sao abundantes, 6 @ perfeito e os demais inteiros s4o deficientes. 18. 25575 = 36.5.7 24, My = 524287 25; 23|m 2 233|Myg 377 CAPITULO 17 10. lle 22. Fis = 610, Fig = 2584, Fog = 6765 Ry - 1 = 1596 Fo, = 46368 Fo, 7 1 = 10945 18. (-1) Fyg + 1 = 988 FiaFis = 229970 Fl ole Fy = 34 Foes e Fy = 144 F,, F. 1? Far Fae Fas Fer Fag Fis Fos Fys Fas Fig 50-5, + F, + Fy IS = Fy + Fo +E, + Big 100 =F, + Fy + Fo + Phy Fi, = 2Fy + Fg, Fly = 6Fg + (Fg - F Lu my CAPITULO 18 1. 2. (724,25), (11,60,61), (17,144,145) (a) (48,20,52); (b) (126,32,130) 3. b= 180 ec = 181 4. b=21 e e329 5. (512,13), (7,24,25), (9,40,41) 6. (16,12,20), (16,63,65), (16,30,39) 7, (16,30,34), (72,30,78), (40,30,50), (224,30, 226) 8. (409,41), (40,399,401) 9. (60,11,61), (60,91,109), (60,221,229), (60,899,901) CAPITULO 19 hy CS5)g oS Lenny “21, ~13, ~5, Sr, ane } 2. Way = 5. = {...,°13, ~4, 5, 14, 23, v1. } 3. m=1,7 4. m=, 2, 3, 4, 642 CAPTTULO 20 1. 890 e 7885 2. 421, 1107, 4159 3+ (20Hgs (435)g 4. (10111010100),, (2001021), (4226), (2037), (1137), 5. (112010), (4223), 6. x= 102, x= 153, x=7 12. 13. 14 17. 19, 379 (34132), (4531), 5 20 « 6763 + 0 * x 0 1 0 0 1 0 9 0 1 1 10 1 0 1 374518 nao @ divisivel por 9 (9128) 1520397 & divisivel por 9 (9|27) 370546 @ divisivel por 11 (11|11) 22481907 nao @ divisivel por 11 aryisy 176521221 & divisivel por 9 9127" e nado @ divisivel por 11 (11}+3) 149235678 @ divisivel por 9 (9|45) e nado @ divisivel por 11 (1149) (a) x= 4; (b) x = 0,3,6,9; (c) x =5 (d) x= 2,8; (e) x=4 (a)x=6,y=5; x=1,y=0 (bo) x=7,y #0; x=3,y=4; x=y=8 (c)x=4,y=0; (4) x=6,y=4 (e)x=3,y=0; x=7,y=5 (f)x=0,y=8; x=8, y=0 380 20. 21. 22. 24, 25. aaaaaa = (1 + 10 + 10% + 103 + 104 + 109 abe — cba = (c + 10b + 100a) - (a + 10b + 100c) = 99(a-c) = 11 x 9a - c) aaa = (1 + 10+ 100) xa=lllxa=37x3xa (2 x ab)ab = 2 x ab x 100 + ab = 201 x ab = 67 x 3 x ab (a) x (b) (c) (a) (e) (£) x x x wl,y=8; x=2,ye4 *S5,y 545 x= 7,y=8; x=8, y= 4 = 2,5,8, y= 0; x =0,3,6,9, y=5 =8,y=0; x=l,y=4; x=4,y=7 =0,y=8; x=8, y=#0 =3,y=2; x=6,y=8 )xas ellllllxa=3x7x11x%13x37 xa 26. ab + ba + ac+ca+be+ch=b+l0ata+l0b+tc+ +lWata+l0c+e+1l0b+ b+ 10c = = 22a + 22b + 22 = 22 (a+b +c) CAPITULO 21 1. (a) 8, 16, 16 (b) 18, 18, 9 (e) 11, 11, 22 2. 4, 2,4, 4, 2, 4, 2 3. 6 e 26 9. 3e7 10. 5, 6, 7, 10, 11, 12, 14 11s 3,10} 93, 44, 15 12, 2e3; 2e5; 2,6,7,e 8; 2,6,7 e 11 EXERCICIOS SUPLEMENTARES 21. 89 22.xs4ey=9 23. a = 62 24. (a) 17; (b) 59; (ce) 1103 29. 1 40. 0 41. 3 42. 1 = 213 382 44. 0 = 5, 8, 12 47. Todo inteiro k>3 50. 17 2 2 2 53. 6° = tg, 35 = t,9, 204) bag, 54. 71 55. pe p’q; 48 = 24.3 56. (a) n = 17,32,34,40,48,60 (b) n = 35,39,45,52,56,70,72,78 84,90 (c) n = 31,62 59, Somente o conjunto (c) 62. n = 33 67. (a) 0, -13, 0, -15, 0 (b) -p (c) 0 (a) -pk 73. p~2e p* = 2p 88. (10°? - on ~ 109/81 39, 2" 90. 13 91. 5 92. (a) a=8,b=4; (b) a 94. 95. 96. 97. 98. 99. 103. 104. 105. 108. 110, 111. 117. 123. 125, 126, 128. 132. 134. 135. 136. 137. 383 x = 250, y = 249 n = 1800 238, 476, 714 e 952 n= 6, 12, 24, 30, 42, 48 n= 45, 90, 180 128 Todos » " nm o 4 16, ova = 1, b = 33 a=6,b=17 2835 127, n=324 ou n=144 1189 (a) 36 (b) 60 (c) 144 (d) 180 (e) 240 (£) 360 16875, (2 x 107 + 1)/3 = (666...67) 5 n= 24 138. 225 e 900 BIBLIOGRAFIA ARMENDARIZ,E, e McADAM,S. Elementary Number Theory; Macmillan; 1980 ANDREWS, G.E. Number Theory; Saunders; 1971 ADAMS,W.W. e@ GOLDSTEIN, L.J. Introduction to Number Theory; Prentice-Hall; 1976 APOSTOL,T.M. Introduccion a la Teoria Analitica de Nameros: Reverté; 1980 u BURTON,D.M. Elementary Number Theory; Allyn and Bacon; 1976 BEILER,A.H. Recreations in the Theory of Numbero; Dover; 1966 DUDLEY,U. Elementary Number Theory; Freeman; 1978 DANTZIG, T. Namero: A Linguagem da Ciéneia: Zahar; 1970 HARDY,G.H. e’ WRIGHT,E.M. Introduction to Theory of Numbers: Oxford: 1979 386 10. il. 12. 13, 14. 15. ITARD, J. Arithmétique et Théorie des Nombres; P.U F.; 1973 LeVEQUE, W.J. Fundamentals of Number Theory; Addison- Wesley; 1977 NIVEN, I. e ZUCKERMAN, H. Introduccion a la Teoria de los Nameros; Limusa; 1976 PETTOFREZZO, A. e BYRKIT, D. Introduccion a la Teoria de los Nimeros; Prentice-Hall; 1972 VOROBYOV, N. Los Nimeros de FIBONACCI; Limusa; 1973 VINOGRADOV, I. Fundamentos de la Teoria de los Niimeros; Mir; 1977 A

Anda mungkin juga menyukai